User login
The Ears Have It (and She Doesn’t Want It)
ANSWER
The correct answer is to obtain a biopsy (choice “d”), the results of which would likely dictate rational and effective treatment. The other choices are largely empirical and not based on available evidence.
DISCUSSION
In this case, the biopsy (with samples from the arm rash as well as from the ears) showed unequivocal evidence of connective tissue disease—almost certainly lupus.
Systemic lupus erythematosus (SLE) has protean manifestations because it can affect so many different organs in so many different ways. Reduced to its simplest elements, lupus is an autoimmune process that results in a form of vasculitis that can affect any perfused tissue.
In terms of the skin, the visible manifestations of lupus are numerous and not always obvious. Sun is a known exacerbating factor, as this case demonstrated quite well: The patient’s rash was pronounced on sun-exposed skin but spared covered skin. When this was brought to her attention, the patient recalled a baby-sitting job earlier in the year (summer) that had required her to spend time outdoors. She also acknowledged that her smoking habit often took her into the backyard, where she would stand in the sun.
That being said, neither the arm rash nor the ear changes are “typical” of lupus (although the latter did include patulous follicular orifices, enlarged pores often seen focally with lupus). The effect is simply the result of the patient’s normal dark skin color; on a white person, the discoloration would have been pink or red.
Although these changes were suspicious for lupus, it was necessary to establish the diagnosis by biopsy—especially since the patient was already being treated for the disease. With that accomplished, the patient was sent back to her rheumatologist, who indicated he would probably treat her with a biologic, plus or minus methotrexate.
ANSWER
The correct answer is to obtain a biopsy (choice “d”), the results of which would likely dictate rational and effective treatment. The other choices are largely empirical and not based on available evidence.
DISCUSSION
In this case, the biopsy (with samples from the arm rash as well as from the ears) showed unequivocal evidence of connective tissue disease—almost certainly lupus.
Systemic lupus erythematosus (SLE) has protean manifestations because it can affect so many different organs in so many different ways. Reduced to its simplest elements, lupus is an autoimmune process that results in a form of vasculitis that can affect any perfused tissue.
In terms of the skin, the visible manifestations of lupus are numerous and not always obvious. Sun is a known exacerbating factor, as this case demonstrated quite well: The patient’s rash was pronounced on sun-exposed skin but spared covered skin. When this was brought to her attention, the patient recalled a baby-sitting job earlier in the year (summer) that had required her to spend time outdoors. She also acknowledged that her smoking habit often took her into the backyard, where she would stand in the sun.
That being said, neither the arm rash nor the ear changes are “typical” of lupus (although the latter did include patulous follicular orifices, enlarged pores often seen focally with lupus). The effect is simply the result of the patient’s normal dark skin color; on a white person, the discoloration would have been pink or red.
Although these changes were suspicious for lupus, it was necessary to establish the diagnosis by biopsy—especially since the patient was already being treated for the disease. With that accomplished, the patient was sent back to her rheumatologist, who indicated he would probably treat her with a biologic, plus or minus methotrexate.
ANSWER
The correct answer is to obtain a biopsy (choice “d”), the results of which would likely dictate rational and effective treatment. The other choices are largely empirical and not based on available evidence.
DISCUSSION
In this case, the biopsy (with samples from the arm rash as well as from the ears) showed unequivocal evidence of connective tissue disease—almost certainly lupus.
Systemic lupus erythematosus (SLE) has protean manifestations because it can affect so many different organs in so many different ways. Reduced to its simplest elements, lupus is an autoimmune process that results in a form of vasculitis that can affect any perfused tissue.
In terms of the skin, the visible manifestations of lupus are numerous and not always obvious. Sun is a known exacerbating factor, as this case demonstrated quite well: The patient’s rash was pronounced on sun-exposed skin but spared covered skin. When this was brought to her attention, the patient recalled a baby-sitting job earlier in the year (summer) that had required her to spend time outdoors. She also acknowledged that her smoking habit often took her into the backyard, where she would stand in the sun.
That being said, neither the arm rash nor the ear changes are “typical” of lupus (although the latter did include patulous follicular orifices, enlarged pores often seen focally with lupus). The effect is simply the result of the patient’s normal dark skin color; on a white person, the discoloration would have been pink or red.
Although these changes were suspicious for lupus, it was necessary to establish the diagnosis by biopsy—especially since the patient was already being treated for the disease. With that accomplished, the patient was sent back to her rheumatologist, who indicated he would probably treat her with a biologic, plus or minus methotrexate.
A 34-year-old black woman is sent to dermatology by her rheumatologist for evaluation of changes to her ears that began several months ago. The patient reports no symptoms, but she is quite distressed by the appearance of her ears. She has been under the care of the rheumatologist for several years for her systemic lupus erythematosus. She takes hydroxychloroquine (400 mg/d), which she says controls most of her systemic symptoms (ie, joint pain and malaise). Further history taking reveals that, within the time frame of the ear changes, the patient also developed an itchy rash on both arms. Application of triamcinolone 0.1% cream has not helped. On examination, the changes to the patient’s ears are immediately obvious: the dark brown to black discoloration contrasts sharply with her light brown skin. In addition to the color change, the surface of the ears is scaly and rough, with enlarged pores evident. There is no redness or swelling noted; palpation elicits neither increased warmth nor adenopathy around the ears or on the adjacent neck. The scaly rash on the patient’s arms is remarkably symmetrical. It affects the sun-exposed lateral portions of both arms, sparing the skin on the medial aspects and on the proximal portions normally covered by clothing.
The simple lab test is sometimes more complex than we think, if we think about it at all
We are all exposed to initiatives from multiple stakeholders telling us to order fewer tests. Many of these efforts to control costs and improve efficiency and quality of care are directed at populations of patients and are broad concepts: provide screening only to those most likely to benefit (eg, don’t screen for prostate cancer in men with a lifespan < 10 years); avoid procedures that provided limited benefit in controlled trials (eg, limit routine arthroscopic treatment of knee osteoarthritis); and avoid reflexive practices unlikely to improve patient outcomes (eg, eliminate routine preoperative testing before elective procedures in otherwise healthy patients).
Whether system-based changes will be implemented and have an impact remains to be determined. But I sense that with all the attention being focused on population management and healthcare practices, including an emphasis on documenting and coding our encounters with patients, whether substantive or simply digital housekeeping, we are increasingly distracted from the patient in front of us and are spending less time reviewing the principles underlying the diagnoses we make and the tests we order—just as we are taking less time to perform relevant physical examinations.1 The latter may mostly relate to time pressures. The former, I believe, is a product of both time pressures and a false sense of confidence in our knowledge of seemingly commonplace laboratory tests.
As I lecture, work with trainees, and reflect on my own patients, I realize that we are slowly but progressively minimizing the importance of a working knowledge of the basic foundations of clinical practice—perhaps because facts can always be looked up. I am not referring to knowledge of arcane biochemical pathways, eponymous references, or the latest recommended treatment of inclusion body myositis. I am thinking instead of the value of regularly refreshing our knowledge of laboratory tests and diagnoses we frequently encounter.
Having access to multiple clinical databases literally in our pockets is likely bolstering a false sense of confidence in our knowledge. The National Library of Medicine may be only a tap on a smart phone away, but accessing it regularly is a different thing. Attending conferences and reading educational journals help to keep our broad-based knowledge of internal medicine refreshed, but time pressures may significantly limit our ability to regularly pursue these activities.
In this issue of the Journal, Drs. Moghadam-Kia et al discuss their approach to asymptomatic elevations in creatine kinase (CK). Although no longer included in the most commonly used lab panels, CK measurement is often ordered in patients taking statins, even if they have no relevant muscle-related symptoms. Thus, evaluating a patient with an asymptomatic elevated CK level is not rare. The authors delve into the clinically relevant test characteristics, and their important caveats about interpreting elevated CK levels are germane not only to the asymptomatic patient, but also to the patient being evaluated for myalgia or weakness. This latter situation is one I frequently face in both the hospital and the outpatient clinic. I am often asked to consult on patients who have incompletely defined symptoms and elevated CK.
As discussed in the article, the laboratory definition of “normal” must first be considered. Laboratory test results must always be interpreted in the clinical context. An isolated elevation in parathyroid hormone cannot be interpreted without knowing the patient’s vitamin D level. Nor can “normal” low-density lipoprotein or serum urate levels be interpreted properly without knowing if the patient is accumulating excess cholesterol or urate deposits. As we order and interpret test results, we must consider the biology of the substance being measured as well as the test characteristics; too often, we react to abnormal laboratory results with an incomplete understanding of these aspects.
Moghadam-Kia et al do not dwell on the organ involvement causing CK elevations, but specificity is another very important aspect when clinically interpreting the results of a CK test. Many patients with muscle damage or inflammation have elevations in serum aspartate aminotransferase and alanine aminotransferase levels (the ratio of elevation depends on the time course of the muscle damage and on the relative clearance rate of the two enzymes). Without knowing that the CK is elevated, one might assume that an aminotransferase elevation reflects hepatitis. I have seen several patients with elevated aminotransferases and complaints of weakness and fatigue who were subjected to liver biopsy before it was recognized that the source of the enzyme elevation (“liver function test changes”) was muscle (or hemolysis). Frequently unrecognized is that aldolase, which has a cell distribution similar to that of lactate dehydrogenase, does not have the relative specificity of localization to muscle that CK has. CK is quite useful in distinguishing myocyte from hepatocyte damage.
This paper presents a wonderful reminder of the value of updating and reviewing what we know about tests that we order, even if we feel comfortable when ordering them. Before initiating a cascade of additional tests and consultations to explore the cause of an abnormal test result, a little time spent reviewing its basic characteristics and biology may pay dividends.
As 2015 comes to a close, we at the Journal share with you our sincere wishes for personal satisfaction and a globally more peaceful 2016.
- Verghese A, Charlton B, Kassirer JP, Ramsey M, Ioannidis JP. Inadequacies of physical examination as a cause of medical errors and adverse events: a collection of vignettes. Am J Med 2015; 128:1322–1324.
We are all exposed to initiatives from multiple stakeholders telling us to order fewer tests. Many of these efforts to control costs and improve efficiency and quality of care are directed at populations of patients and are broad concepts: provide screening only to those most likely to benefit (eg, don’t screen for prostate cancer in men with a lifespan < 10 years); avoid procedures that provided limited benefit in controlled trials (eg, limit routine arthroscopic treatment of knee osteoarthritis); and avoid reflexive practices unlikely to improve patient outcomes (eg, eliminate routine preoperative testing before elective procedures in otherwise healthy patients).
Whether system-based changes will be implemented and have an impact remains to be determined. But I sense that with all the attention being focused on population management and healthcare practices, including an emphasis on documenting and coding our encounters with patients, whether substantive or simply digital housekeeping, we are increasingly distracted from the patient in front of us and are spending less time reviewing the principles underlying the diagnoses we make and the tests we order—just as we are taking less time to perform relevant physical examinations.1 The latter may mostly relate to time pressures. The former, I believe, is a product of both time pressures and a false sense of confidence in our knowledge of seemingly commonplace laboratory tests.
As I lecture, work with trainees, and reflect on my own patients, I realize that we are slowly but progressively minimizing the importance of a working knowledge of the basic foundations of clinical practice—perhaps because facts can always be looked up. I am not referring to knowledge of arcane biochemical pathways, eponymous references, or the latest recommended treatment of inclusion body myositis. I am thinking instead of the value of regularly refreshing our knowledge of laboratory tests and diagnoses we frequently encounter.
Having access to multiple clinical databases literally in our pockets is likely bolstering a false sense of confidence in our knowledge. The National Library of Medicine may be only a tap on a smart phone away, but accessing it regularly is a different thing. Attending conferences and reading educational journals help to keep our broad-based knowledge of internal medicine refreshed, but time pressures may significantly limit our ability to regularly pursue these activities.
In this issue of the Journal, Drs. Moghadam-Kia et al discuss their approach to asymptomatic elevations in creatine kinase (CK). Although no longer included in the most commonly used lab panels, CK measurement is often ordered in patients taking statins, even if they have no relevant muscle-related symptoms. Thus, evaluating a patient with an asymptomatic elevated CK level is not rare. The authors delve into the clinically relevant test characteristics, and their important caveats about interpreting elevated CK levels are germane not only to the asymptomatic patient, but also to the patient being evaluated for myalgia or weakness. This latter situation is one I frequently face in both the hospital and the outpatient clinic. I am often asked to consult on patients who have incompletely defined symptoms and elevated CK.
As discussed in the article, the laboratory definition of “normal” must first be considered. Laboratory test results must always be interpreted in the clinical context. An isolated elevation in parathyroid hormone cannot be interpreted without knowing the patient’s vitamin D level. Nor can “normal” low-density lipoprotein or serum urate levels be interpreted properly without knowing if the patient is accumulating excess cholesterol or urate deposits. As we order and interpret test results, we must consider the biology of the substance being measured as well as the test characteristics; too often, we react to abnormal laboratory results with an incomplete understanding of these aspects.
Moghadam-Kia et al do not dwell on the organ involvement causing CK elevations, but specificity is another very important aspect when clinically interpreting the results of a CK test. Many patients with muscle damage or inflammation have elevations in serum aspartate aminotransferase and alanine aminotransferase levels (the ratio of elevation depends on the time course of the muscle damage and on the relative clearance rate of the two enzymes). Without knowing that the CK is elevated, one might assume that an aminotransferase elevation reflects hepatitis. I have seen several patients with elevated aminotransferases and complaints of weakness and fatigue who were subjected to liver biopsy before it was recognized that the source of the enzyme elevation (“liver function test changes”) was muscle (or hemolysis). Frequently unrecognized is that aldolase, which has a cell distribution similar to that of lactate dehydrogenase, does not have the relative specificity of localization to muscle that CK has. CK is quite useful in distinguishing myocyte from hepatocyte damage.
This paper presents a wonderful reminder of the value of updating and reviewing what we know about tests that we order, even if we feel comfortable when ordering them. Before initiating a cascade of additional tests and consultations to explore the cause of an abnormal test result, a little time spent reviewing its basic characteristics and biology may pay dividends.
As 2015 comes to a close, we at the Journal share with you our sincere wishes for personal satisfaction and a globally more peaceful 2016.
We are all exposed to initiatives from multiple stakeholders telling us to order fewer tests. Many of these efforts to control costs and improve efficiency and quality of care are directed at populations of patients and are broad concepts: provide screening only to those most likely to benefit (eg, don’t screen for prostate cancer in men with a lifespan < 10 years); avoid procedures that provided limited benefit in controlled trials (eg, limit routine arthroscopic treatment of knee osteoarthritis); and avoid reflexive practices unlikely to improve patient outcomes (eg, eliminate routine preoperative testing before elective procedures in otherwise healthy patients).
Whether system-based changes will be implemented and have an impact remains to be determined. But I sense that with all the attention being focused on population management and healthcare practices, including an emphasis on documenting and coding our encounters with patients, whether substantive or simply digital housekeeping, we are increasingly distracted from the patient in front of us and are spending less time reviewing the principles underlying the diagnoses we make and the tests we order—just as we are taking less time to perform relevant physical examinations.1 The latter may mostly relate to time pressures. The former, I believe, is a product of both time pressures and a false sense of confidence in our knowledge of seemingly commonplace laboratory tests.
As I lecture, work with trainees, and reflect on my own patients, I realize that we are slowly but progressively minimizing the importance of a working knowledge of the basic foundations of clinical practice—perhaps because facts can always be looked up. I am not referring to knowledge of arcane biochemical pathways, eponymous references, or the latest recommended treatment of inclusion body myositis. I am thinking instead of the value of regularly refreshing our knowledge of laboratory tests and diagnoses we frequently encounter.
Having access to multiple clinical databases literally in our pockets is likely bolstering a false sense of confidence in our knowledge. The National Library of Medicine may be only a tap on a smart phone away, but accessing it regularly is a different thing. Attending conferences and reading educational journals help to keep our broad-based knowledge of internal medicine refreshed, but time pressures may significantly limit our ability to regularly pursue these activities.
In this issue of the Journal, Drs. Moghadam-Kia et al discuss their approach to asymptomatic elevations in creatine kinase (CK). Although no longer included in the most commonly used lab panels, CK measurement is often ordered in patients taking statins, even if they have no relevant muscle-related symptoms. Thus, evaluating a patient with an asymptomatic elevated CK level is not rare. The authors delve into the clinically relevant test characteristics, and their important caveats about interpreting elevated CK levels are germane not only to the asymptomatic patient, but also to the patient being evaluated for myalgia or weakness. This latter situation is one I frequently face in both the hospital and the outpatient clinic. I am often asked to consult on patients who have incompletely defined symptoms and elevated CK.
As discussed in the article, the laboratory definition of “normal” must first be considered. Laboratory test results must always be interpreted in the clinical context. An isolated elevation in parathyroid hormone cannot be interpreted without knowing the patient’s vitamin D level. Nor can “normal” low-density lipoprotein or serum urate levels be interpreted properly without knowing if the patient is accumulating excess cholesterol or urate deposits. As we order and interpret test results, we must consider the biology of the substance being measured as well as the test characteristics; too often, we react to abnormal laboratory results with an incomplete understanding of these aspects.
Moghadam-Kia et al do not dwell on the organ involvement causing CK elevations, but specificity is another very important aspect when clinically interpreting the results of a CK test. Many patients with muscle damage or inflammation have elevations in serum aspartate aminotransferase and alanine aminotransferase levels (the ratio of elevation depends on the time course of the muscle damage and on the relative clearance rate of the two enzymes). Without knowing that the CK is elevated, one might assume that an aminotransferase elevation reflects hepatitis. I have seen several patients with elevated aminotransferases and complaints of weakness and fatigue who were subjected to liver biopsy before it was recognized that the source of the enzyme elevation (“liver function test changes”) was muscle (or hemolysis). Frequently unrecognized is that aldolase, which has a cell distribution similar to that of lactate dehydrogenase, does not have the relative specificity of localization to muscle that CK has. CK is quite useful in distinguishing myocyte from hepatocyte damage.
This paper presents a wonderful reminder of the value of updating and reviewing what we know about tests that we order, even if we feel comfortable when ordering them. Before initiating a cascade of additional tests and consultations to explore the cause of an abnormal test result, a little time spent reviewing its basic characteristics and biology may pay dividends.
As 2015 comes to a close, we at the Journal share with you our sincere wishes for personal satisfaction and a globally more peaceful 2016.
- Verghese A, Charlton B, Kassirer JP, Ramsey M, Ioannidis JP. Inadequacies of physical examination as a cause of medical errors and adverse events: a collection of vignettes. Am J Med 2015; 128:1322–1324.
- Verghese A, Charlton B, Kassirer JP, Ramsey M, Ioannidis JP. Inadequacies of physical examination as a cause of medical errors and adverse events: a collection of vignettes. Am J Med 2015; 128:1322–1324.
Obstructive sleep apnea: Who should be tested, and how?
Patients who have risk factors for obstructive sleep apnea (OSA) or who report symptoms of OSA should be screened for it, first with a complete sleep history and standardized questionnaire, and then by objective testing if indicated. The gold standard test for OSA is polysomnography performed overnight in a sleep laboratory. Home testing is an option in certain instances.
Common risk factors include obesity, resistant hypertension, retrognathia, large neck circumference (> 17 inches in men, > 16 inches in women), and history of stroke, atrial fibrillation, nocturnal arrhythmias, heart failure, and pulmonary hypertension. Screening is also recommended for any patient who is found on physical examination to have upper-airway narrowing or who reports symptoms such as loud snoring, observed episodes of apnea, gasping or choking at night, unrefreshing sleep, morning headaches, unexplained fatigue, and excessive tiredness during the day.
The American Academy of Sleep Medicine suggests three opportunities to screen for OSA1:
- At routine health maintenance visits
- If the patient reports clinical symptoms of OSA
- If the patient has risk factors.
A DISMAL STATISTIC
The prevalence of OSA in the United States is high, estimated to be 2% in women and 4% in men in the middle-aged work force,2 and even more in blacks, Asians, and older adults.3 Yet only 10% of people with OSA are diagnosed4—a dismal statistic considering the association of OSA with resistant hypertension5 and with a greater risk of stroke,6 cardiovascular disease, and death.7
CONSEQUENCES OF UNTREATED OSA
Untreated OSA is associated with a number of conditions7:
- Hypertension. OSA is one of the most common conditions associated with resistant hypertension. Patients with severe OSA and resistant hypertension who comply with continuous positive airway pressure (CPAP) treatment have significant reductions in blood pressure.
- Coronary artery disease. OSA is twice as common in people with coronary artery disease as in those with no coronary artery disease. In patients with coronary artery disease and OSA, CPAP may reduce the rate of nonfatal and fatal cardiovascular events.
- Heart failure. OSA is common in patients with systolic dysfunction (11% to 37%). OSA also has been detected in more than 50% of patients with heart failure with preserved systolic function. CPAP treatment can improve ejection fraction in patients with systolic dysfunction.
- Arrythmias. Atrial fibrillation, nonsustained ventricular tachycardia, and complex ventricular ectopy have been reported to be significantly more common in people with OSA.8 If the underlying cardiac conduction system is normal and there is no significant thyroid dysfunction, bradyarrhythmias and heart block may be treated effectively with CPAP.7 Treatment of OSA may decrease the incidence and severity of ventricular arrhythmias.7
- Sudden cardiac death. OSA was independently associated with sudden cardiac death in a longitudinal study.9
- Stroke. The Sleep Heart Health Study6 showed that OSA is 30% more common in patients who developed ischemic stroke. Long-term CPAP treatment in moderate to severe OSA and ischemic stroke is associated with a reduction in the mortality rate.10
- Diabetes. The Sleep Heart Health Study showed that OSA is independently associated with glucose intolerance and insulin resistance and may lead to type 2 diabetes mellitus.11
A QUESTIONNAIRE HELPS IDENTIFY WHO NEEDS TESTING
If you suspect OSA, consider administering a sleep disorder questionnaire such as the Berlin,12 the Epworth Sleepiness Scale, or the STOP-Bang questionnaire (Table 1). The STOP-Bang questionnaire is an easy-to-use tool that expands on the STOP questionnaire (snoring, tiredness, observed apnea, high blood pressure) with the addition of body mass index, age, neck size, and gender. The Berlin questionnaire has been validated in the primary care setting.12 The STOP-Bang questionnaire has been validated in preoperative settings13 but not in the primary care setting (although it has been commonly used in primary care).
WHICH TEST TO ORDER?
If the score on the questionnaire indicates a moderate or high risk of OSA, the patient should undergo objective testing with polysomnography or, in certain instances, home testing.1 Polysomnography is the gold standard. Home testing costs less and is easier to arrange, but the American Academy of Sleep Medicine recommends it as an alternative to polysomnography, in conjunction with a comprehensive sleep evaluation, only in the following situations14:
- If the patient has a high pretest probability of moderate to severe OSA
- If immobility or critical illness makes polysomnography unfeasible
- If direct monitoring of the response to non-CPAP treatments for sleep apnea is needed.
Home testing for OSA should not be used in the following situations:
- If the patient has significant morbidity such as moderate to severe pulmonary disease, neuromuscular disease, or congestive heart failure
- In evaluating a patient suspected of having comorbid sleep disorders such as central sleep apnea, periodic limb movement disorder, insomnia, parasomnias, circadian rhythm disorder, or narcolepsy
- In screening of asymptomatic patients.
Home testing has important drawbacks. It may underestimate the severity of sleep apnea. The rate of false-negative results may be as high as 17%. If the home test was thought to be technically inadequate or the results were inconsistent with those that were expected—ie, if the patient has a high pretest probability of OSA based on risk factors or symptoms but negative results on home testing—then the patient should undergo polysomnography.14
DIAGNOSIS
The diagnosis of OSA is confirmed if the number of apnea events per hour (ie, the apnea-hypopnea index) on polysomnography or home testing is more than 15, regardless of symptoms, or more than 5 in a patient who reports OSA symptoms. An apnea-hypopnea index of 5 to 14 indicates mild OSA, 15 to 30 indicates moderate OSA, and greater than 30 indicates severe OSA.
BENEFITS OF TREATMENT
Treatment of OSA with CPAP reduces the 10-year risk of fatal and nonfatal motor vehicle accidents by 52%, the 10-year expected number of myocardial infarctions by 49%, and the 10-year risk of stroke by 31%.7 It has also been found to be cost-effective, for men and women of all ages with moderate to severe OSA.15
- Epstein LJ, Kristo D, Strollo PJ Jr, et al; Adult Obstructive Sleep Apnea Task Force of the American Academy of Sleep Medicine. Clinical guideline for the evaluation, management and long-term care of obstructive sleep apnea in adults. J Clin Sleep Med 2009; 5:263–276.
- Young T, Palta M, Dempsey J, Skatrud J, Weber S, Badr S. The occurrence of sleep-disordered breathing among middle-aged adults. N Engl J Med 1993; 328:1230–1235.
- Punjabi NM. The epidemiology of adult obstructive sleep apnea. Proc Am Thorac Soc 2008; 5:136–143.
- Young T, Evans L, Finn L, Palta M. Estimation of the clinically diagnosed proportion of sleep apnea syndrome in middle-aged men and women. Sleep 1997; 20:705–706.
- Pedrosa RP, Drager LF, Gonzaga CC, et al. Obstructive sleep apnea: the most common secondary cause of hypertension associated with resistant hypertension. Hypertension 2011; 58:811–817.
- Redline S, Yenokyan G, Gottlieb DJ, et al. Obstructive sleep apnea-hypopnea and incident stroke: the Sleep Heart Health Study. Am J Respir Crit Care Med 2010; 182:269–277.
- Somers VK, White DP, Amin R, et al; American Heart Association Council for High Blood Pressure Research Professional Education Committee, Council on Clinical Cardiology; American Heart Association Stroke Council; American Heart Association Council on Cardiovascular Nursing; American College of Cardiology Foundation. Sleep apnea and cardiovascular disease: an American Heart Association/American College Of Cardiology Foundation Scientific Statement from the American Heart Association Council for High Blood Pressure Research Professional Education Committee, Council on Clinical Cardiology, Stroke Council, and Council On Cardiovascular Nursing. in collaboration with the National Heart, Lung, and Blood Institute National Center on Sleep Disorders Research (National Institutes of Health). Circulation 2008; 118:1080–1111.
- Mehra R, Benjamin EJ, Shahar E, et al; Sleep Heart Health Study. Association of nocturnal arrhythmias with sleep-disordered breathing: the Sleep Heart Health Study. Am J Respir Crit Care Med 2006; 173:910–916.
- Gami AS, Olson EJ, Shen WK, et al. Obstructive sleep apnea and the risk of sudden cardiac death: a longitudinal study of 10,701 adults. J Am Coll Cardiol 2013; 62:610–616.
- Martinez-Garcia MA, Soler-Cataluna JJ, Ejarque-Martinez L, et al. Continuous positive airway pressure treatment reduces mortality in patients with ischemic stroke and obstructive sleep apnea: a 5-year follow-up study. Am J Respir Crit Care Med 2009; 180:36–41.
- Punjabi NM, Shahar E, Redline S, Gottlieb DJ, Givelber R, Resnick HE; Sleep Heart Health Study Investigators. Sleep-disordered breathing, glucose intolerance, and insulin resistance: The Sleep Heart Health Study. Am J Epidemiol 2004; 160:521–530.
- Netzer NC, Hoegel JJ, Loube D, et al; Sleep in Primary Care International Study Group. Prevalence of symptoms and risk of sleep apnea in primary care. Chest 2003; 124:1406–1414.
- Chung F, Yegneswaran B, Liao P, et al. STOP questionnaire: a tool to screen patients for obstructive sleep apnea. Anesthesiology 2008; 108:812–821.
- Collop NA, Anderson WM, Boehlecke B, et al; Portable Monitoring Task Force of the American Academy of Sleep Medicine. Clinical guidelines for the use of unattended portable monitors in the diagnosis of obstructive sleep apnea in adult patients. Portable Monitoring Task Force of the American Academy of Sleep Medicine. J Clin Sleep Med 2007; 3:737–747.
- Pietzsch JB, Garner A, Cipriano LE, Linehan JH. An integrated health-economic analysis of diagnostic and therapeutic strategies in the treatment of moderate-to-severe obstructive sleep apnea. Sleep 2011; 34:695–709.
Patients who have risk factors for obstructive sleep apnea (OSA) or who report symptoms of OSA should be screened for it, first with a complete sleep history and standardized questionnaire, and then by objective testing if indicated. The gold standard test for OSA is polysomnography performed overnight in a sleep laboratory. Home testing is an option in certain instances.
Common risk factors include obesity, resistant hypertension, retrognathia, large neck circumference (> 17 inches in men, > 16 inches in women), and history of stroke, atrial fibrillation, nocturnal arrhythmias, heart failure, and pulmonary hypertension. Screening is also recommended for any patient who is found on physical examination to have upper-airway narrowing or who reports symptoms such as loud snoring, observed episodes of apnea, gasping or choking at night, unrefreshing sleep, morning headaches, unexplained fatigue, and excessive tiredness during the day.
The American Academy of Sleep Medicine suggests three opportunities to screen for OSA1:
- At routine health maintenance visits
- If the patient reports clinical symptoms of OSA
- If the patient has risk factors.
A DISMAL STATISTIC
The prevalence of OSA in the United States is high, estimated to be 2% in women and 4% in men in the middle-aged work force,2 and even more in blacks, Asians, and older adults.3 Yet only 10% of people with OSA are diagnosed4—a dismal statistic considering the association of OSA with resistant hypertension5 and with a greater risk of stroke,6 cardiovascular disease, and death.7
CONSEQUENCES OF UNTREATED OSA
Untreated OSA is associated with a number of conditions7:
- Hypertension. OSA is one of the most common conditions associated with resistant hypertension. Patients with severe OSA and resistant hypertension who comply with continuous positive airway pressure (CPAP) treatment have significant reductions in blood pressure.
- Coronary artery disease. OSA is twice as common in people with coronary artery disease as in those with no coronary artery disease. In patients with coronary artery disease and OSA, CPAP may reduce the rate of nonfatal and fatal cardiovascular events.
- Heart failure. OSA is common in patients with systolic dysfunction (11% to 37%). OSA also has been detected in more than 50% of patients with heart failure with preserved systolic function. CPAP treatment can improve ejection fraction in patients with systolic dysfunction.
- Arrythmias. Atrial fibrillation, nonsustained ventricular tachycardia, and complex ventricular ectopy have been reported to be significantly more common in people with OSA.8 If the underlying cardiac conduction system is normal and there is no significant thyroid dysfunction, bradyarrhythmias and heart block may be treated effectively with CPAP.7 Treatment of OSA may decrease the incidence and severity of ventricular arrhythmias.7
- Sudden cardiac death. OSA was independently associated with sudden cardiac death in a longitudinal study.9
- Stroke. The Sleep Heart Health Study6 showed that OSA is 30% more common in patients who developed ischemic stroke. Long-term CPAP treatment in moderate to severe OSA and ischemic stroke is associated with a reduction in the mortality rate.10
- Diabetes. The Sleep Heart Health Study showed that OSA is independently associated with glucose intolerance and insulin resistance and may lead to type 2 diabetes mellitus.11
A QUESTIONNAIRE HELPS IDENTIFY WHO NEEDS TESTING
If you suspect OSA, consider administering a sleep disorder questionnaire such as the Berlin,12 the Epworth Sleepiness Scale, or the STOP-Bang questionnaire (Table 1). The STOP-Bang questionnaire is an easy-to-use tool that expands on the STOP questionnaire (snoring, tiredness, observed apnea, high blood pressure) with the addition of body mass index, age, neck size, and gender. The Berlin questionnaire has been validated in the primary care setting.12 The STOP-Bang questionnaire has been validated in preoperative settings13 but not in the primary care setting (although it has been commonly used in primary care).
WHICH TEST TO ORDER?
If the score on the questionnaire indicates a moderate or high risk of OSA, the patient should undergo objective testing with polysomnography or, in certain instances, home testing.1 Polysomnography is the gold standard. Home testing costs less and is easier to arrange, but the American Academy of Sleep Medicine recommends it as an alternative to polysomnography, in conjunction with a comprehensive sleep evaluation, only in the following situations14:
- If the patient has a high pretest probability of moderate to severe OSA
- If immobility or critical illness makes polysomnography unfeasible
- If direct monitoring of the response to non-CPAP treatments for sleep apnea is needed.
Home testing for OSA should not be used in the following situations:
- If the patient has significant morbidity such as moderate to severe pulmonary disease, neuromuscular disease, or congestive heart failure
- In evaluating a patient suspected of having comorbid sleep disorders such as central sleep apnea, periodic limb movement disorder, insomnia, parasomnias, circadian rhythm disorder, or narcolepsy
- In screening of asymptomatic patients.
Home testing has important drawbacks. It may underestimate the severity of sleep apnea. The rate of false-negative results may be as high as 17%. If the home test was thought to be technically inadequate or the results were inconsistent with those that were expected—ie, if the patient has a high pretest probability of OSA based on risk factors or symptoms but negative results on home testing—then the patient should undergo polysomnography.14
DIAGNOSIS
The diagnosis of OSA is confirmed if the number of apnea events per hour (ie, the apnea-hypopnea index) on polysomnography or home testing is more than 15, regardless of symptoms, or more than 5 in a patient who reports OSA symptoms. An apnea-hypopnea index of 5 to 14 indicates mild OSA, 15 to 30 indicates moderate OSA, and greater than 30 indicates severe OSA.
BENEFITS OF TREATMENT
Treatment of OSA with CPAP reduces the 10-year risk of fatal and nonfatal motor vehicle accidents by 52%, the 10-year expected number of myocardial infarctions by 49%, and the 10-year risk of stroke by 31%.7 It has also been found to be cost-effective, for men and women of all ages with moderate to severe OSA.15
Patients who have risk factors for obstructive sleep apnea (OSA) or who report symptoms of OSA should be screened for it, first with a complete sleep history and standardized questionnaire, and then by objective testing if indicated. The gold standard test for OSA is polysomnography performed overnight in a sleep laboratory. Home testing is an option in certain instances.
Common risk factors include obesity, resistant hypertension, retrognathia, large neck circumference (> 17 inches in men, > 16 inches in women), and history of stroke, atrial fibrillation, nocturnal arrhythmias, heart failure, and pulmonary hypertension. Screening is also recommended for any patient who is found on physical examination to have upper-airway narrowing or who reports symptoms such as loud snoring, observed episodes of apnea, gasping or choking at night, unrefreshing sleep, morning headaches, unexplained fatigue, and excessive tiredness during the day.
The American Academy of Sleep Medicine suggests three opportunities to screen for OSA1:
- At routine health maintenance visits
- If the patient reports clinical symptoms of OSA
- If the patient has risk factors.
A DISMAL STATISTIC
The prevalence of OSA in the United States is high, estimated to be 2% in women and 4% in men in the middle-aged work force,2 and even more in blacks, Asians, and older adults.3 Yet only 10% of people with OSA are diagnosed4—a dismal statistic considering the association of OSA with resistant hypertension5 and with a greater risk of stroke,6 cardiovascular disease, and death.7
CONSEQUENCES OF UNTREATED OSA
Untreated OSA is associated with a number of conditions7:
- Hypertension. OSA is one of the most common conditions associated with resistant hypertension. Patients with severe OSA and resistant hypertension who comply with continuous positive airway pressure (CPAP) treatment have significant reductions in blood pressure.
- Coronary artery disease. OSA is twice as common in people with coronary artery disease as in those with no coronary artery disease. In patients with coronary artery disease and OSA, CPAP may reduce the rate of nonfatal and fatal cardiovascular events.
- Heart failure. OSA is common in patients with systolic dysfunction (11% to 37%). OSA also has been detected in more than 50% of patients with heart failure with preserved systolic function. CPAP treatment can improve ejection fraction in patients with systolic dysfunction.
- Arrythmias. Atrial fibrillation, nonsustained ventricular tachycardia, and complex ventricular ectopy have been reported to be significantly more common in people with OSA.8 If the underlying cardiac conduction system is normal and there is no significant thyroid dysfunction, bradyarrhythmias and heart block may be treated effectively with CPAP.7 Treatment of OSA may decrease the incidence and severity of ventricular arrhythmias.7
- Sudden cardiac death. OSA was independently associated with sudden cardiac death in a longitudinal study.9
- Stroke. The Sleep Heart Health Study6 showed that OSA is 30% more common in patients who developed ischemic stroke. Long-term CPAP treatment in moderate to severe OSA and ischemic stroke is associated with a reduction in the mortality rate.10
- Diabetes. The Sleep Heart Health Study showed that OSA is independently associated with glucose intolerance and insulin resistance and may lead to type 2 diabetes mellitus.11
A QUESTIONNAIRE HELPS IDENTIFY WHO NEEDS TESTING
If you suspect OSA, consider administering a sleep disorder questionnaire such as the Berlin,12 the Epworth Sleepiness Scale, or the STOP-Bang questionnaire (Table 1). The STOP-Bang questionnaire is an easy-to-use tool that expands on the STOP questionnaire (snoring, tiredness, observed apnea, high blood pressure) with the addition of body mass index, age, neck size, and gender. The Berlin questionnaire has been validated in the primary care setting.12 The STOP-Bang questionnaire has been validated in preoperative settings13 but not in the primary care setting (although it has been commonly used in primary care).
WHICH TEST TO ORDER?
If the score on the questionnaire indicates a moderate or high risk of OSA, the patient should undergo objective testing with polysomnography or, in certain instances, home testing.1 Polysomnography is the gold standard. Home testing costs less and is easier to arrange, but the American Academy of Sleep Medicine recommends it as an alternative to polysomnography, in conjunction with a comprehensive sleep evaluation, only in the following situations14:
- If the patient has a high pretest probability of moderate to severe OSA
- If immobility or critical illness makes polysomnography unfeasible
- If direct monitoring of the response to non-CPAP treatments for sleep apnea is needed.
Home testing for OSA should not be used in the following situations:
- If the patient has significant morbidity such as moderate to severe pulmonary disease, neuromuscular disease, or congestive heart failure
- In evaluating a patient suspected of having comorbid sleep disorders such as central sleep apnea, periodic limb movement disorder, insomnia, parasomnias, circadian rhythm disorder, or narcolepsy
- In screening of asymptomatic patients.
Home testing has important drawbacks. It may underestimate the severity of sleep apnea. The rate of false-negative results may be as high as 17%. If the home test was thought to be technically inadequate or the results were inconsistent with those that were expected—ie, if the patient has a high pretest probability of OSA based on risk factors or symptoms but negative results on home testing—then the patient should undergo polysomnography.14
DIAGNOSIS
The diagnosis of OSA is confirmed if the number of apnea events per hour (ie, the apnea-hypopnea index) on polysomnography or home testing is more than 15, regardless of symptoms, or more than 5 in a patient who reports OSA symptoms. An apnea-hypopnea index of 5 to 14 indicates mild OSA, 15 to 30 indicates moderate OSA, and greater than 30 indicates severe OSA.
BENEFITS OF TREATMENT
Treatment of OSA with CPAP reduces the 10-year risk of fatal and nonfatal motor vehicle accidents by 52%, the 10-year expected number of myocardial infarctions by 49%, and the 10-year risk of stroke by 31%.7 It has also been found to be cost-effective, for men and women of all ages with moderate to severe OSA.15
- Epstein LJ, Kristo D, Strollo PJ Jr, et al; Adult Obstructive Sleep Apnea Task Force of the American Academy of Sleep Medicine. Clinical guideline for the evaluation, management and long-term care of obstructive sleep apnea in adults. J Clin Sleep Med 2009; 5:263–276.
- Young T, Palta M, Dempsey J, Skatrud J, Weber S, Badr S. The occurrence of sleep-disordered breathing among middle-aged adults. N Engl J Med 1993; 328:1230–1235.
- Punjabi NM. The epidemiology of adult obstructive sleep apnea. Proc Am Thorac Soc 2008; 5:136–143.
- Young T, Evans L, Finn L, Palta M. Estimation of the clinically diagnosed proportion of sleep apnea syndrome in middle-aged men and women. Sleep 1997; 20:705–706.
- Pedrosa RP, Drager LF, Gonzaga CC, et al. Obstructive sleep apnea: the most common secondary cause of hypertension associated with resistant hypertension. Hypertension 2011; 58:811–817.
- Redline S, Yenokyan G, Gottlieb DJ, et al. Obstructive sleep apnea-hypopnea and incident stroke: the Sleep Heart Health Study. Am J Respir Crit Care Med 2010; 182:269–277.
- Somers VK, White DP, Amin R, et al; American Heart Association Council for High Blood Pressure Research Professional Education Committee, Council on Clinical Cardiology; American Heart Association Stroke Council; American Heart Association Council on Cardiovascular Nursing; American College of Cardiology Foundation. Sleep apnea and cardiovascular disease: an American Heart Association/American College Of Cardiology Foundation Scientific Statement from the American Heart Association Council for High Blood Pressure Research Professional Education Committee, Council on Clinical Cardiology, Stroke Council, and Council On Cardiovascular Nursing. in collaboration with the National Heart, Lung, and Blood Institute National Center on Sleep Disorders Research (National Institutes of Health). Circulation 2008; 118:1080–1111.
- Mehra R, Benjamin EJ, Shahar E, et al; Sleep Heart Health Study. Association of nocturnal arrhythmias with sleep-disordered breathing: the Sleep Heart Health Study. Am J Respir Crit Care Med 2006; 173:910–916.
- Gami AS, Olson EJ, Shen WK, et al. Obstructive sleep apnea and the risk of sudden cardiac death: a longitudinal study of 10,701 adults. J Am Coll Cardiol 2013; 62:610–616.
- Martinez-Garcia MA, Soler-Cataluna JJ, Ejarque-Martinez L, et al. Continuous positive airway pressure treatment reduces mortality in patients with ischemic stroke and obstructive sleep apnea: a 5-year follow-up study. Am J Respir Crit Care Med 2009; 180:36–41.
- Punjabi NM, Shahar E, Redline S, Gottlieb DJ, Givelber R, Resnick HE; Sleep Heart Health Study Investigators. Sleep-disordered breathing, glucose intolerance, and insulin resistance: The Sleep Heart Health Study. Am J Epidemiol 2004; 160:521–530.
- Netzer NC, Hoegel JJ, Loube D, et al; Sleep in Primary Care International Study Group. Prevalence of symptoms and risk of sleep apnea in primary care. Chest 2003; 124:1406–1414.
- Chung F, Yegneswaran B, Liao P, et al. STOP questionnaire: a tool to screen patients for obstructive sleep apnea. Anesthesiology 2008; 108:812–821.
- Collop NA, Anderson WM, Boehlecke B, et al; Portable Monitoring Task Force of the American Academy of Sleep Medicine. Clinical guidelines for the use of unattended portable monitors in the diagnosis of obstructive sleep apnea in adult patients. Portable Monitoring Task Force of the American Academy of Sleep Medicine. J Clin Sleep Med 2007; 3:737–747.
- Pietzsch JB, Garner A, Cipriano LE, Linehan JH. An integrated health-economic analysis of diagnostic and therapeutic strategies in the treatment of moderate-to-severe obstructive sleep apnea. Sleep 2011; 34:695–709.
- Epstein LJ, Kristo D, Strollo PJ Jr, et al; Adult Obstructive Sleep Apnea Task Force of the American Academy of Sleep Medicine. Clinical guideline for the evaluation, management and long-term care of obstructive sleep apnea in adults. J Clin Sleep Med 2009; 5:263–276.
- Young T, Palta M, Dempsey J, Skatrud J, Weber S, Badr S. The occurrence of sleep-disordered breathing among middle-aged adults. N Engl J Med 1993; 328:1230–1235.
- Punjabi NM. The epidemiology of adult obstructive sleep apnea. Proc Am Thorac Soc 2008; 5:136–143.
- Young T, Evans L, Finn L, Palta M. Estimation of the clinically diagnosed proportion of sleep apnea syndrome in middle-aged men and women. Sleep 1997; 20:705–706.
- Pedrosa RP, Drager LF, Gonzaga CC, et al. Obstructive sleep apnea: the most common secondary cause of hypertension associated with resistant hypertension. Hypertension 2011; 58:811–817.
- Redline S, Yenokyan G, Gottlieb DJ, et al. Obstructive sleep apnea-hypopnea and incident stroke: the Sleep Heart Health Study. Am J Respir Crit Care Med 2010; 182:269–277.
- Somers VK, White DP, Amin R, et al; American Heart Association Council for High Blood Pressure Research Professional Education Committee, Council on Clinical Cardiology; American Heart Association Stroke Council; American Heart Association Council on Cardiovascular Nursing; American College of Cardiology Foundation. Sleep apnea and cardiovascular disease: an American Heart Association/American College Of Cardiology Foundation Scientific Statement from the American Heart Association Council for High Blood Pressure Research Professional Education Committee, Council on Clinical Cardiology, Stroke Council, and Council On Cardiovascular Nursing. in collaboration with the National Heart, Lung, and Blood Institute National Center on Sleep Disorders Research (National Institutes of Health). Circulation 2008; 118:1080–1111.
- Mehra R, Benjamin EJ, Shahar E, et al; Sleep Heart Health Study. Association of nocturnal arrhythmias with sleep-disordered breathing: the Sleep Heart Health Study. Am J Respir Crit Care Med 2006; 173:910–916.
- Gami AS, Olson EJ, Shen WK, et al. Obstructive sleep apnea and the risk of sudden cardiac death: a longitudinal study of 10,701 adults. J Am Coll Cardiol 2013; 62:610–616.
- Martinez-Garcia MA, Soler-Cataluna JJ, Ejarque-Martinez L, et al. Continuous positive airway pressure treatment reduces mortality in patients with ischemic stroke and obstructive sleep apnea: a 5-year follow-up study. Am J Respir Crit Care Med 2009; 180:36–41.
- Punjabi NM, Shahar E, Redline S, Gottlieb DJ, Givelber R, Resnick HE; Sleep Heart Health Study Investigators. Sleep-disordered breathing, glucose intolerance, and insulin resistance: The Sleep Heart Health Study. Am J Epidemiol 2004; 160:521–530.
- Netzer NC, Hoegel JJ, Loube D, et al; Sleep in Primary Care International Study Group. Prevalence of symptoms and risk of sleep apnea in primary care. Chest 2003; 124:1406–1414.
- Chung F, Yegneswaran B, Liao P, et al. STOP questionnaire: a tool to screen patients for obstructive sleep apnea. Anesthesiology 2008; 108:812–821.
- Collop NA, Anderson WM, Boehlecke B, et al; Portable Monitoring Task Force of the American Academy of Sleep Medicine. Clinical guidelines for the use of unattended portable monitors in the diagnosis of obstructive sleep apnea in adult patients. Portable Monitoring Task Force of the American Academy of Sleep Medicine. J Clin Sleep Med 2007; 3:737–747.
- Pietzsch JB, Garner A, Cipriano LE, Linehan JH. An integrated health-economic analysis of diagnostic and therapeutic strategies in the treatment of moderate-to-severe obstructive sleep apnea. Sleep 2011; 34:695–709.
Serum allergen-specific IgE testing: How much is too much?
A 25-year-old man is evaluated for angioedema (swelling of lips and tongue) after eating paella at a Spanish restaurant. He has no history of allergies, but he says he had never eaten such a large variety of seafood before, especially shellfish.
He suspects that he is allergic to shellfish and asks the attending physician to order blood tests for seafood allergies, as he heard from a friend that blood tests are superior to other types of tests for allergy. The physician requests a serum immunoglobulin E (IgE) food panel test for this patient.
SERUM ALLERGEN-SPECIFIC IgE TESTING
Many methods of testing for allergy are available, including the skin-prick test, double-blind and single-blind placebo-controlled food challenges, open food challenges, inhalant challenges, drug challenges, and serum IgE tests. In clinical practice, these tests are often used in combination because when used individually, few of them are both highly sensitive and specific (Table 1).1–6
Skin-prick testing is generally the method of choice for the preliminary evaluation of IgE-mediated allergies because it is more sensitive and requires less time to get a result.1 But it is not the preferred test if the patient is at risk of a systemic reaction or has widespread dermatitis, nor is it useful if the patient is taking drugs that suppress the histamine response, such as antihistamines or tricyclic antidepressants.6 Moreover, skin-prick testing is more invasive and time-consuming than serum IgE testing.
Serum IgE testing is an attractive alternative, and it is more convenient because it requires only a single blood draw and poses a lower risk of adverse effects.
NOT A RELIABLE DIAGNOSTIC TOOL
As serum IgE testing has gained popularity, researchers have tried to improve its diagnostic power (ie, maximize its sensitivity and specificity) by determining the best cutoff values for IgE against specific antigens. Unfortunately, these values are difficult to determine because of confounding factors such as the lack of a reference standard, population diversity, patient atopy, and the overwhelming number of allergens that must be examined.
In addition, some researchers have used positive and negative predictive values to evaluate diagnostic cutoffs for serum antigen-specific IgE values. But these are not the most suitable performance measure to evaluate because they depend on disease prevalence and population characteristics.
Despite these efforts, results are still conflicting, and serum antigen-specific IgE testing is not a reliable diagnostic tool.
In an effort to gain insight from the available research data, we evaluated the clinical usefulness of 89 antigen-specific IgE tests, using an approach of summing their sensitivity and specificity. Previously, Wians7 proposed that a test is likely to be clinically useful if the sum of its sensitivity and specificity is equal to or greater than 170. Figure 1 shows the 89 tests, grouped into categories, and their summed sensitivities and specificities. The dashed line indicates a cutoff of 170; any bar that touches or crosses that line indicates that the test may be clinically useful, according to Wians.7
Only 7 of the 89 tests (cow, buckwheat, hazelnut, latex, Alternaria alternata, honey bee venom, and Johnson grass) satisfied this criterion. This suggests that a significant number of serum antigen-specific IgE tests perform suboptimally, and we are left with the question of why they are so commonly ordered.
Inappropriate use can lead to false-positive results, a situation in which patients may be subjected to unnecessary food avoidance that can result in nutritional deficiencies and decreased quality of life. It can also lead to false-negative results, when life-threatening diagnoses are missed and further excessive downstream testing is required—all leading; to negative outcomes for both patients and healthcare providers.
CHOOSING WISELY
The Choosing Wisely campaign in the United States has partnered with the American Academy of Allergy, Asthma, and Immunology to advocate against indiscriminate IgE testing in evaluating allergy.8 Allergy diagnosis and evaluation should be based on a combination of clinical history and judicious ordering of specific IgE tests, whether through skin or blood testing. Ordering of serum allergen-specific IgE tests for food allergies should be consistent with a clinical history of potential IgE-mediated food allergy8 and not food intolerance (Table 2).4,5
Some jurisdictions in Canada have followed suit by restricting the number of serum IgE tests each physician is allowed to order per patient, to encourage more responsible ordering and to lower the number of potential false-positive results, which can lead to increased downstream costs as well as unnecessary patient worry and lifestyle modification.
CLINICAL BOTTOM LINE
Ordering diagnostic tests that have little clinical utility has long-term detrimental effects on both patient safety and healthcare sustainability.
In the case of the 25-year-old evaluated for shellfish allergy, the clinician should first explain that the swelling of the lips and tongue (angioedema) does suggest an IgE-mediated allergic reaction and not a non–IgE-mediated allergic reaction or a food intolerance. Non–IgE-mediated food allergies and food intolerances are marked by symptoms relating mainly to nonimmune aspects of the digestive system, whereas IgE-mediated food allergies affect the immune system and can involve a multitude of organs, including the skin and the respiratory and digestive systems (Table 2).
However, clinicians should avoid indiscriminately ordering food allergen IgE panels and instead should focus on foods likely to be the culprits based on the clinical history.9 Indiscriminate testing can lead to false-positive results and unnecessary food avoidance.
Since the patient developed symptoms of angioedema when he was exposed to his allergen, he may be apprehensive about a skin- prick test and the possibility of being subjected to the same discomfort. Therefore, in this situation, it may be best to perform serum IgE tests, but on a few targeted seafoods rather than the food panel the physician had ordered. A patient can be sensitized to an allergen (possess IgE antibodies) but not experience symptoms when exposed to it (ie, have tolerance).5 Also, false-negative results may occur, so a negative serum allergen-specific IgE test should likewise be interpreted in light of the pretest probability of allergy to a specific antigen.
If the history and the results of testing are not clear and congruent, the patient should be referred to an allergist for diagnosis or for management. The allergist can provide management techniques and periodic assessment as to the progression and resolution of the allergy. Table 2 highlights symptoms that differentiate an IgE-mediated from a non–IgE-mediated food allergy.10,11Table 1 presents clinical indications and suggested diagnostic methods to the five most common allergen groups and the diagnostically invalid tests.1–6
The bottom line is that we must consider the poor performance of serum allergen-specific IgE tests when diagnosing and treating suspected type I allergies and avoid ordering food allergen IgE panels whenever possible.
- Bernstein IL, Li JT, Bernstein DI, et al; American Academy of Allergy, Asthma and Immunology; American College of Allergy, Asthma and Immunology. Allergy diagnostic testing: an updated practice parameter. Ann Allergy Asthma Immunol 2008; 100(suppl 3):S1–S148.
- Bird JA, Crain M, Varshney P. Food allergen panel testing often results in misdiagnosis of food allergy. J Pediatr 2015; 166:97–100.
- Kattan JD, Sicherer SH. Optimizing the diagnosis of food allergy. Immunol Allergy Clin North Am 2015; 35:61–76.
- Sampson HA, Aceves S, Bock SA, et al. Food allergy: a practice parameter update-2014. J Allergy Clin Immunol 2014; 134:1016–1025.e43.
- Sicherer SH, Sampson HA. Food allergy: epidemiology, pathogenesis, diagnosis, and treatment. J Allergy Clin Immunol 2014; 133:291–308.
- Siles RI, Hsieh FH. Allergy blood testing: a practical guide for clinicians. Cleve Clin J Med 2011; 78:585–592.
- Wians FH Jr. Clinical laboratory tests: which, why, and what do the results mean? Lab Medicine 2009; 40:105–113.
- Choosing Wisely. American Academy of Allergy, Asthma & Immunology. Ten Things Physicians and Patients Should Question. www.choosingwisely.org/doctor-patient-lists/american-academy-of-allergy-asthma-immunology/. Accessed December 3, 2015.
- Fleischer DM, Burks AW. Pitfalls in food allergy diagnosis: serum IgE testing. J Pediatr 2015; 166: 8-10.
- Boyce JA, Assa'ad A, Burks AW, et al; NIAID-Sponsored Expert Panel. Guidelines for the diagnosis and management of food allergy in the United States: summary of the NIAID-sponsored expert panel report. J Allergy Clin Immunol 2010; 126:1105–1118.
- Stiefel G, Roberts G. How to use serum-specific IgE measurements in diagnosing and monitoring food allergy. Arch Dis Child Educ Pract Ed 2012; 97:29–36.
A 25-year-old man is evaluated for angioedema (swelling of lips and tongue) after eating paella at a Spanish restaurant. He has no history of allergies, but he says he had never eaten such a large variety of seafood before, especially shellfish.
He suspects that he is allergic to shellfish and asks the attending physician to order blood tests for seafood allergies, as he heard from a friend that blood tests are superior to other types of tests for allergy. The physician requests a serum immunoglobulin E (IgE) food panel test for this patient.
SERUM ALLERGEN-SPECIFIC IgE TESTING
Many methods of testing for allergy are available, including the skin-prick test, double-blind and single-blind placebo-controlled food challenges, open food challenges, inhalant challenges, drug challenges, and serum IgE tests. In clinical practice, these tests are often used in combination because when used individually, few of them are both highly sensitive and specific (Table 1).1–6
Skin-prick testing is generally the method of choice for the preliminary evaluation of IgE-mediated allergies because it is more sensitive and requires less time to get a result.1 But it is not the preferred test if the patient is at risk of a systemic reaction or has widespread dermatitis, nor is it useful if the patient is taking drugs that suppress the histamine response, such as antihistamines or tricyclic antidepressants.6 Moreover, skin-prick testing is more invasive and time-consuming than serum IgE testing.
Serum IgE testing is an attractive alternative, and it is more convenient because it requires only a single blood draw and poses a lower risk of adverse effects.
NOT A RELIABLE DIAGNOSTIC TOOL
As serum IgE testing has gained popularity, researchers have tried to improve its diagnostic power (ie, maximize its sensitivity and specificity) by determining the best cutoff values for IgE against specific antigens. Unfortunately, these values are difficult to determine because of confounding factors such as the lack of a reference standard, population diversity, patient atopy, and the overwhelming number of allergens that must be examined.
In addition, some researchers have used positive and negative predictive values to evaluate diagnostic cutoffs for serum antigen-specific IgE values. But these are not the most suitable performance measure to evaluate because they depend on disease prevalence and population characteristics.
Despite these efforts, results are still conflicting, and serum antigen-specific IgE testing is not a reliable diagnostic tool.
In an effort to gain insight from the available research data, we evaluated the clinical usefulness of 89 antigen-specific IgE tests, using an approach of summing their sensitivity and specificity. Previously, Wians7 proposed that a test is likely to be clinically useful if the sum of its sensitivity and specificity is equal to or greater than 170. Figure 1 shows the 89 tests, grouped into categories, and their summed sensitivities and specificities. The dashed line indicates a cutoff of 170; any bar that touches or crosses that line indicates that the test may be clinically useful, according to Wians.7
Only 7 of the 89 tests (cow, buckwheat, hazelnut, latex, Alternaria alternata, honey bee venom, and Johnson grass) satisfied this criterion. This suggests that a significant number of serum antigen-specific IgE tests perform suboptimally, and we are left with the question of why they are so commonly ordered.
Inappropriate use can lead to false-positive results, a situation in which patients may be subjected to unnecessary food avoidance that can result in nutritional deficiencies and decreased quality of life. It can also lead to false-negative results, when life-threatening diagnoses are missed and further excessive downstream testing is required—all leading; to negative outcomes for both patients and healthcare providers.
CHOOSING WISELY
The Choosing Wisely campaign in the United States has partnered with the American Academy of Allergy, Asthma, and Immunology to advocate against indiscriminate IgE testing in evaluating allergy.8 Allergy diagnosis and evaluation should be based on a combination of clinical history and judicious ordering of specific IgE tests, whether through skin or blood testing. Ordering of serum allergen-specific IgE tests for food allergies should be consistent with a clinical history of potential IgE-mediated food allergy8 and not food intolerance (Table 2).4,5
Some jurisdictions in Canada have followed suit by restricting the number of serum IgE tests each physician is allowed to order per patient, to encourage more responsible ordering and to lower the number of potential false-positive results, which can lead to increased downstream costs as well as unnecessary patient worry and lifestyle modification.
CLINICAL BOTTOM LINE
Ordering diagnostic tests that have little clinical utility has long-term detrimental effects on both patient safety and healthcare sustainability.
In the case of the 25-year-old evaluated for shellfish allergy, the clinician should first explain that the swelling of the lips and tongue (angioedema) does suggest an IgE-mediated allergic reaction and not a non–IgE-mediated allergic reaction or a food intolerance. Non–IgE-mediated food allergies and food intolerances are marked by symptoms relating mainly to nonimmune aspects of the digestive system, whereas IgE-mediated food allergies affect the immune system and can involve a multitude of organs, including the skin and the respiratory and digestive systems (Table 2).
However, clinicians should avoid indiscriminately ordering food allergen IgE panels and instead should focus on foods likely to be the culprits based on the clinical history.9 Indiscriminate testing can lead to false-positive results and unnecessary food avoidance.
Since the patient developed symptoms of angioedema when he was exposed to his allergen, he may be apprehensive about a skin- prick test and the possibility of being subjected to the same discomfort. Therefore, in this situation, it may be best to perform serum IgE tests, but on a few targeted seafoods rather than the food panel the physician had ordered. A patient can be sensitized to an allergen (possess IgE antibodies) but not experience symptoms when exposed to it (ie, have tolerance).5 Also, false-negative results may occur, so a negative serum allergen-specific IgE test should likewise be interpreted in light of the pretest probability of allergy to a specific antigen.
If the history and the results of testing are not clear and congruent, the patient should be referred to an allergist for diagnosis or for management. The allergist can provide management techniques and periodic assessment as to the progression and resolution of the allergy. Table 2 highlights symptoms that differentiate an IgE-mediated from a non–IgE-mediated food allergy.10,11Table 1 presents clinical indications and suggested diagnostic methods to the five most common allergen groups and the diagnostically invalid tests.1–6
The bottom line is that we must consider the poor performance of serum allergen-specific IgE tests when diagnosing and treating suspected type I allergies and avoid ordering food allergen IgE panels whenever possible.
A 25-year-old man is evaluated for angioedema (swelling of lips and tongue) after eating paella at a Spanish restaurant. He has no history of allergies, but he says he had never eaten such a large variety of seafood before, especially shellfish.
He suspects that he is allergic to shellfish and asks the attending physician to order blood tests for seafood allergies, as he heard from a friend that blood tests are superior to other types of tests for allergy. The physician requests a serum immunoglobulin E (IgE) food panel test for this patient.
SERUM ALLERGEN-SPECIFIC IgE TESTING
Many methods of testing for allergy are available, including the skin-prick test, double-blind and single-blind placebo-controlled food challenges, open food challenges, inhalant challenges, drug challenges, and serum IgE tests. In clinical practice, these tests are often used in combination because when used individually, few of them are both highly sensitive and specific (Table 1).1–6
Skin-prick testing is generally the method of choice for the preliminary evaluation of IgE-mediated allergies because it is more sensitive and requires less time to get a result.1 But it is not the preferred test if the patient is at risk of a systemic reaction or has widespread dermatitis, nor is it useful if the patient is taking drugs that suppress the histamine response, such as antihistamines or tricyclic antidepressants.6 Moreover, skin-prick testing is more invasive and time-consuming than serum IgE testing.
Serum IgE testing is an attractive alternative, and it is more convenient because it requires only a single blood draw and poses a lower risk of adverse effects.
NOT A RELIABLE DIAGNOSTIC TOOL
As serum IgE testing has gained popularity, researchers have tried to improve its diagnostic power (ie, maximize its sensitivity and specificity) by determining the best cutoff values for IgE against specific antigens. Unfortunately, these values are difficult to determine because of confounding factors such as the lack of a reference standard, population diversity, patient atopy, and the overwhelming number of allergens that must be examined.
In addition, some researchers have used positive and negative predictive values to evaluate diagnostic cutoffs for serum antigen-specific IgE values. But these are not the most suitable performance measure to evaluate because they depend on disease prevalence and population characteristics.
Despite these efforts, results are still conflicting, and serum antigen-specific IgE testing is not a reliable diagnostic tool.
In an effort to gain insight from the available research data, we evaluated the clinical usefulness of 89 antigen-specific IgE tests, using an approach of summing their sensitivity and specificity. Previously, Wians7 proposed that a test is likely to be clinically useful if the sum of its sensitivity and specificity is equal to or greater than 170. Figure 1 shows the 89 tests, grouped into categories, and their summed sensitivities and specificities. The dashed line indicates a cutoff of 170; any bar that touches or crosses that line indicates that the test may be clinically useful, according to Wians.7
Only 7 of the 89 tests (cow, buckwheat, hazelnut, latex, Alternaria alternata, honey bee venom, and Johnson grass) satisfied this criterion. This suggests that a significant number of serum antigen-specific IgE tests perform suboptimally, and we are left with the question of why they are so commonly ordered.
Inappropriate use can lead to false-positive results, a situation in which patients may be subjected to unnecessary food avoidance that can result in nutritional deficiencies and decreased quality of life. It can also lead to false-negative results, when life-threatening diagnoses are missed and further excessive downstream testing is required—all leading; to negative outcomes for both patients and healthcare providers.
CHOOSING WISELY
The Choosing Wisely campaign in the United States has partnered with the American Academy of Allergy, Asthma, and Immunology to advocate against indiscriminate IgE testing in evaluating allergy.8 Allergy diagnosis and evaluation should be based on a combination of clinical history and judicious ordering of specific IgE tests, whether through skin or blood testing. Ordering of serum allergen-specific IgE tests for food allergies should be consistent with a clinical history of potential IgE-mediated food allergy8 and not food intolerance (Table 2).4,5
Some jurisdictions in Canada have followed suit by restricting the number of serum IgE tests each physician is allowed to order per patient, to encourage more responsible ordering and to lower the number of potential false-positive results, which can lead to increased downstream costs as well as unnecessary patient worry and lifestyle modification.
CLINICAL BOTTOM LINE
Ordering diagnostic tests that have little clinical utility has long-term detrimental effects on both patient safety and healthcare sustainability.
In the case of the 25-year-old evaluated for shellfish allergy, the clinician should first explain that the swelling of the lips and tongue (angioedema) does suggest an IgE-mediated allergic reaction and not a non–IgE-mediated allergic reaction or a food intolerance. Non–IgE-mediated food allergies and food intolerances are marked by symptoms relating mainly to nonimmune aspects of the digestive system, whereas IgE-mediated food allergies affect the immune system and can involve a multitude of organs, including the skin and the respiratory and digestive systems (Table 2).
However, clinicians should avoid indiscriminately ordering food allergen IgE panels and instead should focus on foods likely to be the culprits based on the clinical history.9 Indiscriminate testing can lead to false-positive results and unnecessary food avoidance.
Since the patient developed symptoms of angioedema when he was exposed to his allergen, he may be apprehensive about a skin- prick test and the possibility of being subjected to the same discomfort. Therefore, in this situation, it may be best to perform serum IgE tests, but on a few targeted seafoods rather than the food panel the physician had ordered. A patient can be sensitized to an allergen (possess IgE antibodies) but not experience symptoms when exposed to it (ie, have tolerance).5 Also, false-negative results may occur, so a negative serum allergen-specific IgE test should likewise be interpreted in light of the pretest probability of allergy to a specific antigen.
If the history and the results of testing are not clear and congruent, the patient should be referred to an allergist for diagnosis or for management. The allergist can provide management techniques and periodic assessment as to the progression and resolution of the allergy. Table 2 highlights symptoms that differentiate an IgE-mediated from a non–IgE-mediated food allergy.10,11Table 1 presents clinical indications and suggested diagnostic methods to the five most common allergen groups and the diagnostically invalid tests.1–6
The bottom line is that we must consider the poor performance of serum allergen-specific IgE tests when diagnosing and treating suspected type I allergies and avoid ordering food allergen IgE panels whenever possible.
- Bernstein IL, Li JT, Bernstein DI, et al; American Academy of Allergy, Asthma and Immunology; American College of Allergy, Asthma and Immunology. Allergy diagnostic testing: an updated practice parameter. Ann Allergy Asthma Immunol 2008; 100(suppl 3):S1–S148.
- Bird JA, Crain M, Varshney P. Food allergen panel testing often results in misdiagnosis of food allergy. J Pediatr 2015; 166:97–100.
- Kattan JD, Sicherer SH. Optimizing the diagnosis of food allergy. Immunol Allergy Clin North Am 2015; 35:61–76.
- Sampson HA, Aceves S, Bock SA, et al. Food allergy: a practice parameter update-2014. J Allergy Clin Immunol 2014; 134:1016–1025.e43.
- Sicherer SH, Sampson HA. Food allergy: epidemiology, pathogenesis, diagnosis, and treatment. J Allergy Clin Immunol 2014; 133:291–308.
- Siles RI, Hsieh FH. Allergy blood testing: a practical guide for clinicians. Cleve Clin J Med 2011; 78:585–592.
- Wians FH Jr. Clinical laboratory tests: which, why, and what do the results mean? Lab Medicine 2009; 40:105–113.
- Choosing Wisely. American Academy of Allergy, Asthma & Immunology. Ten Things Physicians and Patients Should Question. www.choosingwisely.org/doctor-patient-lists/american-academy-of-allergy-asthma-immunology/. Accessed December 3, 2015.
- Fleischer DM, Burks AW. Pitfalls in food allergy diagnosis: serum IgE testing. J Pediatr 2015; 166: 8-10.
- Boyce JA, Assa'ad A, Burks AW, et al; NIAID-Sponsored Expert Panel. Guidelines for the diagnosis and management of food allergy in the United States: summary of the NIAID-sponsored expert panel report. J Allergy Clin Immunol 2010; 126:1105–1118.
- Stiefel G, Roberts G. How to use serum-specific IgE measurements in diagnosing and monitoring food allergy. Arch Dis Child Educ Pract Ed 2012; 97:29–36.
- Bernstein IL, Li JT, Bernstein DI, et al; American Academy of Allergy, Asthma and Immunology; American College of Allergy, Asthma and Immunology. Allergy diagnostic testing: an updated practice parameter. Ann Allergy Asthma Immunol 2008; 100(suppl 3):S1–S148.
- Bird JA, Crain M, Varshney P. Food allergen panel testing often results in misdiagnosis of food allergy. J Pediatr 2015; 166:97–100.
- Kattan JD, Sicherer SH. Optimizing the diagnosis of food allergy. Immunol Allergy Clin North Am 2015; 35:61–76.
- Sampson HA, Aceves S, Bock SA, et al. Food allergy: a practice parameter update-2014. J Allergy Clin Immunol 2014; 134:1016–1025.e43.
- Sicherer SH, Sampson HA. Food allergy: epidemiology, pathogenesis, diagnosis, and treatment. J Allergy Clin Immunol 2014; 133:291–308.
- Siles RI, Hsieh FH. Allergy blood testing: a practical guide for clinicians. Cleve Clin J Med 2011; 78:585–592.
- Wians FH Jr. Clinical laboratory tests: which, why, and what do the results mean? Lab Medicine 2009; 40:105–113.
- Choosing Wisely. American Academy of Allergy, Asthma & Immunology. Ten Things Physicians and Patients Should Question. www.choosingwisely.org/doctor-patient-lists/american-academy-of-allergy-asthma-immunology/. Accessed December 3, 2015.
- Fleischer DM, Burks AW. Pitfalls in food allergy diagnosis: serum IgE testing. J Pediatr 2015; 166: 8-10.
- Boyce JA, Assa'ad A, Burks AW, et al; NIAID-Sponsored Expert Panel. Guidelines for the diagnosis and management of food allergy in the United States: summary of the NIAID-sponsored expert panel report. J Allergy Clin Immunol 2010; 126:1105–1118.
- Stiefel G, Roberts G. How to use serum-specific IgE measurements in diagnosing and monitoring food allergy. Arch Dis Child Educ Pract Ed 2012; 97:29–36.
Managing interstitial lung disease detected on CT during lung cancer screening
Primary care physicians are playing a bigger role in evaluating the incidental finding of interstitial lung diseases since the recent publication of guidelines recommending computed tomography (CT) to screen for lung cancer.
In August 2011, the National Cancer Institute published its findings from the National Lung Screening Trial, which demonstrated a 20% reduction in mortality from lung cancer in patients at high risk screened with low-dose CT.1 Based on these results, the American Cancer Society, the American College of Chest Physicians, the American Society of Clinical Oncology, and the National Comprehensive Cancer Network recommended annual screening for lung cancer with low-dose CT in adults ages 55 to 74 who have a 30-pack-year smoking history and who currently smoke or have quit within the past 15 years.2 In December 2013, the US Preventive Services Task Force published similar guidelines but increased the age range to include high-risk patients ages 55 to 80.3
Bach et al4 estimated that, in 2010 in the United States, 8.6 million people met the criteria used in the National Lung Screening Trial for low-dose CT screening. These are the same criteria as in the multisociety recommendations cited above.2 With such large numbers of patients eligible for CT screening, internists and other primary care physicians are undoubtedly encountering the incidental discovery of nonmalignant pulmonary diseases such as interstitial lung disease.
This article reviews the radiographic characteristics of the most common interstitial lung diseases the internist may encounter on screening CT in long-term smokers.
Referral to a specialist has been associated with lower rates of morbidity and death,5 and a diagnosis of interstitial lung disease should be confirmed by a pulmonologist and a radiologist specializing in differentiating the subtypes. But the primary care physician now plays a critical role in recognizing the need for further evaluation.
HOW COMMON IS INTERSTITIAL LUNG DISEASE IN SMOKERS?
Several studies have published data on the prevalence of interstitial lung disease in patients undergoing low-dose CT for lung cancer screening.
A trial at Mayo Clinic in current and former smokers identified “diffuse lung disease” in 9 (0.9%) of 1,049 participants.6
A trial in Ireland identified idiopathic pulmonary fibrosis in 6 (1.3%) of 449 current smokers who underwent low-dose CT screening for lung cancer.7
Sverzellati et al8 evaluated 692 participants in the Multicentric Italian Lung Detection CT screening study and reported a respiratory bronchiolitis pattern in 109 (15.7%), a usual interstitial pneumonia pattern in 2 (0.3%), and other patterns of chronic interstitial pneumonia in 26 (3.8%).
The National Lung Screening Trial reported that the frequency of “clinically significant” incidental findings (including pulmonary fibrosis) in all participants was 7.5%.1 A retrospective analysis of 884 participants at a single site in this trial identified interstitial lung abnormalities in 86 participants (9.7%).9 These abnormalities were further categorized as nonfibrotic in 52 (5.9%) of 884, fibrotic in 19 (2.1%) of 884, and mixed fibrotic and nonfibrotic in 15 (1.7%) of 884.
Follow-up CT at 2 years in this trial demonstrated improvement in 50% and progression in 11% of patients who had nonfibrotic abnormalities, while fibrotic abnormalities improved in no cases and progressed in 37%. Interstitial lung abnormalities were more common in those who currently smoked and in those with more pack-years of cigarette smoking.9
In sum, these trials suggest that low-dose CT screening for lung cancer can detect the most common forms of interstitial lung disease in this at-risk population and can characterize them as fibrotic or nonfibrotic, a distinction important for prognosis and subsequent management.
NONFIBROTIC VS FIBROTIC DISEASE
It is important to distinguish between nonfibrotic and fibrotic interstitial lung disease, as fibrotic disease carries a worse prognosis and is treated differently.
Features of nonfibrotic interstitial lung disease:
- Ground-glass opacities
- Nodules
- Mosaic attenuation or consolidation.
Features of fibrotic interstitial lung disease:
- Combination of ground-glass opacities and reticulation
- Reticulation by itself
- Traction bronchiectasis
- Honeycombing
- Loss of lung volume.
NONFIBROTIC INTERSTITIAL LUNG DISEASES
Given the strong likelihood that a patient undergoing screening CT is either a current or former smoker, physicians may encounter, in addition to emphysema and lung cancer, the following smoking-related interstitial lung diseases, which are primarily nonfibrotic and which frequently coexist (Table 1):
- Respiratory bronchiolitis
- Respiratory bronchiolitis-interstitial lung disease
- Desquamative interstitial pneumonia
- Pulmonary Langerhans cell histiocytosis.
Respiratory bronchiolitis
Respiratory bronchiolitis occurs mostly in smokers and does not necessarily lead to respiratory symptoms in all patients.10 It cannot always be identified radiographically but occasionally appears as predominantly upper-lobe, patchy ground-glass opacities or ill-defined centrilobular nodules without evidence of fibrosis (Figure 1).
Respiratory bronchiolitis-interstitial lung disease
In rare cases, respiratory bronchiolitis leads to peribronchial fibrosis invading the alveolar walls, which is then classified as respiratory bronchiolitis-interstitial lung disease.11 The CT findings in respiratory bronchiolitis-interstitial lung disease are upper-lobe-predominant centrilobular ground-glass nodules, patchy ground-glass opacities, and bronchial wall thickening (Figure 2).10 Occasionally, mild reticulation is noted without honeycombing. Mild air trapping can be seen in the lower lobes, with centrilobular emphysema in the upper lobes.12
The only successful therapy for respiratory bronchiolitis and respiratory bronchiolitis-interstitial lung disease is smoking cessation. Finding either of these diseases should prompt aggressive counseling by the internist and consideration of referral to a specialist in interstitial lung disease.
Desquamative interstitial pneumonia
Although pathologically different from respiratory bronchiolitis-interstitial lung disease, desquamative interstitial pneumonia has a similar clinical and radiographic presentation. Because their features significantly overlap, they are considered a pathomorphologic continuum, representing degrees of severity of the same disease process caused by prolonged tobacco inhalation.10,13
Widespread ground-glass opacities are the predominant CT finding. These are bilateral and symmetric in distribution in 86%, basal and peripheral in 60%, patchy in 20%, and diffuse in 20% (Figure 3).14 Other frequent findings are mild reticulation with traction bronchiectasis and coexistent emphysema (Figure 4).15 The small peripheral cystic spaces noted in this disease most likely represent dilated bronchioles and alveolar ducts rather than honeycombing.16
No additional treatment beyond elimination of smoking has been proven effective for desquamative interstitial pneumonia, and patients who manage to quit smoking generally have a favorable prognosis.17,18
Pulmonary Langerhans cell histiocytosis
The combination of upper-lobe-predominant cysts and nodules in a young heavy smoker are diagnostic of pulmonary Langerhans cell histiocytosis. The cysts are bizarrely shaped, thin- or thick-walled, and nonuniform in size (Figure 5). The irregular cavitary nodules are centrilobular. The disease characteristically spares the costophrenic angles.
Spontaneous pneumothorax is the initial clinical presentation in 15% of patients.16 In the early stages of the disease (nodule-predominant disease without cysts), infection and metastatic disease need to be excluded (Figure 6). In the later stages, the cysts become coalescent, making the distinction between this disease and “burned-out” lymphangioleiomyomatosis or severe emphysema extremely difficult (Figure 7).17 Smoking cessation and corticosteroids are the mainstay of medical therapy for pulmonary Langerhans cell histiocytosis, and about 50% of patients who quit smoking and receive corticosteroids demonstrate partial or complete clearing of the radiographic abnormalities and symptoms (Figure 8).
FIBROTIC INTERSTITIAL LUNG DISEASES
If CT identifies a diffuse fibrotic pattern, the two most common possibilities (Table 2) are:
- Nonspecific interstitial pneumonia
- Usual interstitial pneumonia.
As noted above, these carry a worse prognosis than the nonfibrotic interstitial lung diseases.
Nonspecific interstitial pneumonia
While most frequently idiopathic, the nonspecific interstitial pneumonia pattern can often be seen in connective tissue diseases. It has also been associated with chronic hypersensitivity pneumonitis, drug toxicity, and slowly resolving diffuse alveolar damage.19 Although it is not the only pathologic pattern in interstitial lung disease associated with connective tissue disease, it is the most common pattern in systemic sclerosis, systemic lupus erythematosus, dermatomyositis-polymyositis, and mixed connective tissue disease.20
The parenchymal changes are typically subpleural and symmetric in distribution (Figure 9). In about one-third of cases, there is a peribronchovascular distribution of the abnormalities (Figure 10).
Ground-glass opacities are the dominant imaging findings, seen in 80% of cases.18 In advanced disease (also referred to as fibrotic nonspecific interstitial pneumonia), patients have accompanying fine or coarse reticular opacities, traction bronchiectasis, and consolidation (Figure 11). Honeycombing is seen in 1% to 5% of patients.21
The most specific sign of nonspecific interstitial pneumonia is sparing of the immediate subpleural lung, apparent in 30% to 50% of patients (Figure 12).22 Subpleural sparing with a peribronchovascular distribution of abnormalities, absence of lobular areas with decreased attenuation, and lack of honeycombing are imaging features that increase the diagnostic confidence of nonspecific interstitial pneumonia (Table 3).23 Clinically, compared with those who have usual interstitial pneumonia (see below), patients are younger and more of them are female. These patients also present with extrapulmonary manifestations such as joint involvement, rash, and Raynaud phenomenon. Therefore, these associated symptoms on presentation can help distinguish nonspecific interstitial pneumonia or usual interstitial pneumonia related to connective tissue disease from the idiopathic forms.
The first step in managing nonspecific interstitial pneumonia is to remove all potential exposure to inhaled substances or to drugs. Although immunosuppressive therapy has never been studied in a randomized controlled trial in this disease, numerous reports suggest that patients may respond to prednisone and to steroid-sparing immunosuppressants.24
In several studies, survival rates in nonspecific interstitial pneumonia were significantly greater than in usual interstitial pneumonia independent of the treatment strategy. In long-term follow-up of patients with idiopathic nonspecific interstitial pneumonia treated with immunosuppressive therapy, two-thirds remained stable or improved.25–27
Although most connective tissue diseases cause a lung pattern of nonspecific interstitial pneumonia, some (eg, rheumatoid arthritis) may present with a pattern of usual interstitial pneumonia. In these cases and in those of advanced fibrotic nonspecific interstitial pneumonia, the prognosis is worse, as the disease is less responsive to immunosuppressive therapy.20
Usual interstitial pneumonia
Usual interstitial pneumonia is the most severe form of lung fibrosis. Most cases are idiopathic and are termed idiopathic pulmonary fibrosis. Other causes of the usual interstitial pneumonia pattern include domestic and occupational environmental exposures, connective tissue disease, and drug toxicity.28 An epidemiologic association between smoking and usual interstitial pneumonia is well documented.28
Idiopathic pulmonary fibrosis typically affects men ages 50 to 70. Because its risk factors coincide with those of lung cancer, there is a high likelihood of detecting idiopathic pulmonary fibrosis early in this screening population. It has an especially poor prognosis, with a mean survival of 2 to 5 years from the time of diagnosis.18
The distribution of disease in usual interstitial pneumonia is characteristically subpleural and basal. CT features include coarse subpleural reticulation and honeycombing combined with traction bronchiectasis or bronchiolectasis and architectural distortion (Figure 13).18 Honeycombing is the most specific and key diagnostic CT finding for establishing a definitive diagnosis of usual interstitial pneumonia.29 However, ground-glass opacities are present in most patients, typically in the region of interstitial fibrosis, and are always less extensive than the reticulation.30 The findings demonstrate morphologic heterogeneity, with areas of fibrosis adjacent to areas of normal lung (Figure 14).
In addition to the aforementioned imaging features, the 2011 American Thoracic Society and European Respiratory Society joint guidelines for the CT diagnosis of usual interstitial pneumonia patterns require the absence of atypical features that suggest an alternative diagnosis, including those seen in nonspecific interstitial pneumonia, such as an upper, midlung, or peribronchovascular distribution and extensive ground-glass attenuation.28 Mild mediastinal lymphadenopathy (usually < 1.5 cm in the short axis) is common in usual interstitial pneumonia.31
Because other chronic interstitial pneumonias that may resemble usual interstitial pneumonia have a more favorable course and may respond to immunosuppressive therapy, establishing an early and accurate diagnosis is of the utmost importance.5 Additionally, the emergence of possible new therapies for idiopathic pulmonary fibrosis makes early referral to a specialist paramount in these cases. Recent studies have demonstrated significant slowing of the progression of disease in idiopathic pulmonary fibrosis with both pirfenidone and nintedanib.32,33
DIAGNOSIS AND MANAGEMENT
The diagnosis of these nonfibrotic and fibrotic lung diseases is complex. In all cases in which interstitial lung disease is detected on screening CT for lung cancer, the internist should strongly consider further evaluation with dedicated high-resolution CT and early referral to a specialist (Figure 15).
Because smoking cessation is the only recommended treatment for nonfibrotic smoking-related interstitial lung diseases, particular emphasis on smoking cessation counseling is essential.
Referral for bronchoscopy with transbronchial lung biopsy is generally not helpful in the diagnosis of the interstitial lung diseases discussed in this article unless there is a need to rule out infection or neoplasm.34 Referral for surgical lung biopsy may be indicated in some cases of suspected pulmonary Langerhans cell histiocytosis, desquamative interstitial pneumonia, nonspecific interstitial pneumonia, or usual interstitial pneumonia if the diagnosis is uncertain (Tables 1 and 2).35
The American Thoracic Society/European Respiratory Society guidelines suggest a multidisciplinary team approach that includes a pathologist, radiologist, and clinician.35 This approach more readily determines the correct diagnosis and relies less on invasive methods such as surgical biopsy and more on noninvasive methods such as radiology and clinical history. Overall, this will promote earlier access to appropriate therapies, clinical trial enrollment, and in more severe cases, lung transplant.
Currently, 23% of all lung transplants worldwide are performed in patients with idiopathic pulmonary fibrosis. Other forms of pulmonary fibrosis account for 3% to 4% of lung transplants performed.36
Evidence suggests that early referral reduces rates of morbidity and death in these patients. The results of a single-center study37 of patients with idiopathic pulmonary fibrosis demonstrated that a longer delay from the onset of symptoms to evaluation by a specialist at a tertiary care referral center was associated with a higher rate of death from this disease independent of disease severity. Those with the longest delay in referral had a multivariable-adjusted death rate 3.4 times higher than those with the shortest delay.5,37
In summary, with implementation of the new lung cancer screening guidelines, primary care physicians are more often encountering the incidental finding of interstitial lung disease in their patients. Prompt diagnosis of interstitial lung disease helps ensure that patients receive appropriate care and early consideration for clinical trials and lung transplant.
Primary care physicians play a critical role in the initial identification of key characteristics of the interstitial abnormality—namely, whether the pattern is nonfibrotic or fibrotic—and in the correlation of the history and physical findings to expedite the diagnosis. Subsequently, ordering high-resolution CT for more detailed characterization and prompt referral to a specialist in interstitial lung disease allow for a more rapid and accurate diagnosis, specialized therapy, and supportive care.
- National Lung Screening Trial Research Team; Aberle DR, Adams AM, Berg CD, et al. Reduced lung-cancer mortality with low-dose computed tomographic screening. N Engl J Med 2011; 365:395–409.
- Detterbeck FC, Lewis SZ, Diekemper R, Addrizzo-Harris D, Alberts WM. Executive summary: diagnosis and management of lung cancer, 3rd ed: American College of Chest Physicians evidence-based clinical practice guidelines. Chest 2013; 143(suppl 5):7S–37S.
- Moyer VA; US Preventive Services Task Force. Screening for lung cancer: US Preventive Services Task Force recommendation statement. Ann Intern Med 2014; 160:330–338.
- Bach PB, Mirkin JN, Oliver TK, et al. Benefits and harms of CT screening for lung cancer: a systematic review. JAMA 2012; 307:2418–2429.
- Lamas DJ, Kawut SM, Bagiella E, Philip N, Arcasoy SM, Lederer DJ. Delayed access and survival in idiopathic pulmonary fibrosis: a cohort study. Am J Respir Crit Care Med 2011; 184:842–847.
- Swensen SJ, Jett JR, Hartman TE, et al. Lung cancer screening with CT: Mayo Clinic experience. Radiology 2003; 226:756–761.
- MacRedmond R, Logan PM, Lee M, Kenny D, Foley C, Costello RW. Screening for lung cancer using low dose CT scanning. Thorax 2004; 59:237–241.
- Sverzellati N, Guerci L, Randi G, et al. Interstitial lung diseases in a lung cancer screening trial. Eur Respir J 2011; 38:392–400.
- Jin GY, Lynch D, Chawla A, et al. Interstitial lung abnormalities in a CT lung cancer screening population: prevalence and progression rate. Radiology 2013; 268:563–571.
- Heyneman LE, Ward S, Lynch DA, Remy-Jardin M, Johkoh T, Müller NL. Respiratory bronchiolitis, respiratory bronchiolitis-associated interstitial lung disease, and desquamative interstitial pneumonia: different entities or part of the spectrum of the same disease process? AJR Am J Roentgenol 1999; 173:1617–1622.
- Moon J, du Bois RM, Colby TV, Hansell DM, Nicholson AG. Clinical significance of respiratory bronchiolitis on open lung biopsy and its relationship to smoking related interstitial lung disease. Thorax 1999; 54:1009–1014.
- Holt RM, Schmidt RA, Godwin JD, Raghu G. High resolution CT in respiratory bronchiolitis-associated interstitial lung disease. J Comput Assist Tomogr 1993; 17:46–50.
- Ryu JH, Myers JL, Capizzi SA, Douglas WW, Vassallo R, Decker PA. Desquamative interstitial pneumonia and respiratory bronchiolitis-associated interstitial lung disease. Chest 2005; 127:178–184.
- Hartman TE, Primack SL, Swensen SJ, Hansell D, McGuinness G, Müller NL. Desquamative interstitial pneumonia: thin-section CT findings in 22 patients. Radiology 1993; 187:787–790.
- Akira M, Yamamoto S, Hara H, Sakatani M, Ueda E. Serial computed tomographic evaluation in desquamative interstitial pneumonia. Thorax 1997; 52:333–337.
- Lacronique J, Roth C, Battesti JP, Basset F, Chretien J. Chest radiological features of pulmonary histiocytosis X: a report based on 50 adult cases. Thorax 1982; 37:104–109.
- Remy-Jardin M, Edme JL, Boulenguez C, Remy J, Mastora I, Sobaszek A. Longitudinal follow-up study of smoker’s lung with thin-section CT in correlation with pulmonary function tests. Radiology 2002; 222:261–270.
- Mueller-Mang C, Grosse C, Schmid K, Stiebellehner L, Bankier AA. What every radiologist should know about idiopathic interstitial pneumonias. Radiographics 2007; 27:595–615.
- Katzenstein AL, Fiorelli RF. Nonspecific interstitial pneumonia/fibrosis. Histologic features and clinical significance. Am J Surg Pathol 1994; 18:136–147.
- Bryson T, Sundaram B, Khanna D, Kazerooni EA. Connective tissue disease-associated interstitial pneumonia and idiopathic interstitial pneumonia: similarity and difference. Semin Ultrasound CT MR 2014; 35:29–38.
- Desai SR, Veeraraghavan S, Hansell DM, et al. CT features of lung disease in patients with systemic sclerosis: comparison with idiopathic pulmonary fibrosis and nonspecific interstitial pneumonia. Radiology 2004; 232:560–567.
- Tsubamoto M, Müller NL, Johkoh T, et al. Pathologic subgroups of nonspecific interstitial pneumonia: differential diagnosis from other idiopathic interstitial pneumonias on high-resolution computed tomography. J Comput Assist Tomogr 2005; 29:793–800.
- Silva CI, Müller NL, Lynch DA, et al. Chronic hypersensitivity pneumonitis: differentiation from idiopathic pulmonary fibrosis and nonspecific interstitial pneumonia by using thin-section CT. Radiology 2008; 246:288–297.
- Antin-Ozerkis D, Rubinowitz A. An update on nonspecific interstitial pneumonia. Clin Pulm Med 2010; 17:122–128.
- Daniil ZD, Gilchrist FC, Nicholson AG, et al. A histologic pattern of nonspecific interstitial pneumonia is associated with a better prognosis than usual interstitial pneumonia in patients with cryptogenic fibrosing alveolitis. Am J Respir Crit Care Med 1999; 160:899–905.
- Travis WD, Matsui K, Moss J, Ferrans VJ. Idiopathic nonspecific interstitial pneumonia: prognostic significance of cellular and fibrosing patterns: survival comparison with usual interstitial pneumonia and desquamative interstitial pneumonia. Am J Surg Pathol 2000; 24:19–33.
- Riha RL, Duhig EE, Clarke BE, Steele RH, Slaughter RE, Zimmerman PV. Survival of patients with biopsy-proven usual interstitial pneumonia and nonspecific interstitial pneumonia. Eur Respir J 2002; 19:1114–1118.
- Raghu G, Collard HR, Egan JJ, et al; ATS/ERS/JRS/ALAT Committee on Idiopathic Pulmonary Fibrosis. An official ATS/ERS/JRS/ALAT statement: idiopathic pulmonary fibrosis: evidence-based guidelines for diagnosis and management. Am J Respir Crit Care Med 2011; 183:788–824.
- du Bois RM. An earlier and more confident diagnosis of idiopathic pulmonary fibrosis. Eur Respir Rev 2012; 21:141–146.
- Nishimura K, Kitaichi M, Izumi T, Nagai S, Kanaoka M, Itoh H. Usual interstitial pneumonia: histologic correlation with high-resolution CT. Radiology 1992; 182:337–342.
- Souza CA, Müller NL, Lee KS, Johkoh T, Mitsuhiro H, Chong S. Idiopathic interstitial pneumonias: prevalence of mediastinal lymph node enlargement in 206 patients. AJR Am J Roentgenol 2006; 186:995–999.
- King TE Jr, Bradford WZ, Castro-Bernardini S, et al; ASCEND Study Group. A phase 3 trial of pirfenidone in patients with idiopathic pulmonary fibrosis. N Engl J Med 2014; 370:2083–2092.
- Richeldi L, du Bois RM, Raghu G, et al; INPULSIS Trial Investigators. Efficacy and safety of nintedanib in idiopathic pulmonary fibrosis. N Engl J Med 2014; 370:2071–2082.
- Bradley B, Branley HM, Egan JJ, et al; British Thoracic Society Interstitial Lung Disease Guideline Group, British Thoracic Society Standards of Care Committee; Thoracic Society of Australia; New Zealand Thoracic Society; Irish Thoracic Society. Interstitial lung disease guideline: the British Thoracic Society in collaboration with the Thoracic Society of Australia and New Zealand and the Irish Thoracic Society. Thorax 2008; 63(suppl 5):v1–v58.
- Travis WD, Costabel U, Hansell DM, et al; ATS/ERS Committee on Idiopathic Interstitial Pneumonias. An official American Thoracic Society/European Respiratory Society statement: update of the international multidisciplinary classification of the idiopathic interstitial pneumonias. Am J Respir Crit Care Med 2013; 188:733–748.
- Stehlik J, Edwards LB, Kucheryavaya AY, et al; International Society of Heart and Lung Transplantation. The Registry of the International Society for Heart and Lung Transplantation: 29th official adult heart transplant report—2012. J Heart Lung Transplant 2012; 31:1052–1064.
- Oldham JM, Noth I. Idiopathic pulmonary fibrosis: early detection and referral. Respir Med 2014; 108:819–829.
Primary care physicians are playing a bigger role in evaluating the incidental finding of interstitial lung diseases since the recent publication of guidelines recommending computed tomography (CT) to screen for lung cancer.
In August 2011, the National Cancer Institute published its findings from the National Lung Screening Trial, which demonstrated a 20% reduction in mortality from lung cancer in patients at high risk screened with low-dose CT.1 Based on these results, the American Cancer Society, the American College of Chest Physicians, the American Society of Clinical Oncology, and the National Comprehensive Cancer Network recommended annual screening for lung cancer with low-dose CT in adults ages 55 to 74 who have a 30-pack-year smoking history and who currently smoke or have quit within the past 15 years.2 In December 2013, the US Preventive Services Task Force published similar guidelines but increased the age range to include high-risk patients ages 55 to 80.3
Bach et al4 estimated that, in 2010 in the United States, 8.6 million people met the criteria used in the National Lung Screening Trial for low-dose CT screening. These are the same criteria as in the multisociety recommendations cited above.2 With such large numbers of patients eligible for CT screening, internists and other primary care physicians are undoubtedly encountering the incidental discovery of nonmalignant pulmonary diseases such as interstitial lung disease.
This article reviews the radiographic characteristics of the most common interstitial lung diseases the internist may encounter on screening CT in long-term smokers.
Referral to a specialist has been associated with lower rates of morbidity and death,5 and a diagnosis of interstitial lung disease should be confirmed by a pulmonologist and a radiologist specializing in differentiating the subtypes. But the primary care physician now plays a critical role in recognizing the need for further evaluation.
HOW COMMON IS INTERSTITIAL LUNG DISEASE IN SMOKERS?
Several studies have published data on the prevalence of interstitial lung disease in patients undergoing low-dose CT for lung cancer screening.
A trial at Mayo Clinic in current and former smokers identified “diffuse lung disease” in 9 (0.9%) of 1,049 participants.6
A trial in Ireland identified idiopathic pulmonary fibrosis in 6 (1.3%) of 449 current smokers who underwent low-dose CT screening for lung cancer.7
Sverzellati et al8 evaluated 692 participants in the Multicentric Italian Lung Detection CT screening study and reported a respiratory bronchiolitis pattern in 109 (15.7%), a usual interstitial pneumonia pattern in 2 (0.3%), and other patterns of chronic interstitial pneumonia in 26 (3.8%).
The National Lung Screening Trial reported that the frequency of “clinically significant” incidental findings (including pulmonary fibrosis) in all participants was 7.5%.1 A retrospective analysis of 884 participants at a single site in this trial identified interstitial lung abnormalities in 86 participants (9.7%).9 These abnormalities were further categorized as nonfibrotic in 52 (5.9%) of 884, fibrotic in 19 (2.1%) of 884, and mixed fibrotic and nonfibrotic in 15 (1.7%) of 884.
Follow-up CT at 2 years in this trial demonstrated improvement in 50% and progression in 11% of patients who had nonfibrotic abnormalities, while fibrotic abnormalities improved in no cases and progressed in 37%. Interstitial lung abnormalities were more common in those who currently smoked and in those with more pack-years of cigarette smoking.9
In sum, these trials suggest that low-dose CT screening for lung cancer can detect the most common forms of interstitial lung disease in this at-risk population and can characterize them as fibrotic or nonfibrotic, a distinction important for prognosis and subsequent management.
NONFIBROTIC VS FIBROTIC DISEASE
It is important to distinguish between nonfibrotic and fibrotic interstitial lung disease, as fibrotic disease carries a worse prognosis and is treated differently.
Features of nonfibrotic interstitial lung disease:
- Ground-glass opacities
- Nodules
- Mosaic attenuation or consolidation.
Features of fibrotic interstitial lung disease:
- Combination of ground-glass opacities and reticulation
- Reticulation by itself
- Traction bronchiectasis
- Honeycombing
- Loss of lung volume.
NONFIBROTIC INTERSTITIAL LUNG DISEASES
Given the strong likelihood that a patient undergoing screening CT is either a current or former smoker, physicians may encounter, in addition to emphysema and lung cancer, the following smoking-related interstitial lung diseases, which are primarily nonfibrotic and which frequently coexist (Table 1):
- Respiratory bronchiolitis
- Respiratory bronchiolitis-interstitial lung disease
- Desquamative interstitial pneumonia
- Pulmonary Langerhans cell histiocytosis.
Respiratory bronchiolitis
Respiratory bronchiolitis occurs mostly in smokers and does not necessarily lead to respiratory symptoms in all patients.10 It cannot always be identified radiographically but occasionally appears as predominantly upper-lobe, patchy ground-glass opacities or ill-defined centrilobular nodules without evidence of fibrosis (Figure 1).
Respiratory bronchiolitis-interstitial lung disease
In rare cases, respiratory bronchiolitis leads to peribronchial fibrosis invading the alveolar walls, which is then classified as respiratory bronchiolitis-interstitial lung disease.11 The CT findings in respiratory bronchiolitis-interstitial lung disease are upper-lobe-predominant centrilobular ground-glass nodules, patchy ground-glass opacities, and bronchial wall thickening (Figure 2).10 Occasionally, mild reticulation is noted without honeycombing. Mild air trapping can be seen in the lower lobes, with centrilobular emphysema in the upper lobes.12
The only successful therapy for respiratory bronchiolitis and respiratory bronchiolitis-interstitial lung disease is smoking cessation. Finding either of these diseases should prompt aggressive counseling by the internist and consideration of referral to a specialist in interstitial lung disease.
Desquamative interstitial pneumonia
Although pathologically different from respiratory bronchiolitis-interstitial lung disease, desquamative interstitial pneumonia has a similar clinical and radiographic presentation. Because their features significantly overlap, they are considered a pathomorphologic continuum, representing degrees of severity of the same disease process caused by prolonged tobacco inhalation.10,13
Widespread ground-glass opacities are the predominant CT finding. These are bilateral and symmetric in distribution in 86%, basal and peripheral in 60%, patchy in 20%, and diffuse in 20% (Figure 3).14 Other frequent findings are mild reticulation with traction bronchiectasis and coexistent emphysema (Figure 4).15 The small peripheral cystic spaces noted in this disease most likely represent dilated bronchioles and alveolar ducts rather than honeycombing.16
No additional treatment beyond elimination of smoking has been proven effective for desquamative interstitial pneumonia, and patients who manage to quit smoking generally have a favorable prognosis.17,18
Pulmonary Langerhans cell histiocytosis
The combination of upper-lobe-predominant cysts and nodules in a young heavy smoker are diagnostic of pulmonary Langerhans cell histiocytosis. The cysts are bizarrely shaped, thin- or thick-walled, and nonuniform in size (Figure 5). The irregular cavitary nodules are centrilobular. The disease characteristically spares the costophrenic angles.
Spontaneous pneumothorax is the initial clinical presentation in 15% of patients.16 In the early stages of the disease (nodule-predominant disease without cysts), infection and metastatic disease need to be excluded (Figure 6). In the later stages, the cysts become coalescent, making the distinction between this disease and “burned-out” lymphangioleiomyomatosis or severe emphysema extremely difficult (Figure 7).17 Smoking cessation and corticosteroids are the mainstay of medical therapy for pulmonary Langerhans cell histiocytosis, and about 50% of patients who quit smoking and receive corticosteroids demonstrate partial or complete clearing of the radiographic abnormalities and symptoms (Figure 8).
FIBROTIC INTERSTITIAL LUNG DISEASES
If CT identifies a diffuse fibrotic pattern, the two most common possibilities (Table 2) are:
- Nonspecific interstitial pneumonia
- Usual interstitial pneumonia.
As noted above, these carry a worse prognosis than the nonfibrotic interstitial lung diseases.
Nonspecific interstitial pneumonia
While most frequently idiopathic, the nonspecific interstitial pneumonia pattern can often be seen in connective tissue diseases. It has also been associated with chronic hypersensitivity pneumonitis, drug toxicity, and slowly resolving diffuse alveolar damage.19 Although it is not the only pathologic pattern in interstitial lung disease associated with connective tissue disease, it is the most common pattern in systemic sclerosis, systemic lupus erythematosus, dermatomyositis-polymyositis, and mixed connective tissue disease.20
The parenchymal changes are typically subpleural and symmetric in distribution (Figure 9). In about one-third of cases, there is a peribronchovascular distribution of the abnormalities (Figure 10).
Ground-glass opacities are the dominant imaging findings, seen in 80% of cases.18 In advanced disease (also referred to as fibrotic nonspecific interstitial pneumonia), patients have accompanying fine or coarse reticular opacities, traction bronchiectasis, and consolidation (Figure 11). Honeycombing is seen in 1% to 5% of patients.21
The most specific sign of nonspecific interstitial pneumonia is sparing of the immediate subpleural lung, apparent in 30% to 50% of patients (Figure 12).22 Subpleural sparing with a peribronchovascular distribution of abnormalities, absence of lobular areas with decreased attenuation, and lack of honeycombing are imaging features that increase the diagnostic confidence of nonspecific interstitial pneumonia (Table 3).23 Clinically, compared with those who have usual interstitial pneumonia (see below), patients are younger and more of them are female. These patients also present with extrapulmonary manifestations such as joint involvement, rash, and Raynaud phenomenon. Therefore, these associated symptoms on presentation can help distinguish nonspecific interstitial pneumonia or usual interstitial pneumonia related to connective tissue disease from the idiopathic forms.
The first step in managing nonspecific interstitial pneumonia is to remove all potential exposure to inhaled substances or to drugs. Although immunosuppressive therapy has never been studied in a randomized controlled trial in this disease, numerous reports suggest that patients may respond to prednisone and to steroid-sparing immunosuppressants.24
In several studies, survival rates in nonspecific interstitial pneumonia were significantly greater than in usual interstitial pneumonia independent of the treatment strategy. In long-term follow-up of patients with idiopathic nonspecific interstitial pneumonia treated with immunosuppressive therapy, two-thirds remained stable or improved.25–27
Although most connective tissue diseases cause a lung pattern of nonspecific interstitial pneumonia, some (eg, rheumatoid arthritis) may present with a pattern of usual interstitial pneumonia. In these cases and in those of advanced fibrotic nonspecific interstitial pneumonia, the prognosis is worse, as the disease is less responsive to immunosuppressive therapy.20
Usual interstitial pneumonia
Usual interstitial pneumonia is the most severe form of lung fibrosis. Most cases are idiopathic and are termed idiopathic pulmonary fibrosis. Other causes of the usual interstitial pneumonia pattern include domestic and occupational environmental exposures, connective tissue disease, and drug toxicity.28 An epidemiologic association between smoking and usual interstitial pneumonia is well documented.28
Idiopathic pulmonary fibrosis typically affects men ages 50 to 70. Because its risk factors coincide with those of lung cancer, there is a high likelihood of detecting idiopathic pulmonary fibrosis early in this screening population. It has an especially poor prognosis, with a mean survival of 2 to 5 years from the time of diagnosis.18
The distribution of disease in usual interstitial pneumonia is characteristically subpleural and basal. CT features include coarse subpleural reticulation and honeycombing combined with traction bronchiectasis or bronchiolectasis and architectural distortion (Figure 13).18 Honeycombing is the most specific and key diagnostic CT finding for establishing a definitive diagnosis of usual interstitial pneumonia.29 However, ground-glass opacities are present in most patients, typically in the region of interstitial fibrosis, and are always less extensive than the reticulation.30 The findings demonstrate morphologic heterogeneity, with areas of fibrosis adjacent to areas of normal lung (Figure 14).
In addition to the aforementioned imaging features, the 2011 American Thoracic Society and European Respiratory Society joint guidelines for the CT diagnosis of usual interstitial pneumonia patterns require the absence of atypical features that suggest an alternative diagnosis, including those seen in nonspecific interstitial pneumonia, such as an upper, midlung, or peribronchovascular distribution and extensive ground-glass attenuation.28 Mild mediastinal lymphadenopathy (usually < 1.5 cm in the short axis) is common in usual interstitial pneumonia.31
Because other chronic interstitial pneumonias that may resemble usual interstitial pneumonia have a more favorable course and may respond to immunosuppressive therapy, establishing an early and accurate diagnosis is of the utmost importance.5 Additionally, the emergence of possible new therapies for idiopathic pulmonary fibrosis makes early referral to a specialist paramount in these cases. Recent studies have demonstrated significant slowing of the progression of disease in idiopathic pulmonary fibrosis with both pirfenidone and nintedanib.32,33
DIAGNOSIS AND MANAGEMENT
The diagnosis of these nonfibrotic and fibrotic lung diseases is complex. In all cases in which interstitial lung disease is detected on screening CT for lung cancer, the internist should strongly consider further evaluation with dedicated high-resolution CT and early referral to a specialist (Figure 15).
Because smoking cessation is the only recommended treatment for nonfibrotic smoking-related interstitial lung diseases, particular emphasis on smoking cessation counseling is essential.
Referral for bronchoscopy with transbronchial lung biopsy is generally not helpful in the diagnosis of the interstitial lung diseases discussed in this article unless there is a need to rule out infection or neoplasm.34 Referral for surgical lung biopsy may be indicated in some cases of suspected pulmonary Langerhans cell histiocytosis, desquamative interstitial pneumonia, nonspecific interstitial pneumonia, or usual interstitial pneumonia if the diagnosis is uncertain (Tables 1 and 2).35
The American Thoracic Society/European Respiratory Society guidelines suggest a multidisciplinary team approach that includes a pathologist, radiologist, and clinician.35 This approach more readily determines the correct diagnosis and relies less on invasive methods such as surgical biopsy and more on noninvasive methods such as radiology and clinical history. Overall, this will promote earlier access to appropriate therapies, clinical trial enrollment, and in more severe cases, lung transplant.
Currently, 23% of all lung transplants worldwide are performed in patients with idiopathic pulmonary fibrosis. Other forms of pulmonary fibrosis account for 3% to 4% of lung transplants performed.36
Evidence suggests that early referral reduces rates of morbidity and death in these patients. The results of a single-center study37 of patients with idiopathic pulmonary fibrosis demonstrated that a longer delay from the onset of symptoms to evaluation by a specialist at a tertiary care referral center was associated with a higher rate of death from this disease independent of disease severity. Those with the longest delay in referral had a multivariable-adjusted death rate 3.4 times higher than those with the shortest delay.5,37
In summary, with implementation of the new lung cancer screening guidelines, primary care physicians are more often encountering the incidental finding of interstitial lung disease in their patients. Prompt diagnosis of interstitial lung disease helps ensure that patients receive appropriate care and early consideration for clinical trials and lung transplant.
Primary care physicians play a critical role in the initial identification of key characteristics of the interstitial abnormality—namely, whether the pattern is nonfibrotic or fibrotic—and in the correlation of the history and physical findings to expedite the diagnosis. Subsequently, ordering high-resolution CT for more detailed characterization and prompt referral to a specialist in interstitial lung disease allow for a more rapid and accurate diagnosis, specialized therapy, and supportive care.
Primary care physicians are playing a bigger role in evaluating the incidental finding of interstitial lung diseases since the recent publication of guidelines recommending computed tomography (CT) to screen for lung cancer.
In August 2011, the National Cancer Institute published its findings from the National Lung Screening Trial, which demonstrated a 20% reduction in mortality from lung cancer in patients at high risk screened with low-dose CT.1 Based on these results, the American Cancer Society, the American College of Chest Physicians, the American Society of Clinical Oncology, and the National Comprehensive Cancer Network recommended annual screening for lung cancer with low-dose CT in adults ages 55 to 74 who have a 30-pack-year smoking history and who currently smoke or have quit within the past 15 years.2 In December 2013, the US Preventive Services Task Force published similar guidelines but increased the age range to include high-risk patients ages 55 to 80.3
Bach et al4 estimated that, in 2010 in the United States, 8.6 million people met the criteria used in the National Lung Screening Trial for low-dose CT screening. These are the same criteria as in the multisociety recommendations cited above.2 With such large numbers of patients eligible for CT screening, internists and other primary care physicians are undoubtedly encountering the incidental discovery of nonmalignant pulmonary diseases such as interstitial lung disease.
This article reviews the radiographic characteristics of the most common interstitial lung diseases the internist may encounter on screening CT in long-term smokers.
Referral to a specialist has been associated with lower rates of morbidity and death,5 and a diagnosis of interstitial lung disease should be confirmed by a pulmonologist and a radiologist specializing in differentiating the subtypes. But the primary care physician now plays a critical role in recognizing the need for further evaluation.
HOW COMMON IS INTERSTITIAL LUNG DISEASE IN SMOKERS?
Several studies have published data on the prevalence of interstitial lung disease in patients undergoing low-dose CT for lung cancer screening.
A trial at Mayo Clinic in current and former smokers identified “diffuse lung disease” in 9 (0.9%) of 1,049 participants.6
A trial in Ireland identified idiopathic pulmonary fibrosis in 6 (1.3%) of 449 current smokers who underwent low-dose CT screening for lung cancer.7
Sverzellati et al8 evaluated 692 participants in the Multicentric Italian Lung Detection CT screening study and reported a respiratory bronchiolitis pattern in 109 (15.7%), a usual interstitial pneumonia pattern in 2 (0.3%), and other patterns of chronic interstitial pneumonia in 26 (3.8%).
The National Lung Screening Trial reported that the frequency of “clinically significant” incidental findings (including pulmonary fibrosis) in all participants was 7.5%.1 A retrospective analysis of 884 participants at a single site in this trial identified interstitial lung abnormalities in 86 participants (9.7%).9 These abnormalities were further categorized as nonfibrotic in 52 (5.9%) of 884, fibrotic in 19 (2.1%) of 884, and mixed fibrotic and nonfibrotic in 15 (1.7%) of 884.
Follow-up CT at 2 years in this trial demonstrated improvement in 50% and progression in 11% of patients who had nonfibrotic abnormalities, while fibrotic abnormalities improved in no cases and progressed in 37%. Interstitial lung abnormalities were more common in those who currently smoked and in those with more pack-years of cigarette smoking.9
In sum, these trials suggest that low-dose CT screening for lung cancer can detect the most common forms of interstitial lung disease in this at-risk population and can characterize them as fibrotic or nonfibrotic, a distinction important for prognosis and subsequent management.
NONFIBROTIC VS FIBROTIC DISEASE
It is important to distinguish between nonfibrotic and fibrotic interstitial lung disease, as fibrotic disease carries a worse prognosis and is treated differently.
Features of nonfibrotic interstitial lung disease:
- Ground-glass opacities
- Nodules
- Mosaic attenuation or consolidation.
Features of fibrotic interstitial lung disease:
- Combination of ground-glass opacities and reticulation
- Reticulation by itself
- Traction bronchiectasis
- Honeycombing
- Loss of lung volume.
NONFIBROTIC INTERSTITIAL LUNG DISEASES
Given the strong likelihood that a patient undergoing screening CT is either a current or former smoker, physicians may encounter, in addition to emphysema and lung cancer, the following smoking-related interstitial lung diseases, which are primarily nonfibrotic and which frequently coexist (Table 1):
- Respiratory bronchiolitis
- Respiratory bronchiolitis-interstitial lung disease
- Desquamative interstitial pneumonia
- Pulmonary Langerhans cell histiocytosis.
Respiratory bronchiolitis
Respiratory bronchiolitis occurs mostly in smokers and does not necessarily lead to respiratory symptoms in all patients.10 It cannot always be identified radiographically but occasionally appears as predominantly upper-lobe, patchy ground-glass opacities or ill-defined centrilobular nodules without evidence of fibrosis (Figure 1).
Respiratory bronchiolitis-interstitial lung disease
In rare cases, respiratory bronchiolitis leads to peribronchial fibrosis invading the alveolar walls, which is then classified as respiratory bronchiolitis-interstitial lung disease.11 The CT findings in respiratory bronchiolitis-interstitial lung disease are upper-lobe-predominant centrilobular ground-glass nodules, patchy ground-glass opacities, and bronchial wall thickening (Figure 2).10 Occasionally, mild reticulation is noted without honeycombing. Mild air trapping can be seen in the lower lobes, with centrilobular emphysema in the upper lobes.12
The only successful therapy for respiratory bronchiolitis and respiratory bronchiolitis-interstitial lung disease is smoking cessation. Finding either of these diseases should prompt aggressive counseling by the internist and consideration of referral to a specialist in interstitial lung disease.
Desquamative interstitial pneumonia
Although pathologically different from respiratory bronchiolitis-interstitial lung disease, desquamative interstitial pneumonia has a similar clinical and radiographic presentation. Because their features significantly overlap, they are considered a pathomorphologic continuum, representing degrees of severity of the same disease process caused by prolonged tobacco inhalation.10,13
Widespread ground-glass opacities are the predominant CT finding. These are bilateral and symmetric in distribution in 86%, basal and peripheral in 60%, patchy in 20%, and diffuse in 20% (Figure 3).14 Other frequent findings are mild reticulation with traction bronchiectasis and coexistent emphysema (Figure 4).15 The small peripheral cystic spaces noted in this disease most likely represent dilated bronchioles and alveolar ducts rather than honeycombing.16
No additional treatment beyond elimination of smoking has been proven effective for desquamative interstitial pneumonia, and patients who manage to quit smoking generally have a favorable prognosis.17,18
Pulmonary Langerhans cell histiocytosis
The combination of upper-lobe-predominant cysts and nodules in a young heavy smoker are diagnostic of pulmonary Langerhans cell histiocytosis. The cysts are bizarrely shaped, thin- or thick-walled, and nonuniform in size (Figure 5). The irregular cavitary nodules are centrilobular. The disease characteristically spares the costophrenic angles.
Spontaneous pneumothorax is the initial clinical presentation in 15% of patients.16 In the early stages of the disease (nodule-predominant disease without cysts), infection and metastatic disease need to be excluded (Figure 6). In the later stages, the cysts become coalescent, making the distinction between this disease and “burned-out” lymphangioleiomyomatosis or severe emphysema extremely difficult (Figure 7).17 Smoking cessation and corticosteroids are the mainstay of medical therapy for pulmonary Langerhans cell histiocytosis, and about 50% of patients who quit smoking and receive corticosteroids demonstrate partial or complete clearing of the radiographic abnormalities and symptoms (Figure 8).
FIBROTIC INTERSTITIAL LUNG DISEASES
If CT identifies a diffuse fibrotic pattern, the two most common possibilities (Table 2) are:
- Nonspecific interstitial pneumonia
- Usual interstitial pneumonia.
As noted above, these carry a worse prognosis than the nonfibrotic interstitial lung diseases.
Nonspecific interstitial pneumonia
While most frequently idiopathic, the nonspecific interstitial pneumonia pattern can often be seen in connective tissue diseases. It has also been associated with chronic hypersensitivity pneumonitis, drug toxicity, and slowly resolving diffuse alveolar damage.19 Although it is not the only pathologic pattern in interstitial lung disease associated with connective tissue disease, it is the most common pattern in systemic sclerosis, systemic lupus erythematosus, dermatomyositis-polymyositis, and mixed connective tissue disease.20
The parenchymal changes are typically subpleural and symmetric in distribution (Figure 9). In about one-third of cases, there is a peribronchovascular distribution of the abnormalities (Figure 10).
Ground-glass opacities are the dominant imaging findings, seen in 80% of cases.18 In advanced disease (also referred to as fibrotic nonspecific interstitial pneumonia), patients have accompanying fine or coarse reticular opacities, traction bronchiectasis, and consolidation (Figure 11). Honeycombing is seen in 1% to 5% of patients.21
The most specific sign of nonspecific interstitial pneumonia is sparing of the immediate subpleural lung, apparent in 30% to 50% of patients (Figure 12).22 Subpleural sparing with a peribronchovascular distribution of abnormalities, absence of lobular areas with decreased attenuation, and lack of honeycombing are imaging features that increase the diagnostic confidence of nonspecific interstitial pneumonia (Table 3).23 Clinically, compared with those who have usual interstitial pneumonia (see below), patients are younger and more of them are female. These patients also present with extrapulmonary manifestations such as joint involvement, rash, and Raynaud phenomenon. Therefore, these associated symptoms on presentation can help distinguish nonspecific interstitial pneumonia or usual interstitial pneumonia related to connective tissue disease from the idiopathic forms.
The first step in managing nonspecific interstitial pneumonia is to remove all potential exposure to inhaled substances or to drugs. Although immunosuppressive therapy has never been studied in a randomized controlled trial in this disease, numerous reports suggest that patients may respond to prednisone and to steroid-sparing immunosuppressants.24
In several studies, survival rates in nonspecific interstitial pneumonia were significantly greater than in usual interstitial pneumonia independent of the treatment strategy. In long-term follow-up of patients with idiopathic nonspecific interstitial pneumonia treated with immunosuppressive therapy, two-thirds remained stable or improved.25–27
Although most connective tissue diseases cause a lung pattern of nonspecific interstitial pneumonia, some (eg, rheumatoid arthritis) may present with a pattern of usual interstitial pneumonia. In these cases and in those of advanced fibrotic nonspecific interstitial pneumonia, the prognosis is worse, as the disease is less responsive to immunosuppressive therapy.20
Usual interstitial pneumonia
Usual interstitial pneumonia is the most severe form of lung fibrosis. Most cases are idiopathic and are termed idiopathic pulmonary fibrosis. Other causes of the usual interstitial pneumonia pattern include domestic and occupational environmental exposures, connective tissue disease, and drug toxicity.28 An epidemiologic association between smoking and usual interstitial pneumonia is well documented.28
Idiopathic pulmonary fibrosis typically affects men ages 50 to 70. Because its risk factors coincide with those of lung cancer, there is a high likelihood of detecting idiopathic pulmonary fibrosis early in this screening population. It has an especially poor prognosis, with a mean survival of 2 to 5 years from the time of diagnosis.18
The distribution of disease in usual interstitial pneumonia is characteristically subpleural and basal. CT features include coarse subpleural reticulation and honeycombing combined with traction bronchiectasis or bronchiolectasis and architectural distortion (Figure 13).18 Honeycombing is the most specific and key diagnostic CT finding for establishing a definitive diagnosis of usual interstitial pneumonia.29 However, ground-glass opacities are present in most patients, typically in the region of interstitial fibrosis, and are always less extensive than the reticulation.30 The findings demonstrate morphologic heterogeneity, with areas of fibrosis adjacent to areas of normal lung (Figure 14).
In addition to the aforementioned imaging features, the 2011 American Thoracic Society and European Respiratory Society joint guidelines for the CT diagnosis of usual interstitial pneumonia patterns require the absence of atypical features that suggest an alternative diagnosis, including those seen in nonspecific interstitial pneumonia, such as an upper, midlung, or peribronchovascular distribution and extensive ground-glass attenuation.28 Mild mediastinal lymphadenopathy (usually < 1.5 cm in the short axis) is common in usual interstitial pneumonia.31
Because other chronic interstitial pneumonias that may resemble usual interstitial pneumonia have a more favorable course and may respond to immunosuppressive therapy, establishing an early and accurate diagnosis is of the utmost importance.5 Additionally, the emergence of possible new therapies for idiopathic pulmonary fibrosis makes early referral to a specialist paramount in these cases. Recent studies have demonstrated significant slowing of the progression of disease in idiopathic pulmonary fibrosis with both pirfenidone and nintedanib.32,33
DIAGNOSIS AND MANAGEMENT
The diagnosis of these nonfibrotic and fibrotic lung diseases is complex. In all cases in which interstitial lung disease is detected on screening CT for lung cancer, the internist should strongly consider further evaluation with dedicated high-resolution CT and early referral to a specialist (Figure 15).
Because smoking cessation is the only recommended treatment for nonfibrotic smoking-related interstitial lung diseases, particular emphasis on smoking cessation counseling is essential.
Referral for bronchoscopy with transbronchial lung biopsy is generally not helpful in the diagnosis of the interstitial lung diseases discussed in this article unless there is a need to rule out infection or neoplasm.34 Referral for surgical lung biopsy may be indicated in some cases of suspected pulmonary Langerhans cell histiocytosis, desquamative interstitial pneumonia, nonspecific interstitial pneumonia, or usual interstitial pneumonia if the diagnosis is uncertain (Tables 1 and 2).35
The American Thoracic Society/European Respiratory Society guidelines suggest a multidisciplinary team approach that includes a pathologist, radiologist, and clinician.35 This approach more readily determines the correct diagnosis and relies less on invasive methods such as surgical biopsy and more on noninvasive methods such as radiology and clinical history. Overall, this will promote earlier access to appropriate therapies, clinical trial enrollment, and in more severe cases, lung transplant.
Currently, 23% of all lung transplants worldwide are performed in patients with idiopathic pulmonary fibrosis. Other forms of pulmonary fibrosis account for 3% to 4% of lung transplants performed.36
Evidence suggests that early referral reduces rates of morbidity and death in these patients. The results of a single-center study37 of patients with idiopathic pulmonary fibrosis demonstrated that a longer delay from the onset of symptoms to evaluation by a specialist at a tertiary care referral center was associated with a higher rate of death from this disease independent of disease severity. Those with the longest delay in referral had a multivariable-adjusted death rate 3.4 times higher than those with the shortest delay.5,37
In summary, with implementation of the new lung cancer screening guidelines, primary care physicians are more often encountering the incidental finding of interstitial lung disease in their patients. Prompt diagnosis of interstitial lung disease helps ensure that patients receive appropriate care and early consideration for clinical trials and lung transplant.
Primary care physicians play a critical role in the initial identification of key characteristics of the interstitial abnormality—namely, whether the pattern is nonfibrotic or fibrotic—and in the correlation of the history and physical findings to expedite the diagnosis. Subsequently, ordering high-resolution CT for more detailed characterization and prompt referral to a specialist in interstitial lung disease allow for a more rapid and accurate diagnosis, specialized therapy, and supportive care.
- National Lung Screening Trial Research Team; Aberle DR, Adams AM, Berg CD, et al. Reduced lung-cancer mortality with low-dose computed tomographic screening. N Engl J Med 2011; 365:395–409.
- Detterbeck FC, Lewis SZ, Diekemper R, Addrizzo-Harris D, Alberts WM. Executive summary: diagnosis and management of lung cancer, 3rd ed: American College of Chest Physicians evidence-based clinical practice guidelines. Chest 2013; 143(suppl 5):7S–37S.
- Moyer VA; US Preventive Services Task Force. Screening for lung cancer: US Preventive Services Task Force recommendation statement. Ann Intern Med 2014; 160:330–338.
- Bach PB, Mirkin JN, Oliver TK, et al. Benefits and harms of CT screening for lung cancer: a systematic review. JAMA 2012; 307:2418–2429.
- Lamas DJ, Kawut SM, Bagiella E, Philip N, Arcasoy SM, Lederer DJ. Delayed access and survival in idiopathic pulmonary fibrosis: a cohort study. Am J Respir Crit Care Med 2011; 184:842–847.
- Swensen SJ, Jett JR, Hartman TE, et al. Lung cancer screening with CT: Mayo Clinic experience. Radiology 2003; 226:756–761.
- MacRedmond R, Logan PM, Lee M, Kenny D, Foley C, Costello RW. Screening for lung cancer using low dose CT scanning. Thorax 2004; 59:237–241.
- Sverzellati N, Guerci L, Randi G, et al. Interstitial lung diseases in a lung cancer screening trial. Eur Respir J 2011; 38:392–400.
- Jin GY, Lynch D, Chawla A, et al. Interstitial lung abnormalities in a CT lung cancer screening population: prevalence and progression rate. Radiology 2013; 268:563–571.
- Heyneman LE, Ward S, Lynch DA, Remy-Jardin M, Johkoh T, Müller NL. Respiratory bronchiolitis, respiratory bronchiolitis-associated interstitial lung disease, and desquamative interstitial pneumonia: different entities or part of the spectrum of the same disease process? AJR Am J Roentgenol 1999; 173:1617–1622.
- Moon J, du Bois RM, Colby TV, Hansell DM, Nicholson AG. Clinical significance of respiratory bronchiolitis on open lung biopsy and its relationship to smoking related interstitial lung disease. Thorax 1999; 54:1009–1014.
- Holt RM, Schmidt RA, Godwin JD, Raghu G. High resolution CT in respiratory bronchiolitis-associated interstitial lung disease. J Comput Assist Tomogr 1993; 17:46–50.
- Ryu JH, Myers JL, Capizzi SA, Douglas WW, Vassallo R, Decker PA. Desquamative interstitial pneumonia and respiratory bronchiolitis-associated interstitial lung disease. Chest 2005; 127:178–184.
- Hartman TE, Primack SL, Swensen SJ, Hansell D, McGuinness G, Müller NL. Desquamative interstitial pneumonia: thin-section CT findings in 22 patients. Radiology 1993; 187:787–790.
- Akira M, Yamamoto S, Hara H, Sakatani M, Ueda E. Serial computed tomographic evaluation in desquamative interstitial pneumonia. Thorax 1997; 52:333–337.
- Lacronique J, Roth C, Battesti JP, Basset F, Chretien J. Chest radiological features of pulmonary histiocytosis X: a report based on 50 adult cases. Thorax 1982; 37:104–109.
- Remy-Jardin M, Edme JL, Boulenguez C, Remy J, Mastora I, Sobaszek A. Longitudinal follow-up study of smoker’s lung with thin-section CT in correlation with pulmonary function tests. Radiology 2002; 222:261–270.
- Mueller-Mang C, Grosse C, Schmid K, Stiebellehner L, Bankier AA. What every radiologist should know about idiopathic interstitial pneumonias. Radiographics 2007; 27:595–615.
- Katzenstein AL, Fiorelli RF. Nonspecific interstitial pneumonia/fibrosis. Histologic features and clinical significance. Am J Surg Pathol 1994; 18:136–147.
- Bryson T, Sundaram B, Khanna D, Kazerooni EA. Connective tissue disease-associated interstitial pneumonia and idiopathic interstitial pneumonia: similarity and difference. Semin Ultrasound CT MR 2014; 35:29–38.
- Desai SR, Veeraraghavan S, Hansell DM, et al. CT features of lung disease in patients with systemic sclerosis: comparison with idiopathic pulmonary fibrosis and nonspecific interstitial pneumonia. Radiology 2004; 232:560–567.
- Tsubamoto M, Müller NL, Johkoh T, et al. Pathologic subgroups of nonspecific interstitial pneumonia: differential diagnosis from other idiopathic interstitial pneumonias on high-resolution computed tomography. J Comput Assist Tomogr 2005; 29:793–800.
- Silva CI, Müller NL, Lynch DA, et al. Chronic hypersensitivity pneumonitis: differentiation from idiopathic pulmonary fibrosis and nonspecific interstitial pneumonia by using thin-section CT. Radiology 2008; 246:288–297.
- Antin-Ozerkis D, Rubinowitz A. An update on nonspecific interstitial pneumonia. Clin Pulm Med 2010; 17:122–128.
- Daniil ZD, Gilchrist FC, Nicholson AG, et al. A histologic pattern of nonspecific interstitial pneumonia is associated with a better prognosis than usual interstitial pneumonia in patients with cryptogenic fibrosing alveolitis. Am J Respir Crit Care Med 1999; 160:899–905.
- Travis WD, Matsui K, Moss J, Ferrans VJ. Idiopathic nonspecific interstitial pneumonia: prognostic significance of cellular and fibrosing patterns: survival comparison with usual interstitial pneumonia and desquamative interstitial pneumonia. Am J Surg Pathol 2000; 24:19–33.
- Riha RL, Duhig EE, Clarke BE, Steele RH, Slaughter RE, Zimmerman PV. Survival of patients with biopsy-proven usual interstitial pneumonia and nonspecific interstitial pneumonia. Eur Respir J 2002; 19:1114–1118.
- Raghu G, Collard HR, Egan JJ, et al; ATS/ERS/JRS/ALAT Committee on Idiopathic Pulmonary Fibrosis. An official ATS/ERS/JRS/ALAT statement: idiopathic pulmonary fibrosis: evidence-based guidelines for diagnosis and management. Am J Respir Crit Care Med 2011; 183:788–824.
- du Bois RM. An earlier and more confident diagnosis of idiopathic pulmonary fibrosis. Eur Respir Rev 2012; 21:141–146.
- Nishimura K, Kitaichi M, Izumi T, Nagai S, Kanaoka M, Itoh H. Usual interstitial pneumonia: histologic correlation with high-resolution CT. Radiology 1992; 182:337–342.
- Souza CA, Müller NL, Lee KS, Johkoh T, Mitsuhiro H, Chong S. Idiopathic interstitial pneumonias: prevalence of mediastinal lymph node enlargement in 206 patients. AJR Am J Roentgenol 2006; 186:995–999.
- King TE Jr, Bradford WZ, Castro-Bernardini S, et al; ASCEND Study Group. A phase 3 trial of pirfenidone in patients with idiopathic pulmonary fibrosis. N Engl J Med 2014; 370:2083–2092.
- Richeldi L, du Bois RM, Raghu G, et al; INPULSIS Trial Investigators. Efficacy and safety of nintedanib in idiopathic pulmonary fibrosis. N Engl J Med 2014; 370:2071–2082.
- Bradley B, Branley HM, Egan JJ, et al; British Thoracic Society Interstitial Lung Disease Guideline Group, British Thoracic Society Standards of Care Committee; Thoracic Society of Australia; New Zealand Thoracic Society; Irish Thoracic Society. Interstitial lung disease guideline: the British Thoracic Society in collaboration with the Thoracic Society of Australia and New Zealand and the Irish Thoracic Society. Thorax 2008; 63(suppl 5):v1–v58.
- Travis WD, Costabel U, Hansell DM, et al; ATS/ERS Committee on Idiopathic Interstitial Pneumonias. An official American Thoracic Society/European Respiratory Society statement: update of the international multidisciplinary classification of the idiopathic interstitial pneumonias. Am J Respir Crit Care Med 2013; 188:733–748.
- Stehlik J, Edwards LB, Kucheryavaya AY, et al; International Society of Heart and Lung Transplantation. The Registry of the International Society for Heart and Lung Transplantation: 29th official adult heart transplant report—2012. J Heart Lung Transplant 2012; 31:1052–1064.
- Oldham JM, Noth I. Idiopathic pulmonary fibrosis: early detection and referral. Respir Med 2014; 108:819–829.
- National Lung Screening Trial Research Team; Aberle DR, Adams AM, Berg CD, et al. Reduced lung-cancer mortality with low-dose computed tomographic screening. N Engl J Med 2011; 365:395–409.
- Detterbeck FC, Lewis SZ, Diekemper R, Addrizzo-Harris D, Alberts WM. Executive summary: diagnosis and management of lung cancer, 3rd ed: American College of Chest Physicians evidence-based clinical practice guidelines. Chest 2013; 143(suppl 5):7S–37S.
- Moyer VA; US Preventive Services Task Force. Screening for lung cancer: US Preventive Services Task Force recommendation statement. Ann Intern Med 2014; 160:330–338.
- Bach PB, Mirkin JN, Oliver TK, et al. Benefits and harms of CT screening for lung cancer: a systematic review. JAMA 2012; 307:2418–2429.
- Lamas DJ, Kawut SM, Bagiella E, Philip N, Arcasoy SM, Lederer DJ. Delayed access and survival in idiopathic pulmonary fibrosis: a cohort study. Am J Respir Crit Care Med 2011; 184:842–847.
- Swensen SJ, Jett JR, Hartman TE, et al. Lung cancer screening with CT: Mayo Clinic experience. Radiology 2003; 226:756–761.
- MacRedmond R, Logan PM, Lee M, Kenny D, Foley C, Costello RW. Screening for lung cancer using low dose CT scanning. Thorax 2004; 59:237–241.
- Sverzellati N, Guerci L, Randi G, et al. Interstitial lung diseases in a lung cancer screening trial. Eur Respir J 2011; 38:392–400.
- Jin GY, Lynch D, Chawla A, et al. Interstitial lung abnormalities in a CT lung cancer screening population: prevalence and progression rate. Radiology 2013; 268:563–571.
- Heyneman LE, Ward S, Lynch DA, Remy-Jardin M, Johkoh T, Müller NL. Respiratory bronchiolitis, respiratory bronchiolitis-associated interstitial lung disease, and desquamative interstitial pneumonia: different entities or part of the spectrum of the same disease process? AJR Am J Roentgenol 1999; 173:1617–1622.
- Moon J, du Bois RM, Colby TV, Hansell DM, Nicholson AG. Clinical significance of respiratory bronchiolitis on open lung biopsy and its relationship to smoking related interstitial lung disease. Thorax 1999; 54:1009–1014.
- Holt RM, Schmidt RA, Godwin JD, Raghu G. High resolution CT in respiratory bronchiolitis-associated interstitial lung disease. J Comput Assist Tomogr 1993; 17:46–50.
- Ryu JH, Myers JL, Capizzi SA, Douglas WW, Vassallo R, Decker PA. Desquamative interstitial pneumonia and respiratory bronchiolitis-associated interstitial lung disease. Chest 2005; 127:178–184.
- Hartman TE, Primack SL, Swensen SJ, Hansell D, McGuinness G, Müller NL. Desquamative interstitial pneumonia: thin-section CT findings in 22 patients. Radiology 1993; 187:787–790.
- Akira M, Yamamoto S, Hara H, Sakatani M, Ueda E. Serial computed tomographic evaluation in desquamative interstitial pneumonia. Thorax 1997; 52:333–337.
- Lacronique J, Roth C, Battesti JP, Basset F, Chretien J. Chest radiological features of pulmonary histiocytosis X: a report based on 50 adult cases. Thorax 1982; 37:104–109.
- Remy-Jardin M, Edme JL, Boulenguez C, Remy J, Mastora I, Sobaszek A. Longitudinal follow-up study of smoker’s lung with thin-section CT in correlation with pulmonary function tests. Radiology 2002; 222:261–270.
- Mueller-Mang C, Grosse C, Schmid K, Stiebellehner L, Bankier AA. What every radiologist should know about idiopathic interstitial pneumonias. Radiographics 2007; 27:595–615.
- Katzenstein AL, Fiorelli RF. Nonspecific interstitial pneumonia/fibrosis. Histologic features and clinical significance. Am J Surg Pathol 1994; 18:136–147.
- Bryson T, Sundaram B, Khanna D, Kazerooni EA. Connective tissue disease-associated interstitial pneumonia and idiopathic interstitial pneumonia: similarity and difference. Semin Ultrasound CT MR 2014; 35:29–38.
- Desai SR, Veeraraghavan S, Hansell DM, et al. CT features of lung disease in patients with systemic sclerosis: comparison with idiopathic pulmonary fibrosis and nonspecific interstitial pneumonia. Radiology 2004; 232:560–567.
- Tsubamoto M, Müller NL, Johkoh T, et al. Pathologic subgroups of nonspecific interstitial pneumonia: differential diagnosis from other idiopathic interstitial pneumonias on high-resolution computed tomography. J Comput Assist Tomogr 2005; 29:793–800.
- Silva CI, Müller NL, Lynch DA, et al. Chronic hypersensitivity pneumonitis: differentiation from idiopathic pulmonary fibrosis and nonspecific interstitial pneumonia by using thin-section CT. Radiology 2008; 246:288–297.
- Antin-Ozerkis D, Rubinowitz A. An update on nonspecific interstitial pneumonia. Clin Pulm Med 2010; 17:122–128.
- Daniil ZD, Gilchrist FC, Nicholson AG, et al. A histologic pattern of nonspecific interstitial pneumonia is associated with a better prognosis than usual interstitial pneumonia in patients with cryptogenic fibrosing alveolitis. Am J Respir Crit Care Med 1999; 160:899–905.
- Travis WD, Matsui K, Moss J, Ferrans VJ. Idiopathic nonspecific interstitial pneumonia: prognostic significance of cellular and fibrosing patterns: survival comparison with usual interstitial pneumonia and desquamative interstitial pneumonia. Am J Surg Pathol 2000; 24:19–33.
- Riha RL, Duhig EE, Clarke BE, Steele RH, Slaughter RE, Zimmerman PV. Survival of patients with biopsy-proven usual interstitial pneumonia and nonspecific interstitial pneumonia. Eur Respir J 2002; 19:1114–1118.
- Raghu G, Collard HR, Egan JJ, et al; ATS/ERS/JRS/ALAT Committee on Idiopathic Pulmonary Fibrosis. An official ATS/ERS/JRS/ALAT statement: idiopathic pulmonary fibrosis: evidence-based guidelines for diagnosis and management. Am J Respir Crit Care Med 2011; 183:788–824.
- du Bois RM. An earlier and more confident diagnosis of idiopathic pulmonary fibrosis. Eur Respir Rev 2012; 21:141–146.
- Nishimura K, Kitaichi M, Izumi T, Nagai S, Kanaoka M, Itoh H. Usual interstitial pneumonia: histologic correlation with high-resolution CT. Radiology 1992; 182:337–342.
- Souza CA, Müller NL, Lee KS, Johkoh T, Mitsuhiro H, Chong S. Idiopathic interstitial pneumonias: prevalence of mediastinal lymph node enlargement in 206 patients. AJR Am J Roentgenol 2006; 186:995–999.
- King TE Jr, Bradford WZ, Castro-Bernardini S, et al; ASCEND Study Group. A phase 3 trial of pirfenidone in patients with idiopathic pulmonary fibrosis. N Engl J Med 2014; 370:2083–2092.
- Richeldi L, du Bois RM, Raghu G, et al; INPULSIS Trial Investigators. Efficacy and safety of nintedanib in idiopathic pulmonary fibrosis. N Engl J Med 2014; 370:2071–2082.
- Bradley B, Branley HM, Egan JJ, et al; British Thoracic Society Interstitial Lung Disease Guideline Group, British Thoracic Society Standards of Care Committee; Thoracic Society of Australia; New Zealand Thoracic Society; Irish Thoracic Society. Interstitial lung disease guideline: the British Thoracic Society in collaboration with the Thoracic Society of Australia and New Zealand and the Irish Thoracic Society. Thorax 2008; 63(suppl 5):v1–v58.
- Travis WD, Costabel U, Hansell DM, et al; ATS/ERS Committee on Idiopathic Interstitial Pneumonias. An official American Thoracic Society/European Respiratory Society statement: update of the international multidisciplinary classification of the idiopathic interstitial pneumonias. Am J Respir Crit Care Med 2013; 188:733–748.
- Stehlik J, Edwards LB, Kucheryavaya AY, et al; International Society of Heart and Lung Transplantation. The Registry of the International Society for Heart and Lung Transplantation: 29th official adult heart transplant report—2012. J Heart Lung Transplant 2012; 31:1052–1064.
- Oldham JM, Noth I. Idiopathic pulmonary fibrosis: early detection and referral. Respir Med 2014; 108:819–829.
KEY POINTS
- Smoking-related interstitial lung diseases can broadly be categorized as fibrotic or nonfibrotic on the basis of their appearance on CT. Fibrotic disease generally carries a worse prognosis.
- Nonfibrotic interstitial lung diseases include respiratory bronchiolitis, respiratory bronchiolitis-interstitial lung disease, desquamative interstitial pneumonia, and pulmonary Langerhans cell histiocytosis.
- Smoking-related fibrotic interstitial lung diseases include nonspecific interstitial pneumonia and usual interstitial pneumonia. A subset of usual interstitial pneumonia, called idiopathic pulmonary fibrosis, carries the worst prognosis of all.
- If CT detects interstitial lung disease during screening for lung cancer, the clinician should strongly consider further evaluation with dedicated high-resolution CT and early referral to a specialist. Smoking cessation is extremely important.
Alcohol withdrawal syndrome in medical patients
Deprived of alcohol while in the hospital, up to 80% of patients who are alcohol-dependent risk developing alcohol withdrawal syndrome,1 a potentially life-threatening condition. Clinicians should anticipate the syndrome and be ready to treat and prevent its complications.
Because alcoholism is common, nearly every provider will encounter its complications and withdrawal symptoms. Each year, an estimated 1.2 million hospital admissions are related to alcohol abuse, and about 500,000 episodes of withdrawal symptoms are severe enough to require clinical attention.1–3 Nearly 50% of patients with alcohol withdrawal syndrome are middle-class, highly functional individuals, making withdrawal difficult to recognize.1
While acute trauma patients or those with alcohol withdrawal delirium are often admitted directly to an intensive care unit (ICU), many others are at risk for or develop alcohol withdrawal syndrome and are managed initially or wholly on the acute medical unit. While specific statistics have not been published on non-ICU patients with alcohol withdrawal syndrome, they are an important group of patients who need to be well managed to prevent the progression of alcohol withdrawal syndrome to alcohol withdrawal delirium, alcohol withdrawal-induced seizure, and other complications.
This article reviews how to identify and manage alcohol withdrawal symptoms in noncritical, acutely ill medical patients, with practical recommendations for diagnosis and management.
CAN LEAD TO DELIRIUM TREMENS
In people who are physiologically dependent on alcohol, symptoms of withdrawal usually occur after abrupt cessation.4 If not addressed early in the hospitalization, alcohol withdrawal syndrome can progress to alcohol withdrawal delirium (also known as delirium tremens or DTs), in which the mortality rate is 5% to 10%.5,6 Potential mechanisms of DTs include increased dopamine release and dopamine receptor activity, hypersensitivity to N-methyl-d-aspartate, and reduced levels of gamma-aminobutyric acid (GABA).7
Long-term changes are thought to occur in neurons after repeated detoxification from alcohol, a phenomenon called “kindling.” After each detoxification, alcohol craving and obsessive thoughts increase,8 and subsequent episodes of alcohol withdrawal tend to be progressively worse.
Withdrawal symptoms
Alcohol withdrawal syndrome encompasses a spectrum of symptoms and conditions, from minor (eg, insomnia, tremulousness) to severe (seizures, DTs).2 The symptoms typically depend on the amount of alcohol consumed, the time since the last drink, and the number of previous detoxifications.9
The Diagnostic and Statistical Manual of Mental Disorders, Fifth Edition,1 states that to establish a diagnosis of alcohol withdrawal syndrome, a patient must meet four criteria:
- The patient must have ceased or reduced alcohol intake after heavy or prolonged use.
- Two or more of the following must develop within a few hours to a few days: autonomic hyperactivity (sweating or pulse greater than 100 beats per minute); increased hand tremor; insomnia; nausea or vomiting; transient visual, tactile, or auditory hallucinations or illusions; psychomotor agitation; anxiety; grand mal seizure.
- The above symptoms must cause significant distress or functional impairment.
- The symptoms must not be related to another medical condition.
Some of the symptoms described in the second criterion above can occur while the patient still has a measurable blood alcohol level, usually within 6 hours of cessation of drinking.10 Table 1 describes the timetable of onset of symptoms and their severity.2
The elderly may be affected more severely
While the progression of the symptoms described above is commonly used for medical inpatients, the timeline may be different in an elderly patient. Compared with younger patients, elderly patients may have higher blood alcohol concentrations owing to lower total body water, so small amounts of alcohol can produce significant effects.11,12 Brower et al12 found that elderly patients experienced more withdrawal symptoms, especially cognitive impairment, weakness, and high blood pressure, and for 3 days longer.
In the elderly, alcohol may have a greater impact on the central nervous system because of increased permeability of the blood-brain barrier. And importantly, elderly patients tend to have more concomitant diseases and take more medications, all of which can affect alcohol metabolism.
ASSESSMENT SCALES FOR ALCOHOL WITHDRAWAL SYNDROME
A number of clinical scales for evaluating alcohol withdrawal have been developed. Early ones such as the 30-item Total Severity Assessment (TSA) scale and the 11-item Selected Severity Assessment (SSA) scale were limited because they were extremely detailed, burdensome to nursing staff to administer, and contained items such as daily “sleep disturbances” that were not acute enough to meet specific monitoring needs or to guide drug therapy.13,14
The Clinical Institute Withdrawal Assessment for alcohol (CIWA-A) scale, with 15 items, was derived from the SSA scale and includes acute items for assessment as often as every half-hour.15
The CIWA-Ar scale (Table 2) was developed from the CIWA-A scale by Sullivan et al.15 Using both observation and interview, it focuses on 10 areas: nausea and vomiting, tremor, paroxysmal sweats, anxiety, agitation, headache, disorientation, tactile disturbances, auditory disturbances, and visual disturbances. Scores can range from 0 to 67; a higher score indicates worse withdrawal symptoms and outcomes and therefore necessitates escalation of treatment.
The CIWA-Ar scale is now the one most commonly used in clinical trials16–20 and, we believe, in practice. Other scales, including the CIWA-AD and the Alcohol Withdrawal Scale have been validated but are not widely used in practice.14,21
BASELINE ASSESSMENT AND EARLY SUPPORTIVE CARE
A thorough history and physical examination should be performed on admission in patients known to be or suspected of being alcohol-dependent to assess the patient’s affected body systems. The time elapsed since the patient’s last alcohol drink helps predict the onset of withdrawal complications.
Baseline laboratory tests for most patients with suspected alcohol withdrawal syndrome should include a basic blood chemistry panel, complete blood cell count, and possibly an alcohol and toxicology screen, depending on the patient’s history and presentation.
Hydration and nutritional support are important in patients presenting with alcohol withdrawal syndrome. Severe disturbances in electrolytes can lead to serious complications, including cardiac arrhythmia. Close monitoring and electrolyte replacement as needed are recommended for hospitalized alcoholic patients and should follow hospital protocols.22
Thiamine and folic acid status deserve special attention, since long-standing malnutrition is common in alcoholic patients. Thiamine deficiency can result in Wernicke encephalopathy and Korsakoff syndrome, characterized by delirium, ataxia, vision changes, and amnesia. Alcohol withdrawal guidelines recommend giving thiamine intravenously for the first 2 to 5 days after admission.23 In addition, thiamine must be given before any intravenous glucose product, as thiamine is a cofactor in carbohydrate metabolism.23 Folic acid should also be supplemented, as chronic deficiencies may lead to megaloblastic or macrocytic anemia.
CIWA-Ar scale. To provide consistent monitoring and ongoing treatment, clinicians and institutions are encouraged to use a simple assessment scale that detects and quantifies alcohol withdrawal syndrome and that can be used for reassessment after an intervention.21 The CIWA-Ar scale should be used to facilitate “symptom-triggered therapy” in which, depending on the score, the patient receives pharmacologic treatment followed by a scheduled reevaluation.23,24 Most patients with a CIWA-Ar score of 8 or higher benefit from benzodiazepine therapy.16,18,19
PRIMARY DRUG THERAPIES FOR MEDICAL INPATIENTS
Benzodiazepines are the first-line agents
Benzodiazepines are the first-line agents recommended for preventing and treating alcohol withdrawal syndrome.23 Their various pharmacokinetic profiles, wide therapeutic indices, and safety compared with older sedative hypnotics make them the preferred class.23,25 No single benzodiazepine is preferred over the others for treating alcohol withdrawal syndrome: studies have shown benefits using short-acting, intermediate-acting, and long-acting agents. The choice of drug is variable and patient-specific.16,18,26
Benzodiazepines promote and enhance binding of the inhibitory neurotransmitter GABA to GABAA receptors in the central nervous system.27 As a class, benzodiazepines are all structurally related and produce the same effects—namely, sedation, hypnosis, decreased anxiety, muscle relaxation, anterograde amnesia, and anticonvulsant activity.27
The most studied benzodiazepines for treating and preventing alcohol withdrawal syndrome are chlordiazepoxide, oxazepam, and lorazepam,16–20 whereas diazepam was used in older studies.23
Diazepam and chlordiazepoxide are metabolized by oxidation, each sharing the long-acting active metabolite desmethyldiazepam (half-life 72 hours), and short-acting metabolite oxazepam (half-life 8 hours).27 In addition, the parent drugs also have varying pharmacokinetic profiles: diazepam has a half-life of more than 30 hours and chlordiazepoxide a half-life of about 8 hours. Chlordiazepoxide and diazepam’s combination of both long- and short-acting benzodiazepine activity provides long-term efficacy in attenuating withdrawal symptoms, but chlordiazepoxide’s shorter parent half-life allows more frequent dosing.
Lorazepam (half-life 10–20 hours) and oxazepam (half-life 5–20 hours) undergo glucuronide conjugation and do not have metabolites.27,28 Table 3 provides a pharmacokinetic summary.27,28
Various dosage regimens are used in giving benzodiazepines, the most common being symptom-triggered therapy, governed by assessment scales, and scheduled around-the-clock therapy.29 Current evidence supports symptom-triggered therapy in most inpatients who are not critically ill, as it can reduce both benzodiazepine use and adverse drug events and can reduce the length of stay.16,19
Trials of symptom-triggered benzodiazepine therapy
Most inpatient trials of symptom-triggered therapy (Table 4)3,16–20 used the CIWA-Ar scale for monitoring. In some of the studies, benzodiazepines were given if the score was 8 or higher, but others used cut points as high as 15 or higher. Doses:
- Chlordiazepoxide (first dose 25–100 mg)
- Lorazepam (first dose 0.5–2 mg)
- Oxazepam (30 mg).
After each dose, patients were reevaluated at intervals of 30 minutes to 8 hours.
Most of these trials showed no difference in rates of adverse drug events such as seizures, hallucinations, and lethargy with symptom-triggered therapy compared with scheduled therapy.16,18,20 They also found either no difference in the incidence of delirium tremens, or a lower incidence of delirium tremens with symptom-triggered therapy than with scheduled therapy.16–18,20
Weaver et al19 found no difference in length of stay between scheduled therapy and symptom-triggered therapy, but Saitz et al16 reported a median benzodiazepine treatment duration of 9 hours with symptom-triggered therapy vs 68 hours with fixed dosing. Thus, the study by Saitz et al suggests that hospitalization might be shorter with symptom-triggered therapy.
Many of the trials had notable limitations related to the diversity of patients enrolled and the protocols for both symptom-triggered therapy and fixed dosing. Some trials enrolled only inpatients in detoxification programs; others focused on inpatients with acute medical illness. The inpatient alcohol treatment trials16,18 excluded medically ill patients and those with concurrent psychiatric illness,16,18 and one excluded patients with seizures.16 One of the inpatient alcohol treatment program trials16 excluded patients on beta-blockers or clonidine because of concern that these drugs could mask withdrawal symptoms, whereas trials in medically ill patients allowed these same drugs.17,19,20
Most of the patients were men (approximately 75%, but ranging from 74% to 100%), and therefore the study results may not be as applicable to women.16–20 Most participants were middle-aged, with average ages in all studies between 46 and 55. Finally, the studies used a wide range of medications and dosing, with patient monitoring intervals ranging from every 30 minutes to every 8 hours.16–20
In a 2010 Cochrane analysis, Amato et al29 concluded that the limited evidence available favors symptom-triggered regimens over fixed-dosing regimens, but that differences in isolated trials should be interpreted very cautiously.
Therapeutic ethanol
Aside from the lack of evidence to support its use in alcohol withdrawal syndrome, prescribing oral ethanol to alcoholic patients clearly poses an ethical dilemma. However, giving ethanol intravenously has been studied, mostly in surgical trauma patients.30
Early reports described giving intravenous ethanol on a gram-to-gram basis to match the patient’s consumption before admission to prevent alcohol withdrawal syndrome. But later studies reported prevention of alcohol withdrawal syndrome with very small amounts of intravenous ethanol.30,31 While clinical trials have been limited to ICU patients, ethanol infusion at an initial rate of 2.5 to 5 g per hour and titrated up to 10 g per hour has appeared to be safe and effective for preventing alcohol withdrawal syndrome.30,31 The initial infusion rate of 2.5 to 5 g per hour is equivalent to 4 to 10 alcoholic beverages per 24 hours.
Nevertheless, ethanol infusion carries the potential for toxicities (eg, gastric irritation, precipitation of acute hepatic failure, hypoglycemia, pancreatitis, bone marrow suppression, prolonged wound healing) and drug interactions (eg, with anticoagulants and anticonvulsants). Thus, ethanol is neither widely used nor recommended.25,31
ADJUNCTIVE THERAPIES
Many medications are used adjunctively in the acute setting, both for symptoms of alcohol withdrawal syndrome and for agitation.
Haloperidol
No clinical trial has yet examined haloperidol monotherapy in patients with alcohol withdrawal syndrome in either general medical units or intensive care units. Yet haloperidol remains important and is recommended as an adjunct therapy for agitation.23,32 Dosing of haloperidol in protocols for surgical patients ranged from 2 to 5 mg intravenously every 0.5 to 2 hours, with a maximum dosage of 0.5 mg per kg per 24 hours.7,33
Alpha-2 agonists
Alpha-2 agonists are thought to reduce sympathetic overdrive and the autonomic symptoms associated with alcohol withdrawal syndrome, and these agents (primarily clonidine) have been studied in the treatment of alcohol withdrawal syndrome.34,35
Clonidine. In a Swedish study,34 26 men ages 20 to 55 who presented with the tremor, sweating, dysphoria, tension, anxiety, and tachycardia associated with alcohol withdrawal syndrome received clonidine 4 µg per kg twice daily or carbamazepine 200 mg three to four times daily in addition to an antiepileptic. Adjunctive use of a benzodiazepine was allowed at night in both groups. No statistically significant difference in symptom reduction was noted between the two groups, and there was no difference in total benzodiazepine use.
Dexmedetomidine, given intravenously, has been tested as an adjunct to benzodiazepine treatment in severe alcohol withdrawal syndrome. It has been shown to decrease the amount of total benzodiazepine needed compared with benzodiazepine therapy alone, but no differences have been seen in length of hospital stay.36–39 However, research on this drug so far is limited to ICU patients.
Beta-blockers
Beta-blockers have been used in inpatients with alcohol withdrawal syndrome to reduce heart rate and potentially reduce alcohol craving. However, the data are limited and conflicting.
Atenolol 50 to 100 mg daily, in a study in 120 patients, reduced length of stay (4 vs 5 days), reduced benzodiazepine use, and improved vital signs and behavior compared with placebo.40
Propranolol 40 mg every 6 hours reduced arrhythmias but increased hallucinations when used alone in a study in 47 patients.41 When used in combination with chlordiazepoxide, no benefit was seen in arrhythmia reduction.41
Barbiturates and other antiepileptics
Data continue to emerge on antiepileptics as both monotherapy and adjunctive therapy for alcohol withdrawal syndrome. Barbiturates as monotherapy were largely replaced by benzodiazepines in view of the narrow therapeutic index of barbiturates and their full agonist effect on the GABA receptor complex. However, phenobarbital has been evaluated in patients presenting with severe alcohol withdrawal syndrome or resistant alcohol withdrawal (ie, symptoms despite large or repeated doses of benzodiazepines) as an adjunct to benzodiazepines.42,43
In addition, a newer trial44 involved giving a single dose of phenobarbital in the emergency department in combination with a CIWA-Ar–based benzodiazepine protocol, compared with the benzodiazepine protocol alone. The group that received phenobarbital had fewer ICU admissions; its evaluation is ongoing.
The three other medications with the most data are carbamazepine, valproic acid, and gabapentin.45,46 However, the studies were small and the benefit was modest. Although these agents are possible alternatives in protracted alcohol withdrawal syndrome, no definite conclusion can be made regarding their place in therapy.46
RECOMMENDATIONS FOR DRUG THERAPY AND SUPPORTIVE CARE
Which benzodiazepine to use?
No specific benzodiazepine is recommended over the others for managing alcohol withdrawal syndrome, but studies best support the long-acting agent chlordiazepoxide.16,17,20 Other benzodiazepines such as lorazepam and oxazepam have proved to be beneficial, but drugs should be selected on the basis of patient characteristics and drug metabolism.18,19,27
Patients with severe liver dysfunction and the elderly may have slower oxidative metabolism, so the effects of medications that are primarily oxidized, such as chlordiazepoxide and diazepam, may be prolonged. Therefore, lorazepam and oxazepam would be preferred in these groups.47 While most patients with alcohol withdrawal syndrome and liver dysfunction do not have advanced cirrhosis, we recommend liver function testing (serum aspartate aminotransferase, alanine aminotransferase, and alkaline phosphatase levels) and screening for liver disease, given the drug metabolism and package insert caution for use in those with impaired hepatic function.48
Patients with end-stage renal disease (stage 5 chronic kidney disease) or acute kidney injury should not receive parenteral diazepam or lorazepam. The rationale is the potential accumulation of propylene glycol, the solvent used in these formulations.
In the elderly, the Beers list of drugs to avoid in older adults includes benzodiazepines, not differentiating individual benzodiazepines in terms of risk.49 However, chlordiazepoxide may be preferable to diazepam due to its shorter parent half-life and lower lipophilicity.27 Few studies have been done using benzodiazepines in elderly patients with alcohol withdrawal syndrome, but those published have shown either equivalent dosing required compared with younger patients or more severe withdrawal for which they received greater amounts of chlordiazepoxide.9,12 Lorazepam and oxazepam have less potential to accumulate in the elderly compared with the nonelderly due to the drugs’ metabolic profiles; lorazepam is the preferred agent because of its faster onset of action.47 Ultimately, the choice of benzodiazepine in elderly patients with alcohol withdrawal syndrome should be based on patient-specific characteristics.
How should benzodiazepines be dosed?
While the CIWA-Ar thresholds and subsequent dosing of benzodiazepines varied in different studies, we recommend starting benzodiazepine therapy at a CIWA-Ar score of 8 or higher, with subsequent dosing based on score reassessment. Starting doses of benzodiazepines should be chlordiazepoxide 25 to 50 mg, lorazepam 1 to 2 mg, or oxazepam 15 mg.16–20
Subsequent doses should be titrated upward, increasing by 1.5 to 2 times the previous dose and monitored at least every 1 to 2 hours after dose adjustments. Once a patient is stable and the CIWA-Ar score is less than 8, monitoring intervals can be extended to every 4 to 8 hours. If the CIWA-Ar score is more than 20, studies suggest the need for patient reevaluation for transfer to the ICU; however, some health systems have a lower threshold for this intervention.7,14,50
Dosing algorithms and CIWA-Ar goals may vary slightly from institution to institution, but it has been shown that symptom-triggered therapy works best when hospitals have a protocol for it and staff are adequately trained to assess patients with alcohol withdrawal syndrome.7,50,51 Suggestions for dose ranges and symptom-triggered therapy are shown in Table 5.
In case of benzodiazepine overdose or potential benzodiazepine-induced delirium, flumazenil could be considered.52
Patients who should not receive symptom-triggered therapy include immediate postoperative patients in whom clinicians cannot properly assess withdrawal symptoms and patients with a history of DTs.51 While controversy exists regarding the use of symptom-triggered therapy in patients with complicated medical comorbidities, there are data to support symptom-triggered therapy in some ICU patients, as it has resulted in less benzodiazepine use and reduced mechanical ventilation.53,54
There are limited data to support phenobarbital in treating resistant alcohol withdrawal syndrome, either alone or concurrently with benzodiazepines, in escalating doses ranging from 65 to 260 mg, with a maximum daily dose of 520 mg.42,55,56
Haloperidol
For patients exhibiting agitation despite benzodiazepine therapy, giving haloperidol adjunctively can be beneficial.
Haloperidol can be used in medical patients as an adjunctive therapy for agitation, but caution is advised because of the potential for a lowering of the seizure threshold, extrapyramidal effects, and risk of QTc prolongation leading to arrhythmias. Patients considered at highest risk for torsades de pointes may have a QTc of 500 msec or greater.57
Patients should also be screened for factors that have been shown to be independent predictors of QTc prolongation (female sex, diagnosis of myocardial infarction, septic shock or left ventricular dysfunction, other QT-prolonging drugs, age > 68, baseline QTc ≥ 450 msec, and hypokalemia).58 If combined predictors have been identified, it is recommended that haloperidol be avoided.
If haloperidol is to be given, a baseline electrocardiogram and electrolyte panel should be obtained, with daily electrocardiograms thereafter, as well as ongoing review of the patient’s medications to minimize drug interactions that could further increase the risk for QTc prolongation.
Suggested haloperidol dosing is 2 to 5 mg intravenously every 0.5 to 2 hours with a maximum dose of 0.5 mg/kg/24 hours.8,33 A maximum of 35 mg of intravenous haloperidol should be used in a 24-hour period to avoid QTc prolongation.57
Antihypertensive therapy
Many patients receive symptomatic relief of autonomic hyperreactivity with benzodiazepines. However, some may require additional antihypertensive therapy for cardiac adrenergic symptoms (hypertension, tachycardia) if symptoms do not resolve by treating other medical problems commonly seen in patients with alcohol withdrawal syndrome, such as dehydration and electrolyte imbalances.7
Published protocols suggest giving clonidine 0.1 mg orally every hour up to three times as needed until systolic blood pressure is less than 140 mm Hg (less than 160 mm Hg if the patient is over age 60) and diastolic pressure is less than 90 mm Hg.51 Once the patient is stabilized, the dosing can be scheduled to a maximum of 2.4 mg daily.59 However, we believe that the use of clonidine should be restricted to patients who have a substantial increase in blood pressure over baseline or are nearing a hypertensive urgency or emergency (pressures > 180/120 mm Hg) and should not be used to treat other general symptoms associated with alcohol withdrawal syndrome.42
In addition, based on limited evidence, we recommend using beta-blockers only in patients with symptomatic tachycardia or as an adjunct in hypertension management.40,41
Therapies to avoid in acutely ill medical patients
Ethanol is not recommended. Instead, intravenous benzodiazepines should be given in patients presenting with severe alcohol withdrawal syndrome.
Antiepileptics, including valproic acid, carbamazepine, and pregabalin, lack benefit in these patients either as monotherapy or as adjunctive therapy and so are not recommended.45,60–62
Magnesium supplementation (in patients with normal serum magnesium levels) should not be given, as no clinical benefit has been shown.63
- American Psychiatric Association. Diagnostic and Statistical Manual of Mental Disorders. 5th ed. Arlington, VA: American Psychiatric Association, 2013:501.
- Bayard M, McIntyre J, Hill KR, Woodside J Jr. Alcohol withdrawal syndrome. Am Fam Physician 2004; 69:1443–1450.
- Kosten TR, O'Connor PG. Management of drug and alcohol withdrawal. N Engl J Med 2003; 348:1786–1795.
- Isbell H, Fraser HF, Wilker A, Bellevile RE, Eisenman AJ. An experimental study of the etiology of rum fits and delirium tremens. Q J Stud Alcohol 1955; 16:1–33.
- Khan A, Levy P, DeHorn S, Miller W, Compton S. Predictors of mortality in patients with delirium tremens. Acad Emerg Med 2008; 15:788–790.
- Monte R, Rabuñal R, Casariego E, López-Agreda H, Mateos A, Pértega S. Analysis of the factors determining survival of alcoholic withdrawal syndrome patients in a general hospital. Alcohol Alcohol 2010; 45:151–158.
- Stanley KM, Amabile CM, Simpson KN, Couillard D, Norcross ED, Worrall CL. Impact of an alcohol withdrawal syndrome practice guideline on surgical patient outcomes. Pharmacotherapy 2003; 23:843–854.
- Brousse G, Arnaud B, Vorspan F, et al. Alteration of glutamate/GABA balance during acute alcohol withdrawal in emergency department: a prospective analysis. Alcohol Alcohol 2012; 47:501–508.
- Liskow BI, Rinck C, Campbell J, DeSouza C. Alcohol withdrawal in the elderly. J Stud Alcohol 1989; 50:414–421.
- Etherington JM. Emergency management of acute alcohol problems. Part 1: uncomplicated withdrawal. Can Fam Physician 1996; 42:2186–2190.
- Letizia M, Reinbolz M. Identifying and managing acute alcohol withdrawal in the elderly. Geriatr Nurs 2005; 26:176–183.
- Brower KJ, Mudd S, Blow FC, Young JP, Hill EM. Severity and treatment of alcohol withdrawal in elderly versus younger patients. Alcohol Clin Exp Res 1994; 18:196–201.
- Williams D, Lewis J, McBride A. A comparison of rating scales for the alcohol-withdrawal syndrome. Alcohol Alcohol 2001; 36:104–108.
- Reoux JP, Oreskovich MR. A comparison of two versions of the Clinical Institute Withdrawal Assessment for Alcohol: the CIWA-Ar and CIWA-AD. Am J Addict 2006; 15:85–93.
- Sullivan JT, Sykora K, Schneiderman J, Naranjo CA, Sellers EM. Assessment of alcohol withdrawal: the revised Clinical Institute Withdrawal Assessment for alcohol scale (CIWA-Ar). Br J Addict 1989; 84:1353–1357.
- Saitz R, Mayo-Smith MF, Roberts MS, Redmond HA, Bernard DR, Calkins DR. Individualized treatment for alcohol withdrawal. A randomized double-blind controlled trial. JAMA 1994; 272:519–523.
- Jaeger TM, Lohr RH, Pankratz VS. Symptom-triggered therapy for alcohol withdrawal syndrome in medical inpatients. Mayo Clin Proc 2001; 76:695–701.
- Daeppen JB, Gache P, Landry U, et al. Symptom-triggered vs fixed-schedule doses of benzodiazepine for alcohol withdrawal: a randomized treatment trial. Arch Intern Med 2002; 162:1117–1121.
- Weaver MF, Hoffman HJ, Johnson RE, Mauck K. Alcohol withdrawal pharmacotherapy for inpatients with medical comorbidity. J Addict Dis 2006; 25:17–24.
- Reoux JP, Miller K. Routine hospital alcohol detoxification practice compared to symptom triggered management with an Objective Withdrawal Scale (CIWA-Ar). Am J Addict 2000; 9:135–144.
- Wetterling T, Kanitz RD, Besters B, et al. A new rating scale for the assessment of the alcohol-withdrawal syndrome (AWS scale). Alcohol Alcohol 1997; 32:753–760.
- Myrick H, Anton RF. Treatment of alcohol withdrawal. Alcohol Health Res World 1998; 22:38–43.
- Mayo-Smith MF, Beecher LH, Fischer TL, et al; Working Group on the Management of Alcohol Withdrawal Delirium, Practice Guidelines Committee, American Society of Addiction Medicine. Management of alcohol withdrawal delirium. An evidence-based practice guideline. Arch Intern Med 2004; 164:1405–1412.
- Sellers EM, Sullivan JT, Somer G, Sykora K. Characterization of DSM-III-R criteria for uncomplicated alcohol withdrawal provides an empirical basis for DSM-IV. Arch Gen Psychiatry 1991; 48:442–447.
- Sarff M, Gold JA. Alcohol withdrawal syndromes in the intensive care unit. Crit Care Med 2010; 38(suppl):S494–S501.
- Kumar CN, Andrade C, Murthy P. A randomized, double-blind comparison of lorazepam and chlordiazepoxide in patients with uncomplicated alcohol withdrawal. J Stud Alcohol Drugs 2009; 70:467–474.
- Bird RD, Makela EH. Alcohol withdrawal: what is the benzodiazepine of choice? Ann Pharmacother 1994; 28:67–71.
- Perry EC. Inpatient management of acute alcohol withdrawal syndrome. CNS Drugs 2014; 28:401–410.
- Amato L, Minozzi S, Vecchi S, Davoli M. Benzodiazepines for alcohol withdrawal. Cochrane Database Syst Rev 2010; 3:CD005063.
- Weinberg JA, Magnotti LJ, Fischer PE, et al. Comparison of intravenous ethanol versus diazepam for alcohol withdrawal prophylaxis in the trauma ICU: results of a randomized trial. J Trauma 2008; 64:99–104.
- Craft PP, Foil MB, Cunningham PR, Patselas PC, Long-Snyder BM, Collier MS. Intravenous ethanol for alcohol detoxification in trauma patients. South Med J 1994; 87:47–54.
- Ungur LA, Neuner B, John S, Wernecke K, Spies C. Prevention and therapy of alcohol withdrawal on intensive care units: systematic review of controlled trials. Alcohol Clin Exp Res 2013; 37:675–686.
- Lansford CD, Guerriero CH, Kocan MJ, et al. Improved outcomes in patients with head and neck cancer using a standardized care protocol for postoperative alcohol withdrawal. Arch Otolaryngol Head Neck Surg 2008; 134:865–872.
- Walinder J, Balldin J, Bokstrom K, Karlsson I, Lundstrom B, Svensson TH. Clonidine suppression of the alcohol withdrawal syndrome. Drug Alcohol Depend 1981; 8:345–348.
- Muzyk AJ, Fowler JA, Norwood DK, Chilipko A. Role of alpha2-agonists in the treatment of acute alcohol withdrawal. Ann Pharmacother 2011; 45:649–657.
- Crispo AL, Daley MJ, Pepin JL, Harford PH, Brown CV. Comparison of clinical outcomes in nonintubated patients with severe alcohol withdrawal syndrome treated with continuous-infusion sedatives: dexmedetomidine versus benzodiazepines. Pharmacotherapy 2014; 34:910–917.
- VanderWeide LA, Foster CJ, MacLaren R, Kiser TH, Fish DN, Mueller SW. Evaluation of early dexmedetomidine addition to the standard of care for severe alcohol withdrawal in the ICU: a retrospective controlled cohort study. J Intensive Care Med 2014. [Epub ahead of print October 16, 2014]
- Rayner SG, Weinert CR, Peng H, Jepsen S, Broccard AF. Dexmedetomidine as adjunct treatment for severe alcohol withdrawal in the ICU. Ann Intensive Care 2012; 2:12.
- Muzyk AJ, Kerns S, Brudney S, Gagliardi JP. Dexmedetomidine for the treatment of alcohol withdrawal syndrome: rationale and current status of research. CNS Drugs 2013; 27:913–920.
- Kraus ML, Gottlieb LD, Horwitz RI, Anscher M. Randomized clinical trial of atenolol in patients with alcohol withdrawal. N Engl J Med 1985; 313:905–909.
- Zilm DH, Jacob MS, MacLeod SM, Sellers EM, Ti TY. Propranolol and chlordiazepoxide effects on cardiac arrhythmias during alcohol withdrawal. Alcohol Clin Exp Res 1980; 4:400–405.
- Hack JB, Hoffmann RS, Nelson LS. Resistant alcohol withdrawal: does an unexpectedly large sedative requirement identify these patients early? J Med Toxicol 2006; 2:55–60.
- Hayner CE, Wuestefeld NL, Bolton PJ. Phenobarbital treatment in a patient with resistant alcohol withdrawal syndrome. Pharmacotherapy 2009; 29:875–878.
- Rosenson J, Clements C, Simon B, et al. Phenobarbital for acute alcohol withdrawal: a prospective randomized double-blind placebo-controlled study. J Emerg Med 2013; 44:592–598.e2.
- Prince V, Turpin KR. Treatment of alcohol withdrawal syndrome with carbamazepine, gabapentin, and nitrous oxide. Am J Health Syst Pharm 2008; 65:1039–1047.
- Leggio L, Kenna GA, Swift RM. New developments for the pharmacological treatment of alcohol withdrawal syndrome. A focus on non-benzodiazepine GABAergic medications. Prog Neuropsychopharmacol Biol Psychiatry 2008; 32:1106–1117.
- Peppers MP. Benzodiazepines for alcohol withdrawal in the elderly and in patients with liver disease. Pharmacotherapy 1996; 16:49–57.
- Valeant Pharmaceuticals North America LLC. Librium—chlordiazepoxide hydrochloride capsule, gelatin coated. http://dailymed.nlm.nih.gov/dailymed/archives/fdaDrugInfo.cfm?archiveid=125207. Accessed November 20, 2015.
- American Geriatrics Society 2012 Beers Criteria Update Expert Panel. American Geriatrics Society updated Beers Criteria for potentially inappropriate medication use in older adults. J Am Geriatr Soc 2012; 60:616–631.
- Hecksel KA, Bostwick JM, Jaeger TM, Cha SS. Inappropriate use of symptom-triggered therapy for alcohol withdrawal in the general hospital. Mayo Clin Proc 2008; 83:274–279.
- Manasco A, Chang S, Larriviere J, Hamm LL, Glass M. Alcohol withdrawal. South Med J 2012; 105:607–612.
- Moore PW, Donovan JW, Burkhart KK, et al. Safety and efficacy of flumazenil for reversal of iatrogenic benzodiazepine-associated delirium toxicity during treatment of alcohol withdrawal, a retrospective review at one center. J Med Toxicol 2014; 10:126–132.
- Bostwick JM, Lapid MI. False positives on the clinical institute withdrawal assessment for alcohol-revised: is this scale appropriate for use in the medically ill? Psychosomatics 2004; 45:256–261.
- de Wit M, Jones DG, Sessler CN, Zilberberg MD, Weaver MF. Alcohol-use disorders in the critically ill patient. Chest 2010; 138:994–1003.
- Young GP, Rores C, Murphy C, Dailey RH. Intravenous phenobarbital for alcohol withdrawal and convulsions. Ann Emerg Med 1987; 16:847–850.
- Hendey GW, Dery RA, Barnes RL, Snowden B, Mentler P. A prospective, randomized trial of phenobarbital versus benzodiazepines for acute alcohol withdrawal. Am J Emerg Med 2011; 29:382–385.
- Sharma ND, Rosman HS, Padhi ID, Tisdale JE. Torsades de pointes associated with intravenous haloperidol in critically ill patients. Am J Cardiol 1998; 81:238–240.
- Tisdale JE, Jaynes HA, Kingery JR, et al. Development and validation of a risk score to predict QT interval prolongation in hospitalized patients. Circ Cardiovasc Qual Outcomes 2013; 6:479–487.
- Boehringer Ingelheim Pharmaceuticals, Inc. Product Information: Catapres oral tablets, clonidine HCl oral tablets, 2012.
- Reoux JP, Saxon AJ, Malte CA, Baer JS, Sloan KL. Divalproex sodium in alcohol withdrawal: a randomized double-blind placebo-controlled clinical trial. Alcohol Clin Exp Res 2001; 25:1324–1329.
- Malcolm R, Ballenger JC, Sturgis ET, Anton R. Double-blind controlled trial comparing carbamazepine to oxazepam treatment of alcohol withdrawal. Am J Psychiatry 1989; 146:617–621.
- Förg A, Hein J, Volkmar K, et al. Efficacy and safety of pregabalin in the treatment of alcohol withdrawal syndrome: a randomized placebo-controlled trial. Alcohol Alcohol 2012; 47:149–155.
- Wilson A, Vulcano B. A double-blind, placebo-controlled trial of magnesium sulfate in the ethanol withdrawal syndrome. Alcohol Clin Exp Res 1984; 8:542–545.
Deprived of alcohol while in the hospital, up to 80% of patients who are alcohol-dependent risk developing alcohol withdrawal syndrome,1 a potentially life-threatening condition. Clinicians should anticipate the syndrome and be ready to treat and prevent its complications.
Because alcoholism is common, nearly every provider will encounter its complications and withdrawal symptoms. Each year, an estimated 1.2 million hospital admissions are related to alcohol abuse, and about 500,000 episodes of withdrawal symptoms are severe enough to require clinical attention.1–3 Nearly 50% of patients with alcohol withdrawal syndrome are middle-class, highly functional individuals, making withdrawal difficult to recognize.1
While acute trauma patients or those with alcohol withdrawal delirium are often admitted directly to an intensive care unit (ICU), many others are at risk for or develop alcohol withdrawal syndrome and are managed initially or wholly on the acute medical unit. While specific statistics have not been published on non-ICU patients with alcohol withdrawal syndrome, they are an important group of patients who need to be well managed to prevent the progression of alcohol withdrawal syndrome to alcohol withdrawal delirium, alcohol withdrawal-induced seizure, and other complications.
This article reviews how to identify and manage alcohol withdrawal symptoms in noncritical, acutely ill medical patients, with practical recommendations for diagnosis and management.
CAN LEAD TO DELIRIUM TREMENS
In people who are physiologically dependent on alcohol, symptoms of withdrawal usually occur after abrupt cessation.4 If not addressed early in the hospitalization, alcohol withdrawal syndrome can progress to alcohol withdrawal delirium (also known as delirium tremens or DTs), in which the mortality rate is 5% to 10%.5,6 Potential mechanisms of DTs include increased dopamine release and dopamine receptor activity, hypersensitivity to N-methyl-d-aspartate, and reduced levels of gamma-aminobutyric acid (GABA).7
Long-term changes are thought to occur in neurons after repeated detoxification from alcohol, a phenomenon called “kindling.” After each detoxification, alcohol craving and obsessive thoughts increase,8 and subsequent episodes of alcohol withdrawal tend to be progressively worse.
Withdrawal symptoms
Alcohol withdrawal syndrome encompasses a spectrum of symptoms and conditions, from minor (eg, insomnia, tremulousness) to severe (seizures, DTs).2 The symptoms typically depend on the amount of alcohol consumed, the time since the last drink, and the number of previous detoxifications.9
The Diagnostic and Statistical Manual of Mental Disorders, Fifth Edition,1 states that to establish a diagnosis of alcohol withdrawal syndrome, a patient must meet four criteria:
- The patient must have ceased or reduced alcohol intake after heavy or prolonged use.
- Two or more of the following must develop within a few hours to a few days: autonomic hyperactivity (sweating or pulse greater than 100 beats per minute); increased hand tremor; insomnia; nausea or vomiting; transient visual, tactile, or auditory hallucinations or illusions; psychomotor agitation; anxiety; grand mal seizure.
- The above symptoms must cause significant distress or functional impairment.
- The symptoms must not be related to another medical condition.
Some of the symptoms described in the second criterion above can occur while the patient still has a measurable blood alcohol level, usually within 6 hours of cessation of drinking.10 Table 1 describes the timetable of onset of symptoms and their severity.2
The elderly may be affected more severely
While the progression of the symptoms described above is commonly used for medical inpatients, the timeline may be different in an elderly patient. Compared with younger patients, elderly patients may have higher blood alcohol concentrations owing to lower total body water, so small amounts of alcohol can produce significant effects.11,12 Brower et al12 found that elderly patients experienced more withdrawal symptoms, especially cognitive impairment, weakness, and high blood pressure, and for 3 days longer.
In the elderly, alcohol may have a greater impact on the central nervous system because of increased permeability of the blood-brain barrier. And importantly, elderly patients tend to have more concomitant diseases and take more medications, all of which can affect alcohol metabolism.
ASSESSMENT SCALES FOR ALCOHOL WITHDRAWAL SYNDROME
A number of clinical scales for evaluating alcohol withdrawal have been developed. Early ones such as the 30-item Total Severity Assessment (TSA) scale and the 11-item Selected Severity Assessment (SSA) scale were limited because they were extremely detailed, burdensome to nursing staff to administer, and contained items such as daily “sleep disturbances” that were not acute enough to meet specific monitoring needs or to guide drug therapy.13,14
The Clinical Institute Withdrawal Assessment for alcohol (CIWA-A) scale, with 15 items, was derived from the SSA scale and includes acute items for assessment as often as every half-hour.15
The CIWA-Ar scale (Table 2) was developed from the CIWA-A scale by Sullivan et al.15 Using both observation and interview, it focuses on 10 areas: nausea and vomiting, tremor, paroxysmal sweats, anxiety, agitation, headache, disorientation, tactile disturbances, auditory disturbances, and visual disturbances. Scores can range from 0 to 67; a higher score indicates worse withdrawal symptoms and outcomes and therefore necessitates escalation of treatment.
The CIWA-Ar scale is now the one most commonly used in clinical trials16–20 and, we believe, in practice. Other scales, including the CIWA-AD and the Alcohol Withdrawal Scale have been validated but are not widely used in practice.14,21
BASELINE ASSESSMENT AND EARLY SUPPORTIVE CARE
A thorough history and physical examination should be performed on admission in patients known to be or suspected of being alcohol-dependent to assess the patient’s affected body systems. The time elapsed since the patient’s last alcohol drink helps predict the onset of withdrawal complications.
Baseline laboratory tests for most patients with suspected alcohol withdrawal syndrome should include a basic blood chemistry panel, complete blood cell count, and possibly an alcohol and toxicology screen, depending on the patient’s history and presentation.
Hydration and nutritional support are important in patients presenting with alcohol withdrawal syndrome. Severe disturbances in electrolytes can lead to serious complications, including cardiac arrhythmia. Close monitoring and electrolyte replacement as needed are recommended for hospitalized alcoholic patients and should follow hospital protocols.22
Thiamine and folic acid status deserve special attention, since long-standing malnutrition is common in alcoholic patients. Thiamine deficiency can result in Wernicke encephalopathy and Korsakoff syndrome, characterized by delirium, ataxia, vision changes, and amnesia. Alcohol withdrawal guidelines recommend giving thiamine intravenously for the first 2 to 5 days after admission.23 In addition, thiamine must be given before any intravenous glucose product, as thiamine is a cofactor in carbohydrate metabolism.23 Folic acid should also be supplemented, as chronic deficiencies may lead to megaloblastic or macrocytic anemia.
CIWA-Ar scale. To provide consistent monitoring and ongoing treatment, clinicians and institutions are encouraged to use a simple assessment scale that detects and quantifies alcohol withdrawal syndrome and that can be used for reassessment after an intervention.21 The CIWA-Ar scale should be used to facilitate “symptom-triggered therapy” in which, depending on the score, the patient receives pharmacologic treatment followed by a scheduled reevaluation.23,24 Most patients with a CIWA-Ar score of 8 or higher benefit from benzodiazepine therapy.16,18,19
PRIMARY DRUG THERAPIES FOR MEDICAL INPATIENTS
Benzodiazepines are the first-line agents
Benzodiazepines are the first-line agents recommended for preventing and treating alcohol withdrawal syndrome.23 Their various pharmacokinetic profiles, wide therapeutic indices, and safety compared with older sedative hypnotics make them the preferred class.23,25 No single benzodiazepine is preferred over the others for treating alcohol withdrawal syndrome: studies have shown benefits using short-acting, intermediate-acting, and long-acting agents. The choice of drug is variable and patient-specific.16,18,26
Benzodiazepines promote and enhance binding of the inhibitory neurotransmitter GABA to GABAA receptors in the central nervous system.27 As a class, benzodiazepines are all structurally related and produce the same effects—namely, sedation, hypnosis, decreased anxiety, muscle relaxation, anterograde amnesia, and anticonvulsant activity.27
The most studied benzodiazepines for treating and preventing alcohol withdrawal syndrome are chlordiazepoxide, oxazepam, and lorazepam,16–20 whereas diazepam was used in older studies.23
Diazepam and chlordiazepoxide are metabolized by oxidation, each sharing the long-acting active metabolite desmethyldiazepam (half-life 72 hours), and short-acting metabolite oxazepam (half-life 8 hours).27 In addition, the parent drugs also have varying pharmacokinetic profiles: diazepam has a half-life of more than 30 hours and chlordiazepoxide a half-life of about 8 hours. Chlordiazepoxide and diazepam’s combination of both long- and short-acting benzodiazepine activity provides long-term efficacy in attenuating withdrawal symptoms, but chlordiazepoxide’s shorter parent half-life allows more frequent dosing.
Lorazepam (half-life 10–20 hours) and oxazepam (half-life 5–20 hours) undergo glucuronide conjugation and do not have metabolites.27,28 Table 3 provides a pharmacokinetic summary.27,28
Various dosage regimens are used in giving benzodiazepines, the most common being symptom-triggered therapy, governed by assessment scales, and scheduled around-the-clock therapy.29 Current evidence supports symptom-triggered therapy in most inpatients who are not critically ill, as it can reduce both benzodiazepine use and adverse drug events and can reduce the length of stay.16,19
Trials of symptom-triggered benzodiazepine therapy
Most inpatient trials of symptom-triggered therapy (Table 4)3,16–20 used the CIWA-Ar scale for monitoring. In some of the studies, benzodiazepines were given if the score was 8 or higher, but others used cut points as high as 15 or higher. Doses:
- Chlordiazepoxide (first dose 25–100 mg)
- Lorazepam (first dose 0.5–2 mg)
- Oxazepam (30 mg).
After each dose, patients were reevaluated at intervals of 30 minutes to 8 hours.
Most of these trials showed no difference in rates of adverse drug events such as seizures, hallucinations, and lethargy with symptom-triggered therapy compared with scheduled therapy.16,18,20 They also found either no difference in the incidence of delirium tremens, or a lower incidence of delirium tremens with symptom-triggered therapy than with scheduled therapy.16–18,20
Weaver et al19 found no difference in length of stay between scheduled therapy and symptom-triggered therapy, but Saitz et al16 reported a median benzodiazepine treatment duration of 9 hours with symptom-triggered therapy vs 68 hours with fixed dosing. Thus, the study by Saitz et al suggests that hospitalization might be shorter with symptom-triggered therapy.
Many of the trials had notable limitations related to the diversity of patients enrolled and the protocols for both symptom-triggered therapy and fixed dosing. Some trials enrolled only inpatients in detoxification programs; others focused on inpatients with acute medical illness. The inpatient alcohol treatment trials16,18 excluded medically ill patients and those with concurrent psychiatric illness,16,18 and one excluded patients with seizures.16 One of the inpatient alcohol treatment program trials16 excluded patients on beta-blockers or clonidine because of concern that these drugs could mask withdrawal symptoms, whereas trials in medically ill patients allowed these same drugs.17,19,20
Most of the patients were men (approximately 75%, but ranging from 74% to 100%), and therefore the study results may not be as applicable to women.16–20 Most participants were middle-aged, with average ages in all studies between 46 and 55. Finally, the studies used a wide range of medications and dosing, with patient monitoring intervals ranging from every 30 minutes to every 8 hours.16–20
In a 2010 Cochrane analysis, Amato et al29 concluded that the limited evidence available favors symptom-triggered regimens over fixed-dosing regimens, but that differences in isolated trials should be interpreted very cautiously.
Therapeutic ethanol
Aside from the lack of evidence to support its use in alcohol withdrawal syndrome, prescribing oral ethanol to alcoholic patients clearly poses an ethical dilemma. However, giving ethanol intravenously has been studied, mostly in surgical trauma patients.30
Early reports described giving intravenous ethanol on a gram-to-gram basis to match the patient’s consumption before admission to prevent alcohol withdrawal syndrome. But later studies reported prevention of alcohol withdrawal syndrome with very small amounts of intravenous ethanol.30,31 While clinical trials have been limited to ICU patients, ethanol infusion at an initial rate of 2.5 to 5 g per hour and titrated up to 10 g per hour has appeared to be safe and effective for preventing alcohol withdrawal syndrome.30,31 The initial infusion rate of 2.5 to 5 g per hour is equivalent to 4 to 10 alcoholic beverages per 24 hours.
Nevertheless, ethanol infusion carries the potential for toxicities (eg, gastric irritation, precipitation of acute hepatic failure, hypoglycemia, pancreatitis, bone marrow suppression, prolonged wound healing) and drug interactions (eg, with anticoagulants and anticonvulsants). Thus, ethanol is neither widely used nor recommended.25,31
ADJUNCTIVE THERAPIES
Many medications are used adjunctively in the acute setting, both for symptoms of alcohol withdrawal syndrome and for agitation.
Haloperidol
No clinical trial has yet examined haloperidol monotherapy in patients with alcohol withdrawal syndrome in either general medical units or intensive care units. Yet haloperidol remains important and is recommended as an adjunct therapy for agitation.23,32 Dosing of haloperidol in protocols for surgical patients ranged from 2 to 5 mg intravenously every 0.5 to 2 hours, with a maximum dosage of 0.5 mg per kg per 24 hours.7,33
Alpha-2 agonists
Alpha-2 agonists are thought to reduce sympathetic overdrive and the autonomic symptoms associated with alcohol withdrawal syndrome, and these agents (primarily clonidine) have been studied in the treatment of alcohol withdrawal syndrome.34,35
Clonidine. In a Swedish study,34 26 men ages 20 to 55 who presented with the tremor, sweating, dysphoria, tension, anxiety, and tachycardia associated with alcohol withdrawal syndrome received clonidine 4 µg per kg twice daily or carbamazepine 200 mg three to four times daily in addition to an antiepileptic. Adjunctive use of a benzodiazepine was allowed at night in both groups. No statistically significant difference in symptom reduction was noted between the two groups, and there was no difference in total benzodiazepine use.
Dexmedetomidine, given intravenously, has been tested as an adjunct to benzodiazepine treatment in severe alcohol withdrawal syndrome. It has been shown to decrease the amount of total benzodiazepine needed compared with benzodiazepine therapy alone, but no differences have been seen in length of hospital stay.36–39 However, research on this drug so far is limited to ICU patients.
Beta-blockers
Beta-blockers have been used in inpatients with alcohol withdrawal syndrome to reduce heart rate and potentially reduce alcohol craving. However, the data are limited and conflicting.
Atenolol 50 to 100 mg daily, in a study in 120 patients, reduced length of stay (4 vs 5 days), reduced benzodiazepine use, and improved vital signs and behavior compared with placebo.40
Propranolol 40 mg every 6 hours reduced arrhythmias but increased hallucinations when used alone in a study in 47 patients.41 When used in combination with chlordiazepoxide, no benefit was seen in arrhythmia reduction.41
Barbiturates and other antiepileptics
Data continue to emerge on antiepileptics as both monotherapy and adjunctive therapy for alcohol withdrawal syndrome. Barbiturates as monotherapy were largely replaced by benzodiazepines in view of the narrow therapeutic index of barbiturates and their full agonist effect on the GABA receptor complex. However, phenobarbital has been evaluated in patients presenting with severe alcohol withdrawal syndrome or resistant alcohol withdrawal (ie, symptoms despite large or repeated doses of benzodiazepines) as an adjunct to benzodiazepines.42,43
In addition, a newer trial44 involved giving a single dose of phenobarbital in the emergency department in combination with a CIWA-Ar–based benzodiazepine protocol, compared with the benzodiazepine protocol alone. The group that received phenobarbital had fewer ICU admissions; its evaluation is ongoing.
The three other medications with the most data are carbamazepine, valproic acid, and gabapentin.45,46 However, the studies were small and the benefit was modest. Although these agents are possible alternatives in protracted alcohol withdrawal syndrome, no definite conclusion can be made regarding their place in therapy.46
RECOMMENDATIONS FOR DRUG THERAPY AND SUPPORTIVE CARE
Which benzodiazepine to use?
No specific benzodiazepine is recommended over the others for managing alcohol withdrawal syndrome, but studies best support the long-acting agent chlordiazepoxide.16,17,20 Other benzodiazepines such as lorazepam and oxazepam have proved to be beneficial, but drugs should be selected on the basis of patient characteristics and drug metabolism.18,19,27
Patients with severe liver dysfunction and the elderly may have slower oxidative metabolism, so the effects of medications that are primarily oxidized, such as chlordiazepoxide and diazepam, may be prolonged. Therefore, lorazepam and oxazepam would be preferred in these groups.47 While most patients with alcohol withdrawal syndrome and liver dysfunction do not have advanced cirrhosis, we recommend liver function testing (serum aspartate aminotransferase, alanine aminotransferase, and alkaline phosphatase levels) and screening for liver disease, given the drug metabolism and package insert caution for use in those with impaired hepatic function.48
Patients with end-stage renal disease (stage 5 chronic kidney disease) or acute kidney injury should not receive parenteral diazepam or lorazepam. The rationale is the potential accumulation of propylene glycol, the solvent used in these formulations.
In the elderly, the Beers list of drugs to avoid in older adults includes benzodiazepines, not differentiating individual benzodiazepines in terms of risk.49 However, chlordiazepoxide may be preferable to diazepam due to its shorter parent half-life and lower lipophilicity.27 Few studies have been done using benzodiazepines in elderly patients with alcohol withdrawal syndrome, but those published have shown either equivalent dosing required compared with younger patients or more severe withdrawal for which they received greater amounts of chlordiazepoxide.9,12 Lorazepam and oxazepam have less potential to accumulate in the elderly compared with the nonelderly due to the drugs’ metabolic profiles; lorazepam is the preferred agent because of its faster onset of action.47 Ultimately, the choice of benzodiazepine in elderly patients with alcohol withdrawal syndrome should be based on patient-specific characteristics.
How should benzodiazepines be dosed?
While the CIWA-Ar thresholds and subsequent dosing of benzodiazepines varied in different studies, we recommend starting benzodiazepine therapy at a CIWA-Ar score of 8 or higher, with subsequent dosing based on score reassessment. Starting doses of benzodiazepines should be chlordiazepoxide 25 to 50 mg, lorazepam 1 to 2 mg, or oxazepam 15 mg.16–20
Subsequent doses should be titrated upward, increasing by 1.5 to 2 times the previous dose and monitored at least every 1 to 2 hours after dose adjustments. Once a patient is stable and the CIWA-Ar score is less than 8, monitoring intervals can be extended to every 4 to 8 hours. If the CIWA-Ar score is more than 20, studies suggest the need for patient reevaluation for transfer to the ICU; however, some health systems have a lower threshold for this intervention.7,14,50
Dosing algorithms and CIWA-Ar goals may vary slightly from institution to institution, but it has been shown that symptom-triggered therapy works best when hospitals have a protocol for it and staff are adequately trained to assess patients with alcohol withdrawal syndrome.7,50,51 Suggestions for dose ranges and symptom-triggered therapy are shown in Table 5.
In case of benzodiazepine overdose or potential benzodiazepine-induced delirium, flumazenil could be considered.52
Patients who should not receive symptom-triggered therapy include immediate postoperative patients in whom clinicians cannot properly assess withdrawal symptoms and patients with a history of DTs.51 While controversy exists regarding the use of symptom-triggered therapy in patients with complicated medical comorbidities, there are data to support symptom-triggered therapy in some ICU patients, as it has resulted in less benzodiazepine use and reduced mechanical ventilation.53,54
There are limited data to support phenobarbital in treating resistant alcohol withdrawal syndrome, either alone or concurrently with benzodiazepines, in escalating doses ranging from 65 to 260 mg, with a maximum daily dose of 520 mg.42,55,56
Haloperidol
For patients exhibiting agitation despite benzodiazepine therapy, giving haloperidol adjunctively can be beneficial.
Haloperidol can be used in medical patients as an adjunctive therapy for agitation, but caution is advised because of the potential for a lowering of the seizure threshold, extrapyramidal effects, and risk of QTc prolongation leading to arrhythmias. Patients considered at highest risk for torsades de pointes may have a QTc of 500 msec or greater.57
Patients should also be screened for factors that have been shown to be independent predictors of QTc prolongation (female sex, diagnosis of myocardial infarction, septic shock or left ventricular dysfunction, other QT-prolonging drugs, age > 68, baseline QTc ≥ 450 msec, and hypokalemia).58 If combined predictors have been identified, it is recommended that haloperidol be avoided.
If haloperidol is to be given, a baseline electrocardiogram and electrolyte panel should be obtained, with daily electrocardiograms thereafter, as well as ongoing review of the patient’s medications to minimize drug interactions that could further increase the risk for QTc prolongation.
Suggested haloperidol dosing is 2 to 5 mg intravenously every 0.5 to 2 hours with a maximum dose of 0.5 mg/kg/24 hours.8,33 A maximum of 35 mg of intravenous haloperidol should be used in a 24-hour period to avoid QTc prolongation.57
Antihypertensive therapy
Many patients receive symptomatic relief of autonomic hyperreactivity with benzodiazepines. However, some may require additional antihypertensive therapy for cardiac adrenergic symptoms (hypertension, tachycardia) if symptoms do not resolve by treating other medical problems commonly seen in patients with alcohol withdrawal syndrome, such as dehydration and electrolyte imbalances.7
Published protocols suggest giving clonidine 0.1 mg orally every hour up to three times as needed until systolic blood pressure is less than 140 mm Hg (less than 160 mm Hg if the patient is over age 60) and diastolic pressure is less than 90 mm Hg.51 Once the patient is stabilized, the dosing can be scheduled to a maximum of 2.4 mg daily.59 However, we believe that the use of clonidine should be restricted to patients who have a substantial increase in blood pressure over baseline or are nearing a hypertensive urgency or emergency (pressures > 180/120 mm Hg) and should not be used to treat other general symptoms associated with alcohol withdrawal syndrome.42
In addition, based on limited evidence, we recommend using beta-blockers only in patients with symptomatic tachycardia or as an adjunct in hypertension management.40,41
Therapies to avoid in acutely ill medical patients
Ethanol is not recommended. Instead, intravenous benzodiazepines should be given in patients presenting with severe alcohol withdrawal syndrome.
Antiepileptics, including valproic acid, carbamazepine, and pregabalin, lack benefit in these patients either as monotherapy or as adjunctive therapy and so are not recommended.45,60–62
Magnesium supplementation (in patients with normal serum magnesium levels) should not be given, as no clinical benefit has been shown.63
Deprived of alcohol while in the hospital, up to 80% of patients who are alcohol-dependent risk developing alcohol withdrawal syndrome,1 a potentially life-threatening condition. Clinicians should anticipate the syndrome and be ready to treat and prevent its complications.
Because alcoholism is common, nearly every provider will encounter its complications and withdrawal symptoms. Each year, an estimated 1.2 million hospital admissions are related to alcohol abuse, and about 500,000 episodes of withdrawal symptoms are severe enough to require clinical attention.1–3 Nearly 50% of patients with alcohol withdrawal syndrome are middle-class, highly functional individuals, making withdrawal difficult to recognize.1
While acute trauma patients or those with alcohol withdrawal delirium are often admitted directly to an intensive care unit (ICU), many others are at risk for or develop alcohol withdrawal syndrome and are managed initially or wholly on the acute medical unit. While specific statistics have not been published on non-ICU patients with alcohol withdrawal syndrome, they are an important group of patients who need to be well managed to prevent the progression of alcohol withdrawal syndrome to alcohol withdrawal delirium, alcohol withdrawal-induced seizure, and other complications.
This article reviews how to identify and manage alcohol withdrawal symptoms in noncritical, acutely ill medical patients, with practical recommendations for diagnosis and management.
CAN LEAD TO DELIRIUM TREMENS
In people who are physiologically dependent on alcohol, symptoms of withdrawal usually occur after abrupt cessation.4 If not addressed early in the hospitalization, alcohol withdrawal syndrome can progress to alcohol withdrawal delirium (also known as delirium tremens or DTs), in which the mortality rate is 5% to 10%.5,6 Potential mechanisms of DTs include increased dopamine release and dopamine receptor activity, hypersensitivity to N-methyl-d-aspartate, and reduced levels of gamma-aminobutyric acid (GABA).7
Long-term changes are thought to occur in neurons after repeated detoxification from alcohol, a phenomenon called “kindling.” After each detoxification, alcohol craving and obsessive thoughts increase,8 and subsequent episodes of alcohol withdrawal tend to be progressively worse.
Withdrawal symptoms
Alcohol withdrawal syndrome encompasses a spectrum of symptoms and conditions, from minor (eg, insomnia, tremulousness) to severe (seizures, DTs).2 The symptoms typically depend on the amount of alcohol consumed, the time since the last drink, and the number of previous detoxifications.9
The Diagnostic and Statistical Manual of Mental Disorders, Fifth Edition,1 states that to establish a diagnosis of alcohol withdrawal syndrome, a patient must meet four criteria:
- The patient must have ceased or reduced alcohol intake after heavy or prolonged use.
- Two or more of the following must develop within a few hours to a few days: autonomic hyperactivity (sweating or pulse greater than 100 beats per minute); increased hand tremor; insomnia; nausea or vomiting; transient visual, tactile, or auditory hallucinations or illusions; psychomotor agitation; anxiety; grand mal seizure.
- The above symptoms must cause significant distress or functional impairment.
- The symptoms must not be related to another medical condition.
Some of the symptoms described in the second criterion above can occur while the patient still has a measurable blood alcohol level, usually within 6 hours of cessation of drinking.10 Table 1 describes the timetable of onset of symptoms and their severity.2
The elderly may be affected more severely
While the progression of the symptoms described above is commonly used for medical inpatients, the timeline may be different in an elderly patient. Compared with younger patients, elderly patients may have higher blood alcohol concentrations owing to lower total body water, so small amounts of alcohol can produce significant effects.11,12 Brower et al12 found that elderly patients experienced more withdrawal symptoms, especially cognitive impairment, weakness, and high blood pressure, and for 3 days longer.
In the elderly, alcohol may have a greater impact on the central nervous system because of increased permeability of the blood-brain barrier. And importantly, elderly patients tend to have more concomitant diseases and take more medications, all of which can affect alcohol metabolism.
ASSESSMENT SCALES FOR ALCOHOL WITHDRAWAL SYNDROME
A number of clinical scales for evaluating alcohol withdrawal have been developed. Early ones such as the 30-item Total Severity Assessment (TSA) scale and the 11-item Selected Severity Assessment (SSA) scale were limited because they were extremely detailed, burdensome to nursing staff to administer, and contained items such as daily “sleep disturbances” that were not acute enough to meet specific monitoring needs or to guide drug therapy.13,14
The Clinical Institute Withdrawal Assessment for alcohol (CIWA-A) scale, with 15 items, was derived from the SSA scale and includes acute items for assessment as often as every half-hour.15
The CIWA-Ar scale (Table 2) was developed from the CIWA-A scale by Sullivan et al.15 Using both observation and interview, it focuses on 10 areas: nausea and vomiting, tremor, paroxysmal sweats, anxiety, agitation, headache, disorientation, tactile disturbances, auditory disturbances, and visual disturbances. Scores can range from 0 to 67; a higher score indicates worse withdrawal symptoms and outcomes and therefore necessitates escalation of treatment.
The CIWA-Ar scale is now the one most commonly used in clinical trials16–20 and, we believe, in practice. Other scales, including the CIWA-AD and the Alcohol Withdrawal Scale have been validated but are not widely used in practice.14,21
BASELINE ASSESSMENT AND EARLY SUPPORTIVE CARE
A thorough history and physical examination should be performed on admission in patients known to be or suspected of being alcohol-dependent to assess the patient’s affected body systems. The time elapsed since the patient’s last alcohol drink helps predict the onset of withdrawal complications.
Baseline laboratory tests for most patients with suspected alcohol withdrawal syndrome should include a basic blood chemistry panel, complete blood cell count, and possibly an alcohol and toxicology screen, depending on the patient’s history and presentation.
Hydration and nutritional support are important in patients presenting with alcohol withdrawal syndrome. Severe disturbances in electrolytes can lead to serious complications, including cardiac arrhythmia. Close monitoring and electrolyte replacement as needed are recommended for hospitalized alcoholic patients and should follow hospital protocols.22
Thiamine and folic acid status deserve special attention, since long-standing malnutrition is common in alcoholic patients. Thiamine deficiency can result in Wernicke encephalopathy and Korsakoff syndrome, characterized by delirium, ataxia, vision changes, and amnesia. Alcohol withdrawal guidelines recommend giving thiamine intravenously for the first 2 to 5 days after admission.23 In addition, thiamine must be given before any intravenous glucose product, as thiamine is a cofactor in carbohydrate metabolism.23 Folic acid should also be supplemented, as chronic deficiencies may lead to megaloblastic or macrocytic anemia.
CIWA-Ar scale. To provide consistent monitoring and ongoing treatment, clinicians and institutions are encouraged to use a simple assessment scale that detects and quantifies alcohol withdrawal syndrome and that can be used for reassessment after an intervention.21 The CIWA-Ar scale should be used to facilitate “symptom-triggered therapy” in which, depending on the score, the patient receives pharmacologic treatment followed by a scheduled reevaluation.23,24 Most patients with a CIWA-Ar score of 8 or higher benefit from benzodiazepine therapy.16,18,19
PRIMARY DRUG THERAPIES FOR MEDICAL INPATIENTS
Benzodiazepines are the first-line agents
Benzodiazepines are the first-line agents recommended for preventing and treating alcohol withdrawal syndrome.23 Their various pharmacokinetic profiles, wide therapeutic indices, and safety compared with older sedative hypnotics make them the preferred class.23,25 No single benzodiazepine is preferred over the others for treating alcohol withdrawal syndrome: studies have shown benefits using short-acting, intermediate-acting, and long-acting agents. The choice of drug is variable and patient-specific.16,18,26
Benzodiazepines promote and enhance binding of the inhibitory neurotransmitter GABA to GABAA receptors in the central nervous system.27 As a class, benzodiazepines are all structurally related and produce the same effects—namely, sedation, hypnosis, decreased anxiety, muscle relaxation, anterograde amnesia, and anticonvulsant activity.27
The most studied benzodiazepines for treating and preventing alcohol withdrawal syndrome are chlordiazepoxide, oxazepam, and lorazepam,16–20 whereas diazepam was used in older studies.23
Diazepam and chlordiazepoxide are metabolized by oxidation, each sharing the long-acting active metabolite desmethyldiazepam (half-life 72 hours), and short-acting metabolite oxazepam (half-life 8 hours).27 In addition, the parent drugs also have varying pharmacokinetic profiles: diazepam has a half-life of more than 30 hours and chlordiazepoxide a half-life of about 8 hours. Chlordiazepoxide and diazepam’s combination of both long- and short-acting benzodiazepine activity provides long-term efficacy in attenuating withdrawal symptoms, but chlordiazepoxide’s shorter parent half-life allows more frequent dosing.
Lorazepam (half-life 10–20 hours) and oxazepam (half-life 5–20 hours) undergo glucuronide conjugation and do not have metabolites.27,28 Table 3 provides a pharmacokinetic summary.27,28
Various dosage regimens are used in giving benzodiazepines, the most common being symptom-triggered therapy, governed by assessment scales, and scheduled around-the-clock therapy.29 Current evidence supports symptom-triggered therapy in most inpatients who are not critically ill, as it can reduce both benzodiazepine use and adverse drug events and can reduce the length of stay.16,19
Trials of symptom-triggered benzodiazepine therapy
Most inpatient trials of symptom-triggered therapy (Table 4)3,16–20 used the CIWA-Ar scale for monitoring. In some of the studies, benzodiazepines were given if the score was 8 or higher, but others used cut points as high as 15 or higher. Doses:
- Chlordiazepoxide (first dose 25–100 mg)
- Lorazepam (first dose 0.5–2 mg)
- Oxazepam (30 mg).
After each dose, patients were reevaluated at intervals of 30 minutes to 8 hours.
Most of these trials showed no difference in rates of adverse drug events such as seizures, hallucinations, and lethargy with symptom-triggered therapy compared with scheduled therapy.16,18,20 They also found either no difference in the incidence of delirium tremens, or a lower incidence of delirium tremens with symptom-triggered therapy than with scheduled therapy.16–18,20
Weaver et al19 found no difference in length of stay between scheduled therapy and symptom-triggered therapy, but Saitz et al16 reported a median benzodiazepine treatment duration of 9 hours with symptom-triggered therapy vs 68 hours with fixed dosing. Thus, the study by Saitz et al suggests that hospitalization might be shorter with symptom-triggered therapy.
Many of the trials had notable limitations related to the diversity of patients enrolled and the protocols for both symptom-triggered therapy and fixed dosing. Some trials enrolled only inpatients in detoxification programs; others focused on inpatients with acute medical illness. The inpatient alcohol treatment trials16,18 excluded medically ill patients and those with concurrent psychiatric illness,16,18 and one excluded patients with seizures.16 One of the inpatient alcohol treatment program trials16 excluded patients on beta-blockers or clonidine because of concern that these drugs could mask withdrawal symptoms, whereas trials in medically ill patients allowed these same drugs.17,19,20
Most of the patients were men (approximately 75%, but ranging from 74% to 100%), and therefore the study results may not be as applicable to women.16–20 Most participants were middle-aged, with average ages in all studies between 46 and 55. Finally, the studies used a wide range of medications and dosing, with patient monitoring intervals ranging from every 30 minutes to every 8 hours.16–20
In a 2010 Cochrane analysis, Amato et al29 concluded that the limited evidence available favors symptom-triggered regimens over fixed-dosing regimens, but that differences in isolated trials should be interpreted very cautiously.
Therapeutic ethanol
Aside from the lack of evidence to support its use in alcohol withdrawal syndrome, prescribing oral ethanol to alcoholic patients clearly poses an ethical dilemma. However, giving ethanol intravenously has been studied, mostly in surgical trauma patients.30
Early reports described giving intravenous ethanol on a gram-to-gram basis to match the patient’s consumption before admission to prevent alcohol withdrawal syndrome. But later studies reported prevention of alcohol withdrawal syndrome with very small amounts of intravenous ethanol.30,31 While clinical trials have been limited to ICU patients, ethanol infusion at an initial rate of 2.5 to 5 g per hour and titrated up to 10 g per hour has appeared to be safe and effective for preventing alcohol withdrawal syndrome.30,31 The initial infusion rate of 2.5 to 5 g per hour is equivalent to 4 to 10 alcoholic beverages per 24 hours.
Nevertheless, ethanol infusion carries the potential for toxicities (eg, gastric irritation, precipitation of acute hepatic failure, hypoglycemia, pancreatitis, bone marrow suppression, prolonged wound healing) and drug interactions (eg, with anticoagulants and anticonvulsants). Thus, ethanol is neither widely used nor recommended.25,31
ADJUNCTIVE THERAPIES
Many medications are used adjunctively in the acute setting, both for symptoms of alcohol withdrawal syndrome and for agitation.
Haloperidol
No clinical trial has yet examined haloperidol monotherapy in patients with alcohol withdrawal syndrome in either general medical units or intensive care units. Yet haloperidol remains important and is recommended as an adjunct therapy for agitation.23,32 Dosing of haloperidol in protocols for surgical patients ranged from 2 to 5 mg intravenously every 0.5 to 2 hours, with a maximum dosage of 0.5 mg per kg per 24 hours.7,33
Alpha-2 agonists
Alpha-2 agonists are thought to reduce sympathetic overdrive and the autonomic symptoms associated with alcohol withdrawal syndrome, and these agents (primarily clonidine) have been studied in the treatment of alcohol withdrawal syndrome.34,35
Clonidine. In a Swedish study,34 26 men ages 20 to 55 who presented with the tremor, sweating, dysphoria, tension, anxiety, and tachycardia associated with alcohol withdrawal syndrome received clonidine 4 µg per kg twice daily or carbamazepine 200 mg three to four times daily in addition to an antiepileptic. Adjunctive use of a benzodiazepine was allowed at night in both groups. No statistically significant difference in symptom reduction was noted between the two groups, and there was no difference in total benzodiazepine use.
Dexmedetomidine, given intravenously, has been tested as an adjunct to benzodiazepine treatment in severe alcohol withdrawal syndrome. It has been shown to decrease the amount of total benzodiazepine needed compared with benzodiazepine therapy alone, but no differences have been seen in length of hospital stay.36–39 However, research on this drug so far is limited to ICU patients.
Beta-blockers
Beta-blockers have been used in inpatients with alcohol withdrawal syndrome to reduce heart rate and potentially reduce alcohol craving. However, the data are limited and conflicting.
Atenolol 50 to 100 mg daily, in a study in 120 patients, reduced length of stay (4 vs 5 days), reduced benzodiazepine use, and improved vital signs and behavior compared with placebo.40
Propranolol 40 mg every 6 hours reduced arrhythmias but increased hallucinations when used alone in a study in 47 patients.41 When used in combination with chlordiazepoxide, no benefit was seen in arrhythmia reduction.41
Barbiturates and other antiepileptics
Data continue to emerge on antiepileptics as both monotherapy and adjunctive therapy for alcohol withdrawal syndrome. Barbiturates as monotherapy were largely replaced by benzodiazepines in view of the narrow therapeutic index of barbiturates and their full agonist effect on the GABA receptor complex. However, phenobarbital has been evaluated in patients presenting with severe alcohol withdrawal syndrome or resistant alcohol withdrawal (ie, symptoms despite large or repeated doses of benzodiazepines) as an adjunct to benzodiazepines.42,43
In addition, a newer trial44 involved giving a single dose of phenobarbital in the emergency department in combination with a CIWA-Ar–based benzodiazepine protocol, compared with the benzodiazepine protocol alone. The group that received phenobarbital had fewer ICU admissions; its evaluation is ongoing.
The three other medications with the most data are carbamazepine, valproic acid, and gabapentin.45,46 However, the studies were small and the benefit was modest. Although these agents are possible alternatives in protracted alcohol withdrawal syndrome, no definite conclusion can be made regarding their place in therapy.46
RECOMMENDATIONS FOR DRUG THERAPY AND SUPPORTIVE CARE
Which benzodiazepine to use?
No specific benzodiazepine is recommended over the others for managing alcohol withdrawal syndrome, but studies best support the long-acting agent chlordiazepoxide.16,17,20 Other benzodiazepines such as lorazepam and oxazepam have proved to be beneficial, but drugs should be selected on the basis of patient characteristics and drug metabolism.18,19,27
Patients with severe liver dysfunction and the elderly may have slower oxidative metabolism, so the effects of medications that are primarily oxidized, such as chlordiazepoxide and diazepam, may be prolonged. Therefore, lorazepam and oxazepam would be preferred in these groups.47 While most patients with alcohol withdrawal syndrome and liver dysfunction do not have advanced cirrhosis, we recommend liver function testing (serum aspartate aminotransferase, alanine aminotransferase, and alkaline phosphatase levels) and screening for liver disease, given the drug metabolism and package insert caution for use in those with impaired hepatic function.48
Patients with end-stage renal disease (stage 5 chronic kidney disease) or acute kidney injury should not receive parenteral diazepam or lorazepam. The rationale is the potential accumulation of propylene glycol, the solvent used in these formulations.
In the elderly, the Beers list of drugs to avoid in older adults includes benzodiazepines, not differentiating individual benzodiazepines in terms of risk.49 However, chlordiazepoxide may be preferable to diazepam due to its shorter parent half-life and lower lipophilicity.27 Few studies have been done using benzodiazepines in elderly patients with alcohol withdrawal syndrome, but those published have shown either equivalent dosing required compared with younger patients or more severe withdrawal for which they received greater amounts of chlordiazepoxide.9,12 Lorazepam and oxazepam have less potential to accumulate in the elderly compared with the nonelderly due to the drugs’ metabolic profiles; lorazepam is the preferred agent because of its faster onset of action.47 Ultimately, the choice of benzodiazepine in elderly patients with alcohol withdrawal syndrome should be based on patient-specific characteristics.
How should benzodiazepines be dosed?
While the CIWA-Ar thresholds and subsequent dosing of benzodiazepines varied in different studies, we recommend starting benzodiazepine therapy at a CIWA-Ar score of 8 or higher, with subsequent dosing based on score reassessment. Starting doses of benzodiazepines should be chlordiazepoxide 25 to 50 mg, lorazepam 1 to 2 mg, or oxazepam 15 mg.16–20
Subsequent doses should be titrated upward, increasing by 1.5 to 2 times the previous dose and monitored at least every 1 to 2 hours after dose adjustments. Once a patient is stable and the CIWA-Ar score is less than 8, monitoring intervals can be extended to every 4 to 8 hours. If the CIWA-Ar score is more than 20, studies suggest the need for patient reevaluation for transfer to the ICU; however, some health systems have a lower threshold for this intervention.7,14,50
Dosing algorithms and CIWA-Ar goals may vary slightly from institution to institution, but it has been shown that symptom-triggered therapy works best when hospitals have a protocol for it and staff are adequately trained to assess patients with alcohol withdrawal syndrome.7,50,51 Suggestions for dose ranges and symptom-triggered therapy are shown in Table 5.
In case of benzodiazepine overdose or potential benzodiazepine-induced delirium, flumazenil could be considered.52
Patients who should not receive symptom-triggered therapy include immediate postoperative patients in whom clinicians cannot properly assess withdrawal symptoms and patients with a history of DTs.51 While controversy exists regarding the use of symptom-triggered therapy in patients with complicated medical comorbidities, there are data to support symptom-triggered therapy in some ICU patients, as it has resulted in less benzodiazepine use and reduced mechanical ventilation.53,54
There are limited data to support phenobarbital in treating resistant alcohol withdrawal syndrome, either alone or concurrently with benzodiazepines, in escalating doses ranging from 65 to 260 mg, with a maximum daily dose of 520 mg.42,55,56
Haloperidol
For patients exhibiting agitation despite benzodiazepine therapy, giving haloperidol adjunctively can be beneficial.
Haloperidol can be used in medical patients as an adjunctive therapy for agitation, but caution is advised because of the potential for a lowering of the seizure threshold, extrapyramidal effects, and risk of QTc prolongation leading to arrhythmias. Patients considered at highest risk for torsades de pointes may have a QTc of 500 msec or greater.57
Patients should also be screened for factors that have been shown to be independent predictors of QTc prolongation (female sex, diagnosis of myocardial infarction, septic shock or left ventricular dysfunction, other QT-prolonging drugs, age > 68, baseline QTc ≥ 450 msec, and hypokalemia).58 If combined predictors have been identified, it is recommended that haloperidol be avoided.
If haloperidol is to be given, a baseline electrocardiogram and electrolyte panel should be obtained, with daily electrocardiograms thereafter, as well as ongoing review of the patient’s medications to minimize drug interactions that could further increase the risk for QTc prolongation.
Suggested haloperidol dosing is 2 to 5 mg intravenously every 0.5 to 2 hours with a maximum dose of 0.5 mg/kg/24 hours.8,33 A maximum of 35 mg of intravenous haloperidol should be used in a 24-hour period to avoid QTc prolongation.57
Antihypertensive therapy
Many patients receive symptomatic relief of autonomic hyperreactivity with benzodiazepines. However, some may require additional antihypertensive therapy for cardiac adrenergic symptoms (hypertension, tachycardia) if symptoms do not resolve by treating other medical problems commonly seen in patients with alcohol withdrawal syndrome, such as dehydration and electrolyte imbalances.7
Published protocols suggest giving clonidine 0.1 mg orally every hour up to three times as needed until systolic blood pressure is less than 140 mm Hg (less than 160 mm Hg if the patient is over age 60) and diastolic pressure is less than 90 mm Hg.51 Once the patient is stabilized, the dosing can be scheduled to a maximum of 2.4 mg daily.59 However, we believe that the use of clonidine should be restricted to patients who have a substantial increase in blood pressure over baseline or are nearing a hypertensive urgency or emergency (pressures > 180/120 mm Hg) and should not be used to treat other general symptoms associated with alcohol withdrawal syndrome.42
In addition, based on limited evidence, we recommend using beta-blockers only in patients with symptomatic tachycardia or as an adjunct in hypertension management.40,41
Therapies to avoid in acutely ill medical patients
Ethanol is not recommended. Instead, intravenous benzodiazepines should be given in patients presenting with severe alcohol withdrawal syndrome.
Antiepileptics, including valproic acid, carbamazepine, and pregabalin, lack benefit in these patients either as monotherapy or as adjunctive therapy and so are not recommended.45,60–62
Magnesium supplementation (in patients with normal serum magnesium levels) should not be given, as no clinical benefit has been shown.63
- American Psychiatric Association. Diagnostic and Statistical Manual of Mental Disorders. 5th ed. Arlington, VA: American Psychiatric Association, 2013:501.
- Bayard M, McIntyre J, Hill KR, Woodside J Jr. Alcohol withdrawal syndrome. Am Fam Physician 2004; 69:1443–1450.
- Kosten TR, O'Connor PG. Management of drug and alcohol withdrawal. N Engl J Med 2003; 348:1786–1795.
- Isbell H, Fraser HF, Wilker A, Bellevile RE, Eisenman AJ. An experimental study of the etiology of rum fits and delirium tremens. Q J Stud Alcohol 1955; 16:1–33.
- Khan A, Levy P, DeHorn S, Miller W, Compton S. Predictors of mortality in patients with delirium tremens. Acad Emerg Med 2008; 15:788–790.
- Monte R, Rabuñal R, Casariego E, López-Agreda H, Mateos A, Pértega S. Analysis of the factors determining survival of alcoholic withdrawal syndrome patients in a general hospital. Alcohol Alcohol 2010; 45:151–158.
- Stanley KM, Amabile CM, Simpson KN, Couillard D, Norcross ED, Worrall CL. Impact of an alcohol withdrawal syndrome practice guideline on surgical patient outcomes. Pharmacotherapy 2003; 23:843–854.
- Brousse G, Arnaud B, Vorspan F, et al. Alteration of glutamate/GABA balance during acute alcohol withdrawal in emergency department: a prospective analysis. Alcohol Alcohol 2012; 47:501–508.
- Liskow BI, Rinck C, Campbell J, DeSouza C. Alcohol withdrawal in the elderly. J Stud Alcohol 1989; 50:414–421.
- Etherington JM. Emergency management of acute alcohol problems. Part 1: uncomplicated withdrawal. Can Fam Physician 1996; 42:2186–2190.
- Letizia M, Reinbolz M. Identifying and managing acute alcohol withdrawal in the elderly. Geriatr Nurs 2005; 26:176–183.
- Brower KJ, Mudd S, Blow FC, Young JP, Hill EM. Severity and treatment of alcohol withdrawal in elderly versus younger patients. Alcohol Clin Exp Res 1994; 18:196–201.
- Williams D, Lewis J, McBride A. A comparison of rating scales for the alcohol-withdrawal syndrome. Alcohol Alcohol 2001; 36:104–108.
- Reoux JP, Oreskovich MR. A comparison of two versions of the Clinical Institute Withdrawal Assessment for Alcohol: the CIWA-Ar and CIWA-AD. Am J Addict 2006; 15:85–93.
- Sullivan JT, Sykora K, Schneiderman J, Naranjo CA, Sellers EM. Assessment of alcohol withdrawal: the revised Clinical Institute Withdrawal Assessment for alcohol scale (CIWA-Ar). Br J Addict 1989; 84:1353–1357.
- Saitz R, Mayo-Smith MF, Roberts MS, Redmond HA, Bernard DR, Calkins DR. Individualized treatment for alcohol withdrawal. A randomized double-blind controlled trial. JAMA 1994; 272:519–523.
- Jaeger TM, Lohr RH, Pankratz VS. Symptom-triggered therapy for alcohol withdrawal syndrome in medical inpatients. Mayo Clin Proc 2001; 76:695–701.
- Daeppen JB, Gache P, Landry U, et al. Symptom-triggered vs fixed-schedule doses of benzodiazepine for alcohol withdrawal: a randomized treatment trial. Arch Intern Med 2002; 162:1117–1121.
- Weaver MF, Hoffman HJ, Johnson RE, Mauck K. Alcohol withdrawal pharmacotherapy for inpatients with medical comorbidity. J Addict Dis 2006; 25:17–24.
- Reoux JP, Miller K. Routine hospital alcohol detoxification practice compared to symptom triggered management with an Objective Withdrawal Scale (CIWA-Ar). Am J Addict 2000; 9:135–144.
- Wetterling T, Kanitz RD, Besters B, et al. A new rating scale for the assessment of the alcohol-withdrawal syndrome (AWS scale). Alcohol Alcohol 1997; 32:753–760.
- Myrick H, Anton RF. Treatment of alcohol withdrawal. Alcohol Health Res World 1998; 22:38–43.
- Mayo-Smith MF, Beecher LH, Fischer TL, et al; Working Group on the Management of Alcohol Withdrawal Delirium, Practice Guidelines Committee, American Society of Addiction Medicine. Management of alcohol withdrawal delirium. An evidence-based practice guideline. Arch Intern Med 2004; 164:1405–1412.
- Sellers EM, Sullivan JT, Somer G, Sykora K. Characterization of DSM-III-R criteria for uncomplicated alcohol withdrawal provides an empirical basis for DSM-IV. Arch Gen Psychiatry 1991; 48:442–447.
- Sarff M, Gold JA. Alcohol withdrawal syndromes in the intensive care unit. Crit Care Med 2010; 38(suppl):S494–S501.
- Kumar CN, Andrade C, Murthy P. A randomized, double-blind comparison of lorazepam and chlordiazepoxide in patients with uncomplicated alcohol withdrawal. J Stud Alcohol Drugs 2009; 70:467–474.
- Bird RD, Makela EH. Alcohol withdrawal: what is the benzodiazepine of choice? Ann Pharmacother 1994; 28:67–71.
- Perry EC. Inpatient management of acute alcohol withdrawal syndrome. CNS Drugs 2014; 28:401–410.
- Amato L, Minozzi S, Vecchi S, Davoli M. Benzodiazepines for alcohol withdrawal. Cochrane Database Syst Rev 2010; 3:CD005063.
- Weinberg JA, Magnotti LJ, Fischer PE, et al. Comparison of intravenous ethanol versus diazepam for alcohol withdrawal prophylaxis in the trauma ICU: results of a randomized trial. J Trauma 2008; 64:99–104.
- Craft PP, Foil MB, Cunningham PR, Patselas PC, Long-Snyder BM, Collier MS. Intravenous ethanol for alcohol detoxification in trauma patients. South Med J 1994; 87:47–54.
- Ungur LA, Neuner B, John S, Wernecke K, Spies C. Prevention and therapy of alcohol withdrawal on intensive care units: systematic review of controlled trials. Alcohol Clin Exp Res 2013; 37:675–686.
- Lansford CD, Guerriero CH, Kocan MJ, et al. Improved outcomes in patients with head and neck cancer using a standardized care protocol for postoperative alcohol withdrawal. Arch Otolaryngol Head Neck Surg 2008; 134:865–872.
- Walinder J, Balldin J, Bokstrom K, Karlsson I, Lundstrom B, Svensson TH. Clonidine suppression of the alcohol withdrawal syndrome. Drug Alcohol Depend 1981; 8:345–348.
- Muzyk AJ, Fowler JA, Norwood DK, Chilipko A. Role of alpha2-agonists in the treatment of acute alcohol withdrawal. Ann Pharmacother 2011; 45:649–657.
- Crispo AL, Daley MJ, Pepin JL, Harford PH, Brown CV. Comparison of clinical outcomes in nonintubated patients with severe alcohol withdrawal syndrome treated with continuous-infusion sedatives: dexmedetomidine versus benzodiazepines. Pharmacotherapy 2014; 34:910–917.
- VanderWeide LA, Foster CJ, MacLaren R, Kiser TH, Fish DN, Mueller SW. Evaluation of early dexmedetomidine addition to the standard of care for severe alcohol withdrawal in the ICU: a retrospective controlled cohort study. J Intensive Care Med 2014. [Epub ahead of print October 16, 2014]
- Rayner SG, Weinert CR, Peng H, Jepsen S, Broccard AF. Dexmedetomidine as adjunct treatment for severe alcohol withdrawal in the ICU. Ann Intensive Care 2012; 2:12.
- Muzyk AJ, Kerns S, Brudney S, Gagliardi JP. Dexmedetomidine for the treatment of alcohol withdrawal syndrome: rationale and current status of research. CNS Drugs 2013; 27:913–920.
- Kraus ML, Gottlieb LD, Horwitz RI, Anscher M. Randomized clinical trial of atenolol in patients with alcohol withdrawal. N Engl J Med 1985; 313:905–909.
- Zilm DH, Jacob MS, MacLeod SM, Sellers EM, Ti TY. Propranolol and chlordiazepoxide effects on cardiac arrhythmias during alcohol withdrawal. Alcohol Clin Exp Res 1980; 4:400–405.
- Hack JB, Hoffmann RS, Nelson LS. Resistant alcohol withdrawal: does an unexpectedly large sedative requirement identify these patients early? J Med Toxicol 2006; 2:55–60.
- Hayner CE, Wuestefeld NL, Bolton PJ. Phenobarbital treatment in a patient with resistant alcohol withdrawal syndrome. Pharmacotherapy 2009; 29:875–878.
- Rosenson J, Clements C, Simon B, et al. Phenobarbital for acute alcohol withdrawal: a prospective randomized double-blind placebo-controlled study. J Emerg Med 2013; 44:592–598.e2.
- Prince V, Turpin KR. Treatment of alcohol withdrawal syndrome with carbamazepine, gabapentin, and nitrous oxide. Am J Health Syst Pharm 2008; 65:1039–1047.
- Leggio L, Kenna GA, Swift RM. New developments for the pharmacological treatment of alcohol withdrawal syndrome. A focus on non-benzodiazepine GABAergic medications. Prog Neuropsychopharmacol Biol Psychiatry 2008; 32:1106–1117.
- Peppers MP. Benzodiazepines for alcohol withdrawal in the elderly and in patients with liver disease. Pharmacotherapy 1996; 16:49–57.
- Valeant Pharmaceuticals North America LLC. Librium—chlordiazepoxide hydrochloride capsule, gelatin coated. http://dailymed.nlm.nih.gov/dailymed/archives/fdaDrugInfo.cfm?archiveid=125207. Accessed November 20, 2015.
- American Geriatrics Society 2012 Beers Criteria Update Expert Panel. American Geriatrics Society updated Beers Criteria for potentially inappropriate medication use in older adults. J Am Geriatr Soc 2012; 60:616–631.
- Hecksel KA, Bostwick JM, Jaeger TM, Cha SS. Inappropriate use of symptom-triggered therapy for alcohol withdrawal in the general hospital. Mayo Clin Proc 2008; 83:274–279.
- Manasco A, Chang S, Larriviere J, Hamm LL, Glass M. Alcohol withdrawal. South Med J 2012; 105:607–612.
- Moore PW, Donovan JW, Burkhart KK, et al. Safety and efficacy of flumazenil for reversal of iatrogenic benzodiazepine-associated delirium toxicity during treatment of alcohol withdrawal, a retrospective review at one center. J Med Toxicol 2014; 10:126–132.
- Bostwick JM, Lapid MI. False positives on the clinical institute withdrawal assessment for alcohol-revised: is this scale appropriate for use in the medically ill? Psychosomatics 2004; 45:256–261.
- de Wit M, Jones DG, Sessler CN, Zilberberg MD, Weaver MF. Alcohol-use disorders in the critically ill patient. Chest 2010; 138:994–1003.
- Young GP, Rores C, Murphy C, Dailey RH. Intravenous phenobarbital for alcohol withdrawal and convulsions. Ann Emerg Med 1987; 16:847–850.
- Hendey GW, Dery RA, Barnes RL, Snowden B, Mentler P. A prospective, randomized trial of phenobarbital versus benzodiazepines for acute alcohol withdrawal. Am J Emerg Med 2011; 29:382–385.
- Sharma ND, Rosman HS, Padhi ID, Tisdale JE. Torsades de pointes associated with intravenous haloperidol in critically ill patients. Am J Cardiol 1998; 81:238–240.
- Tisdale JE, Jaynes HA, Kingery JR, et al. Development and validation of a risk score to predict QT interval prolongation in hospitalized patients. Circ Cardiovasc Qual Outcomes 2013; 6:479–487.
- Boehringer Ingelheim Pharmaceuticals, Inc. Product Information: Catapres oral tablets, clonidine HCl oral tablets, 2012.
- Reoux JP, Saxon AJ, Malte CA, Baer JS, Sloan KL. Divalproex sodium in alcohol withdrawal: a randomized double-blind placebo-controlled clinical trial. Alcohol Clin Exp Res 2001; 25:1324–1329.
- Malcolm R, Ballenger JC, Sturgis ET, Anton R. Double-blind controlled trial comparing carbamazepine to oxazepam treatment of alcohol withdrawal. Am J Psychiatry 1989; 146:617–621.
- Förg A, Hein J, Volkmar K, et al. Efficacy and safety of pregabalin in the treatment of alcohol withdrawal syndrome: a randomized placebo-controlled trial. Alcohol Alcohol 2012; 47:149–155.
- Wilson A, Vulcano B. A double-blind, placebo-controlled trial of magnesium sulfate in the ethanol withdrawal syndrome. Alcohol Clin Exp Res 1984; 8:542–545.
- American Psychiatric Association. Diagnostic and Statistical Manual of Mental Disorders. 5th ed. Arlington, VA: American Psychiatric Association, 2013:501.
- Bayard M, McIntyre J, Hill KR, Woodside J Jr. Alcohol withdrawal syndrome. Am Fam Physician 2004; 69:1443–1450.
- Kosten TR, O'Connor PG. Management of drug and alcohol withdrawal. N Engl J Med 2003; 348:1786–1795.
- Isbell H, Fraser HF, Wilker A, Bellevile RE, Eisenman AJ. An experimental study of the etiology of rum fits and delirium tremens. Q J Stud Alcohol 1955; 16:1–33.
- Khan A, Levy P, DeHorn S, Miller W, Compton S. Predictors of mortality in patients with delirium tremens. Acad Emerg Med 2008; 15:788–790.
- Monte R, Rabuñal R, Casariego E, López-Agreda H, Mateos A, Pértega S. Analysis of the factors determining survival of alcoholic withdrawal syndrome patients in a general hospital. Alcohol Alcohol 2010; 45:151–158.
- Stanley KM, Amabile CM, Simpson KN, Couillard D, Norcross ED, Worrall CL. Impact of an alcohol withdrawal syndrome practice guideline on surgical patient outcomes. Pharmacotherapy 2003; 23:843–854.
- Brousse G, Arnaud B, Vorspan F, et al. Alteration of glutamate/GABA balance during acute alcohol withdrawal in emergency department: a prospective analysis. Alcohol Alcohol 2012; 47:501–508.
- Liskow BI, Rinck C, Campbell J, DeSouza C. Alcohol withdrawal in the elderly. J Stud Alcohol 1989; 50:414–421.
- Etherington JM. Emergency management of acute alcohol problems. Part 1: uncomplicated withdrawal. Can Fam Physician 1996; 42:2186–2190.
- Letizia M, Reinbolz M. Identifying and managing acute alcohol withdrawal in the elderly. Geriatr Nurs 2005; 26:176–183.
- Brower KJ, Mudd S, Blow FC, Young JP, Hill EM. Severity and treatment of alcohol withdrawal in elderly versus younger patients. Alcohol Clin Exp Res 1994; 18:196–201.
- Williams D, Lewis J, McBride A. A comparison of rating scales for the alcohol-withdrawal syndrome. Alcohol Alcohol 2001; 36:104–108.
- Reoux JP, Oreskovich MR. A comparison of two versions of the Clinical Institute Withdrawal Assessment for Alcohol: the CIWA-Ar and CIWA-AD. Am J Addict 2006; 15:85–93.
- Sullivan JT, Sykora K, Schneiderman J, Naranjo CA, Sellers EM. Assessment of alcohol withdrawal: the revised Clinical Institute Withdrawal Assessment for alcohol scale (CIWA-Ar). Br J Addict 1989; 84:1353–1357.
- Saitz R, Mayo-Smith MF, Roberts MS, Redmond HA, Bernard DR, Calkins DR. Individualized treatment for alcohol withdrawal. A randomized double-blind controlled trial. JAMA 1994; 272:519–523.
- Jaeger TM, Lohr RH, Pankratz VS. Symptom-triggered therapy for alcohol withdrawal syndrome in medical inpatients. Mayo Clin Proc 2001; 76:695–701.
- Daeppen JB, Gache P, Landry U, et al. Symptom-triggered vs fixed-schedule doses of benzodiazepine for alcohol withdrawal: a randomized treatment trial. Arch Intern Med 2002; 162:1117–1121.
- Weaver MF, Hoffman HJ, Johnson RE, Mauck K. Alcohol withdrawal pharmacotherapy for inpatients with medical comorbidity. J Addict Dis 2006; 25:17–24.
- Reoux JP, Miller K. Routine hospital alcohol detoxification practice compared to symptom triggered management with an Objective Withdrawal Scale (CIWA-Ar). Am J Addict 2000; 9:135–144.
- Wetterling T, Kanitz RD, Besters B, et al. A new rating scale for the assessment of the alcohol-withdrawal syndrome (AWS scale). Alcohol Alcohol 1997; 32:753–760.
- Myrick H, Anton RF. Treatment of alcohol withdrawal. Alcohol Health Res World 1998; 22:38–43.
- Mayo-Smith MF, Beecher LH, Fischer TL, et al; Working Group on the Management of Alcohol Withdrawal Delirium, Practice Guidelines Committee, American Society of Addiction Medicine. Management of alcohol withdrawal delirium. An evidence-based practice guideline. Arch Intern Med 2004; 164:1405–1412.
- Sellers EM, Sullivan JT, Somer G, Sykora K. Characterization of DSM-III-R criteria for uncomplicated alcohol withdrawal provides an empirical basis for DSM-IV. Arch Gen Psychiatry 1991; 48:442–447.
- Sarff M, Gold JA. Alcohol withdrawal syndromes in the intensive care unit. Crit Care Med 2010; 38(suppl):S494–S501.
- Kumar CN, Andrade C, Murthy P. A randomized, double-blind comparison of lorazepam and chlordiazepoxide in patients with uncomplicated alcohol withdrawal. J Stud Alcohol Drugs 2009; 70:467–474.
- Bird RD, Makela EH. Alcohol withdrawal: what is the benzodiazepine of choice? Ann Pharmacother 1994; 28:67–71.
- Perry EC. Inpatient management of acute alcohol withdrawal syndrome. CNS Drugs 2014; 28:401–410.
- Amato L, Minozzi S, Vecchi S, Davoli M. Benzodiazepines for alcohol withdrawal. Cochrane Database Syst Rev 2010; 3:CD005063.
- Weinberg JA, Magnotti LJ, Fischer PE, et al. Comparison of intravenous ethanol versus diazepam for alcohol withdrawal prophylaxis in the trauma ICU: results of a randomized trial. J Trauma 2008; 64:99–104.
- Craft PP, Foil MB, Cunningham PR, Patselas PC, Long-Snyder BM, Collier MS. Intravenous ethanol for alcohol detoxification in trauma patients. South Med J 1994; 87:47–54.
- Ungur LA, Neuner B, John S, Wernecke K, Spies C. Prevention and therapy of alcohol withdrawal on intensive care units: systematic review of controlled trials. Alcohol Clin Exp Res 2013; 37:675–686.
- Lansford CD, Guerriero CH, Kocan MJ, et al. Improved outcomes in patients with head and neck cancer using a standardized care protocol for postoperative alcohol withdrawal. Arch Otolaryngol Head Neck Surg 2008; 134:865–872.
- Walinder J, Balldin J, Bokstrom K, Karlsson I, Lundstrom B, Svensson TH. Clonidine suppression of the alcohol withdrawal syndrome. Drug Alcohol Depend 1981; 8:345–348.
- Muzyk AJ, Fowler JA, Norwood DK, Chilipko A. Role of alpha2-agonists in the treatment of acute alcohol withdrawal. Ann Pharmacother 2011; 45:649–657.
- Crispo AL, Daley MJ, Pepin JL, Harford PH, Brown CV. Comparison of clinical outcomes in nonintubated patients with severe alcohol withdrawal syndrome treated with continuous-infusion sedatives: dexmedetomidine versus benzodiazepines. Pharmacotherapy 2014; 34:910–917.
- VanderWeide LA, Foster CJ, MacLaren R, Kiser TH, Fish DN, Mueller SW. Evaluation of early dexmedetomidine addition to the standard of care for severe alcohol withdrawal in the ICU: a retrospective controlled cohort study. J Intensive Care Med 2014. [Epub ahead of print October 16, 2014]
- Rayner SG, Weinert CR, Peng H, Jepsen S, Broccard AF. Dexmedetomidine as adjunct treatment for severe alcohol withdrawal in the ICU. Ann Intensive Care 2012; 2:12.
- Muzyk AJ, Kerns S, Brudney S, Gagliardi JP. Dexmedetomidine for the treatment of alcohol withdrawal syndrome: rationale and current status of research. CNS Drugs 2013; 27:913–920.
- Kraus ML, Gottlieb LD, Horwitz RI, Anscher M. Randomized clinical trial of atenolol in patients with alcohol withdrawal. N Engl J Med 1985; 313:905–909.
- Zilm DH, Jacob MS, MacLeod SM, Sellers EM, Ti TY. Propranolol and chlordiazepoxide effects on cardiac arrhythmias during alcohol withdrawal. Alcohol Clin Exp Res 1980; 4:400–405.
- Hack JB, Hoffmann RS, Nelson LS. Resistant alcohol withdrawal: does an unexpectedly large sedative requirement identify these patients early? J Med Toxicol 2006; 2:55–60.
- Hayner CE, Wuestefeld NL, Bolton PJ. Phenobarbital treatment in a patient with resistant alcohol withdrawal syndrome. Pharmacotherapy 2009; 29:875–878.
- Rosenson J, Clements C, Simon B, et al. Phenobarbital for acute alcohol withdrawal: a prospective randomized double-blind placebo-controlled study. J Emerg Med 2013; 44:592–598.e2.
- Prince V, Turpin KR. Treatment of alcohol withdrawal syndrome with carbamazepine, gabapentin, and nitrous oxide. Am J Health Syst Pharm 2008; 65:1039–1047.
- Leggio L, Kenna GA, Swift RM. New developments for the pharmacological treatment of alcohol withdrawal syndrome. A focus on non-benzodiazepine GABAergic medications. Prog Neuropsychopharmacol Biol Psychiatry 2008; 32:1106–1117.
- Peppers MP. Benzodiazepines for alcohol withdrawal in the elderly and in patients with liver disease. Pharmacotherapy 1996; 16:49–57.
- Valeant Pharmaceuticals North America LLC. Librium—chlordiazepoxide hydrochloride capsule, gelatin coated. http://dailymed.nlm.nih.gov/dailymed/archives/fdaDrugInfo.cfm?archiveid=125207. Accessed November 20, 2015.
- American Geriatrics Society 2012 Beers Criteria Update Expert Panel. American Geriatrics Society updated Beers Criteria for potentially inappropriate medication use in older adults. J Am Geriatr Soc 2012; 60:616–631.
- Hecksel KA, Bostwick JM, Jaeger TM, Cha SS. Inappropriate use of symptom-triggered therapy for alcohol withdrawal in the general hospital. Mayo Clin Proc 2008; 83:274–279.
- Manasco A, Chang S, Larriviere J, Hamm LL, Glass M. Alcohol withdrawal. South Med J 2012; 105:607–612.
- Moore PW, Donovan JW, Burkhart KK, et al. Safety and efficacy of flumazenil for reversal of iatrogenic benzodiazepine-associated delirium toxicity during treatment of alcohol withdrawal, a retrospective review at one center. J Med Toxicol 2014; 10:126–132.
- Bostwick JM, Lapid MI. False positives on the clinical institute withdrawal assessment for alcohol-revised: is this scale appropriate for use in the medically ill? Psychosomatics 2004; 45:256–261.
- de Wit M, Jones DG, Sessler CN, Zilberberg MD, Weaver MF. Alcohol-use disorders in the critically ill patient. Chest 2010; 138:994–1003.
- Young GP, Rores C, Murphy C, Dailey RH. Intravenous phenobarbital for alcohol withdrawal and convulsions. Ann Emerg Med 1987; 16:847–850.
- Hendey GW, Dery RA, Barnes RL, Snowden B, Mentler P. A prospective, randomized trial of phenobarbital versus benzodiazepines for acute alcohol withdrawal. Am J Emerg Med 2011; 29:382–385.
- Sharma ND, Rosman HS, Padhi ID, Tisdale JE. Torsades de pointes associated with intravenous haloperidol in critically ill patients. Am J Cardiol 1998; 81:238–240.
- Tisdale JE, Jaynes HA, Kingery JR, et al. Development and validation of a risk score to predict QT interval prolongation in hospitalized patients. Circ Cardiovasc Qual Outcomes 2013; 6:479–487.
- Boehringer Ingelheim Pharmaceuticals, Inc. Product Information: Catapres oral tablets, clonidine HCl oral tablets, 2012.
- Reoux JP, Saxon AJ, Malte CA, Baer JS, Sloan KL. Divalproex sodium in alcohol withdrawal: a randomized double-blind placebo-controlled clinical trial. Alcohol Clin Exp Res 2001; 25:1324–1329.
- Malcolm R, Ballenger JC, Sturgis ET, Anton R. Double-blind controlled trial comparing carbamazepine to oxazepam treatment of alcohol withdrawal. Am J Psychiatry 1989; 146:617–621.
- Förg A, Hein J, Volkmar K, et al. Efficacy and safety of pregabalin in the treatment of alcohol withdrawal syndrome: a randomized placebo-controlled trial. Alcohol Alcohol 2012; 47:149–155.
- Wilson A, Vulcano B. A double-blind, placebo-controlled trial of magnesium sulfate in the ethanol withdrawal syndrome. Alcohol Clin Exp Res 1984; 8:542–545.
KEY POINTS
- Patients diagnosed with or suspected of having alcohol withdrawal syndrome need a thorough history and physical examination, appropriate laboratory tests, and monitoring using the revised Clinical Institute Withdrawal Assessment for Alcohol scale (CIWA-Ar) or a similar scale.
- For most patients, benzodiazepines should be given in a symptom-triggered fashion, using the CIWA-Ar score as a monitoring tool. Alternatively, scheduled benzodiazepine dosing should be considered for patients with a history of alcohol withdrawal delirium or for patients in whom withdrawal symptoms cannot be easily assessed.
- The choice of benzodiazepine should be individualized, based on the half-life of the drug, comorbid diseases, and monitoring plans.
- Many patients with alcohol withdrawal syndrome require fluid and electrolyte replacement, as well as adjunctive therapies such as haloperidol for delirium and antihypertensives for cardiac or adrenergic symptoms. No standard currently exists for drug dosing, administration, and assessment protocols in these patients. Therefore, clinicians are adapting study designs and assessment scales to meet patients’ individual needs.
Approach to asymptomatic creatine kinase elevation
Measuring serum creatine kinase (CK) is an important part of the evaluation of patients with muscle weakness or myalgia, and of assessing patients with myopathies or rhabdomyolysis. But elevated CK sometimes is an incidental finding in a patient without muscle-related symptoms or with only minimal nonspecific muscle symptoms (eg, cramps, spasms, fatigue) that do not significantly interfere with activities of daily living. This condition is sometimes referred to as “asymptomatic hyper-CK-emia.” Four other muscle enzymes that may also be elevated are aspartate aminotransferase, alanine aminotransferase, lactate dehydrogenase, and aldolase.
This review focuses on the evaluation of patients with elevated CK without significant muscle-related symptoms and proposes an algorithm for this purpose (Figure 1).
CURRENT THRESHOLDS MAY BE LOW
What appears to be an elevated CK level may in fact be normal, and it is important to determine in the initial assessment whether a CK value is truly abnormal.
Most laboratories use the central 95% of observations in white people as a reference range for serum CK, assuming that levels have a gaussian (bell-shaped) distribution, which is usually about 0 to 200 IU/L. Using these parameters, an abnormal CK level was observed in 19% of men and 5% of women in a study of nearly 1,000 healthy young people,1 leading to overdiagnosis.
The actual distribution of serum CK levels in a healthy population is markedly skewed toward higher values and is nongaussian.1–3 A 97.5% normal threshold is associated with a much lower false-positive rate and is recommended by the European Federation of Neurological Societies (now the European Academy of Neurology).4 This group also recommends pursuing further investigation only for patients whose level is at least 1.5 times the upper limit of normal; this threshold results in only a small reduction in sensitivity.
CK levels vary significantly by sex and race.5 Possible reasons include differences in muscle mass or total body mass and inherited differences in the permeability of the sarcolemma to CK.6 There is also a small reduction in CK levels as people age.2
The European Federation of Neurological Societies suggests redefining elevated CK as values 1.5 times beyond the upper limit of normal. Based on a 97.5% threshold and normal values determined by Brewster et al3 for black and white men and women, the following thresholds can be used to help decide whether to pursue further evaluation4:
- White women—325 IU/L
- White men—504 IU/L
- Black women—621 IU/L
- Black men—1,200 IU/L
PHYSICAL ACTIVITY RAISES CK
CK levels transiently rise after exercise or heavy manual labor. Serum CK levels may increase to as much as 30 times the upper limit of normal within 24 hours of strenuous physical activity, then slowly decline over the next 7 days. The degree of CK elevation depends on the type and duration of exercise, with greater elevation in those who are untrained.2,4
In assessing asymptomatic or minimally symptomatic CK elevation, the test should be repeated after 7 days without exercise. A large community study in Norway found that repeat CK levels in people with incidentally discovered elevated CK were normal after 3 days of rest in 70% of cases.2
NONNEUROMUSCULAR CAUSES
NEED TO BE INVESTIGATED
Asymptomatic or minimally symptomatic elevated CK can be due to a primary neuromuscular disease or a variety of nonneuromuscular causes.
Patients who still have elevated CK after taking into account the 97.5% threshold, repeat testing after a week of rest, and a level more than 1.5 times the upper limit of normal for sex and race should first be evaluated for the many nonneuromuscular conditions that can cause elevated CK (Table 1).7–9
Cardiac causes should be evaluated by history and physical examination, electrocardiography, and possibly testing for cardiac troponins.
Drugs commonly elevate CK
Prescription drugs and supplements are an important and common cause of CK elevation, so it is important to carefully review medications the patient is taking.
Statins can cause myalgia, muscle weakness, and rhabdomyolysis. Up to 5% of users develop CK elevation, typically 2 to 10 times the upper limit of normal.10 CK usually drops after stopping statins but may require weeks to months to normalize. Rarely, statin users develop a serious immune-mediated necrotizing myopathy.11–13
The diversity of response to statin therapy appears to have a genetic basis. The SEARCH Collaborative Group14 conducted a genome-wide association study of 300,000 markers in 85 patients with definite or incipient myopathy and in 90 controls, all of whom were taking simvastatin 80 mg daily. They identified a single-nucleotide polymorphism in the SLCO1B1 gene on chromosome 12 that was strongly associated with a higher risk of statin-induced myopathy.
Patients with statin-related myopathy seem to have a higher frequency of occult metabolic muscle disease than the general population, also suggesting genetic susceptibility, although ascertainment bias could be a factor.14
Mechanisms of CK elevation in response to statins include increased muscle membrane fragility due to decreased cholesterol content, inhibition of isoprenoid production (a necessary step in the synthesis of membrane proteins), and depletion of ubiquinone, leading to mitochondrial dysfunction.
Macro CK: An abnormal enzyme complex
About 4% of patients with asymptomatic or minimally symptomatic elevated CK have “macro CK,” an enzyme complex with an atypically high molecular mass and reduced clearance, resulting in abnormally high blood levels of CK. Macro CK type 1 is more common and is found in up to 1.2% of the general population: complexes are composed of CK and immunoglobulin and are associated with autoimmune diseases.9,15 Macro CK type 2 complexes consist of CK and an undetermined protein and are associated with malignancies.
CK electrophoresis is required to detect macro CK. Types 1 and 2 can be distinguished by protein G affinity chromatography.9,15
Endocrine disorders
Muscle involvement in endocrine disorders often presents with muscle weakness in addition to muscle enzyme abnormalities.
Hypothyroidism often causes weakness, cramps, myalgia, and a mild to moderate serum CK elevation.16 Severe CK elevation has been reported to occur after vigorous exercise.17 Thyroid replacement usually results in normalization of serum CK levels in 1 to 2 months.18
Hyperthyroidism is typically associated with normal serum CK concentrations, but in rare cases it can cause rhabdomyolysis.19
NEUROMUSCULAR CAUSES ARE NOT ALWAYS WORTH PURSUING
Only after the nonneuromuscular causes of elevated CK have been ruled out should neuromuscular disorders be considered (Table 2). Evaluation with electromyography, nerve conduction studies, and muscle biopsy may lead to the diagnosis of a specific neuromuscular disorder: patients may be in the presymptomatic stage of disease and may or may not eventually develop muscle weakness or other symptoms.20,21
Is testing needed?
Most adult dystrophies and metabolic myopathies have no available treatment and their course is often benign, particularly if they present only with asymptomatic elevated CK. The value of a potentially extensive, expensive, and invasive evaluation for a specific neuromuscular cause should be weighed against the limited yield and treatment options. Moreover, specialized testing such as biochemical muscle enzyme analysis, sarcolemmal protein staining, and genetic testing are not available at all centers.
The European Federation of Neurological Societies guidelines recommend biopsy for patients with asymptomatic elevated CK who also have any of the following:
- Abnormal (myopathic) findings on electromyography
- CK more than three times the upper limit of normal
- Age less than 25
- Exercise intolerance.4
Idiopathic inflammatory myopathies rarely present with asymptomatic elevated CK.22–26 In one study,27 they were found in just 5% of patients with asymptomatic elevated CK.
Hypomyopathic dermatomyositis and inclusion body myositis can present with mild CK elevations with normal muscle strength, especially early in the disease course. A myositis subset of antisynthetase syndrome can present with mildly elevated CK and interstitial lung disease.27 Many of the inflammatory myopathies respond to treatment so are worth investigating.
In view of complexities in diagnosis of these conditions, one should proceed with testing only after discussing it with patients. Referral to a rheumatology specialist is preferred.
MUSCLE BIOPSY, ELECTROMYOGRAPHY, AND NERVE CONDUCTION STUDIES
Electromyography, nerve conduction studies, or muscle biopsy, or a combination of these tests, is usually needed to investigate neuromuscular causes of elevated CK.
Muscle biopsy abnormalities are found in about two-thirds of cases of asymptomatic elevated CK, but most abnormalities include nonspecific myopathic changes that are not diagnostic. A muscle biopsy that may include special stains for sarcolemmal proteins for muscular dystrophy and biochemical muscle enzyme analysis for metabolic myopathies is diagnostic in only 20% to 25% cases of asymptomatic elevated CK on average, with a variation between different series of 0% to 79%.7,21,27–33
Electromyography and nerve conduction studies alone add little to the workup of asymptomatic elevated CK apart from a modest negative predictive value and as a guide for muscle biopsy. For a very few neuromuscular disorders causing an elevated CK (eg, motor neuron disease, Charcot-Marie-Tooth disease, myotonic dystrophy), electromyography and nerve conduction studies could suffice to make the diagnosis.
Electromyography and nerve conduction studies detect abnormalities in nearly half of cases of asymptomatic CK elevation,7,21,27,28,30,31,33 but, as with biopsy, most changes are nonspecific. Although electromyography and nerve conduction studies can help distinguish primary neuropathic from myopathic disorders, the sensitivity and specificity are low for diagnosis. Normal studies do not rule out a condition, and abnormal studies are not diagnostic of a particular condition, although completely normal studies provide strong evidence against a severe neuromuscular disorder.
Combined testing
Using combined muscle biopsy, electromyography, and nerve conduction studies, the likelihood of making a diagnosis in patients with asymptomatic elevated CK is 28% on average (range of studies 4%–79%),2,7,21,26–28,30–32 and findings are nonspecific in 30% to 40% of cases. Findings are normal in about 30% to 40% of cases, which are thus diagnosed as idiopathic asymptomatic elevated CK.28–31,34
Prelle et al31 retrospectively reviewed the cases of 114 patients, ages 3 to 70, with incidentally discovered elevated CK and few or no symptoms, who underwent muscle biopsy, electromyography, and nerve conduction studies after nonneuromuscular causes were ruled out. Although muscle biopsy findings were abnormal in 39% of cases, a diagnosis was established in only 18% of cases after an extensive workup: the diagnosis was definitive in only 10% and included dystrophinopathies, metabolic myopathies, and rare noninflammatory myopathies. For the remaining 8%, the diagnosis was probable and included four cases of partial carnitine palmitoyl transferase deficiency, three cases of malignant hyperthermia, and two rare inherited disorders.
DNA testing
In women with a serum CK less than three times the upper limit of normal who have a family history of Duchenne or Becker muscular dystrophy, DNA analysis of blood lymphocytes identifies 70% of carriers.4
IDIOPATHIC ELEVATED SERUM CK
Rowland et al35 first coined the term “idiopathic hyper-CK-emia” and defined it as persistent elevation of serum CK despite a normal neurologic examination and testing, including electromyography, nerve conduction studies, and muscle biopsy.35,36 To receive this diagnosis, patients must also have no family history or clinical evidence of neuromuscular disease.
Idiopathic elevated serum CK is sometimes familial. In one study,37 elevated CK was found in family members of 13 of 28 unrelated probands. In the 13 families, 41 individuals had elevated CK. Genetic studies revealed that the condition is genetically heterogeneous and autosomal dominant in at least 60% of cases, with higher penetrance in men.
D’Adda et al26 followed 55 people with idiopathic elevated CK for 7 years. Ten percent were eventually diagnosed with a neuromuscular disorder, 10% developed malignancy, and the remaining 80% developed no new condition. The CK level normalized or decreased in many patients, but most continued to have persistent CK elevations with minimal or no symptoms.
- Lev EI, Tur-Kaspa I, Ashkenazy I, et al. Distribution of serum creatine kinase activity in young healthy persons. Clin Chim Acta 1999; 279:107–115.
- Lilleng H, Abeler K, Johnsen SH, et al. Variation of serum creatine kinase (CK) levels and prevalence of persistent hyperCKemia in a Norwegian normal population. The Tromsø Study. Neuromuscul Disord 2011; 21:494–500.
- Brewster LM, Mairuhu G, Sturk A, van Montfrans GA. Distribution of creatine kinase in the general population: implications for statin therapy. Am Heart J 2007; 154:655–661.
- Kyriakides T, Angelini C, Schaefer J, et al; European Federation of Neurological Societies. EFNS guidelines on the diagnostic approach to pauci- or asymptomatic hyperCKemia. Eur J Neurol 2010; 17:767–773.
- Prisant LM, Downton M, Watkins LO, et al. Efficacy and tolerability of lovastatin in 459 African-Americans with hypercholesterolemia. Am J Cardiol 1996; 78:420–444.
- Wong ET, Cobb C, Umehara MK, et al. Heterogeneity of serum creatine kinase activity among racial and gender groups of the population. Am J Clin Pathol 1983; 79:582–586.
- Brewster LM, de Visser M. Persistent hyperCKemia: fourteen patients studied in retrospect. Acta Neurol Scand 1988; 77:60–63.
- Weglinski MR, Wedel DJ, Engel AG. Malignant hyperthermia testing in patients with persistently increased serum creatine kinase levels. Anesth Analg 1997; 84:1038–1041.
- Galarraga B, Sinclair D, Fahie-Wilson MN, McCrae FC, Hull RG, Ledingham JM. A rare but important cause for a raised serum creatine kinase concentration: two case reports and a literature review. Rheumatology (Oxford) 2003; 42:186–188.
- Mancini GB, Tashakkor AY, Baker S, et al. Diagnosis, prevention, and management of statin adverse effects and intolerance: Canadian Working Group Consensus update. Can J Cardiol 2013; 29:1553–1568.
- Arora R, Liebo M, Maldonado F. Statin-induced myopathy: the two faces of Janus. J Cardiovasc Pharmacol Ther 2006; 11:105–112.
- Joy TR, Hegele RA. Narrative review: statin-related myopathy. Ann Intern Med 2009; 150:858–868.
- Talbert RL. Safety issues with statin therapy. J Am Pharm Assoc (2003) 2006; 46:479–490.
- SEARCH Collaborative Group; Link E, Parish S, Armitage J, et al. SLCO1B1 variants and statin-induced myopathy—a genomewide study. N Engl J Med 2008; 359:789–799.
- Wyness SP, Hunsaker JJ, La’ulu SL, Rao LV, Roberts WL. Detection of macro-creatine kinase and macroamylase by polyethylene glycol precipitation and ultrafiltration methods. Clin Chim Acta 2011; 412:2052–2057.
- Duyff RF, Van den Bosch J, Laman DM, van Loon BJ, Linssen WH. Neuromuscular findings in thyroid dysfunction: a prospective clinical and electrodiagnostic study. J Neurol Neurosurg Psychiatry 2000; 68:750–755.
- Riggs JE. Acute exertional rhabdomyolysis in hypothyroidism: the result of a reversible defect in glycogenolysis? Mil Med 1990; 155:171–172.
- Mastaglia FL, Ojeda VJ, Sarnat HB, Kakulas BA. Myopathies associated with hypothyroidism: a review based upon 13 cases. Aust N Z J Med 1988; 18:799–806.
- Alshanti M, Eledrisi MS, Jones E. Rhabdomyolysis associated with hyperthyroidism. Am J Emerg Med 2001; 19:317.
- Rosalki SB. Serum enzymes in disease of skeletal muscle. Clin Lab Med 1989; 9:767–781.
- Joy JL, Oh SJ. Asymptomatic hyper-CK-emia: an electrophysiologic and histopathologic study. Muscle Nerve 1989; 12:206–209.
- Merlini L, Sabatelli P, Columbaro M, et al. Hyper-CK-emia as the sole manifestation of myotonic dystrophy type 2. Muscle Nerve 2005; 31:764–767.
- Eeg-Olofsson O, Kalimo H, Eeg-Olofsson KE, et al. Duchenne muscular dystrophy and idiopathic hyperCKemia in the same family. Eur J Paediatr Neurol 2008; 12:404–407.
- Dwianingsih EK, Takeshima Y, Itoh K, et al. A Japanese child with asymptomatic elevation of serum creatine kinase shows PTRF-CAVIN mutation matching with congenital generalized lipodystrophy type 4. Mol Genet Metab 2010; 101:233–237.
- Carbone I, Bruno C, Sotgia F, et al. Mutation in the CAV3 gene causes partial caveolin-3 deficiency and hyperCKemia. Neurology 2000; 54:1373–1376.
- D’Adda E, Sciacco M, Fruguglietti ME, et al. Follow-up of a large population of asymptomatic/oligosymptomatic hyperckemic subjects. J Neurol 2006; 253:1399–1403.
- Fernandez C, de Paula AM, Figarella-Branger D, et al. Diagnostic evaluation of clinically normal subjects with chronic hyperCKemia. Neurology 2006; 66:1585–1587.
- Simmons Z, Peterlin BL, Boyer PJ, Towfighi J. Muscle biopsy in the evaluation of patients with modestly elevated creatine kinase levels. Muscle Nerve 2003; 27:242–244.
- Filosto M, Tonin P, Vattemi G, et al. The role of muscle biopsy in investigating isolated muscle pain. Neurology 2007; 68:181–186.
- Malandrini A, Orrico A, Gaudiano C, et al. Muscle biopsy and in vitro contracture test in subjects with idiopathic hyperCKemia. Anesthesiology 2008; 109:625–628.
- Prelle A, Tancredi L, Sciacco M, et al. Retrospective study of a large population of patients with asymptomatic or minimally symptomatic raised serum creatine kinase levels. J Neurol 2002; 249:305–311.
- Dabby R, Sadeh M, Herman O, et al. Asymptomatic or minimally symptomatic hyperCKemia: histopathologic correlates. Isr Med Assoc J 2006; 8:110–113.
- Reijneveld JC, Notermans NC, Linssen WH, Wokke JH. Benign prognosis in idiopathic hyper-CK-emia. Muscle Nerve 2000; 23:575–579.
- Restivo DA, Pavone V, Nicotra A. Single-fiber electromyography in hyperCKemia: the value of fiber density. Neurol Sci 2012; 33:819–824.
- Rowland LP, Willner J, Cerri C, DiMauro S, Miranda A. Approaches to the membrane theory of Duchenne muscular dystrophy. In: Angelini C, Danielli GA, Fontanari D, editors. Muscular Dystrophy Research: Advances and New Trends, Amsterdam: Excerpta Medica; 1980:3–13.
- Reijneveld JC, Notermans NC, Linssen WH, Bär PR, Wokke JH. Hyper-CK-aemia revisited. Neuromuscul Disord 2001; 11:163–164.
- Capasso M, De Angelis MV, Di Muzio A, et al. Familial idiopathic hyper-CK-emia: an underrecognized condition. Muscle Nerve 2006; 33:760–765.
Measuring serum creatine kinase (CK) is an important part of the evaluation of patients with muscle weakness or myalgia, and of assessing patients with myopathies or rhabdomyolysis. But elevated CK sometimes is an incidental finding in a patient without muscle-related symptoms or with only minimal nonspecific muscle symptoms (eg, cramps, spasms, fatigue) that do not significantly interfere with activities of daily living. This condition is sometimes referred to as “asymptomatic hyper-CK-emia.” Four other muscle enzymes that may also be elevated are aspartate aminotransferase, alanine aminotransferase, lactate dehydrogenase, and aldolase.
This review focuses on the evaluation of patients with elevated CK without significant muscle-related symptoms and proposes an algorithm for this purpose (Figure 1).
CURRENT THRESHOLDS MAY BE LOW
What appears to be an elevated CK level may in fact be normal, and it is important to determine in the initial assessment whether a CK value is truly abnormal.
Most laboratories use the central 95% of observations in white people as a reference range for serum CK, assuming that levels have a gaussian (bell-shaped) distribution, which is usually about 0 to 200 IU/L. Using these parameters, an abnormal CK level was observed in 19% of men and 5% of women in a study of nearly 1,000 healthy young people,1 leading to overdiagnosis.
The actual distribution of serum CK levels in a healthy population is markedly skewed toward higher values and is nongaussian.1–3 A 97.5% normal threshold is associated with a much lower false-positive rate and is recommended by the European Federation of Neurological Societies (now the European Academy of Neurology).4 This group also recommends pursuing further investigation only for patients whose level is at least 1.5 times the upper limit of normal; this threshold results in only a small reduction in sensitivity.
CK levels vary significantly by sex and race.5 Possible reasons include differences in muscle mass or total body mass and inherited differences in the permeability of the sarcolemma to CK.6 There is also a small reduction in CK levels as people age.2
The European Federation of Neurological Societies suggests redefining elevated CK as values 1.5 times beyond the upper limit of normal. Based on a 97.5% threshold and normal values determined by Brewster et al3 for black and white men and women, the following thresholds can be used to help decide whether to pursue further evaluation4:
- White women—325 IU/L
- White men—504 IU/L
- Black women—621 IU/L
- Black men—1,200 IU/L
PHYSICAL ACTIVITY RAISES CK
CK levels transiently rise after exercise or heavy manual labor. Serum CK levels may increase to as much as 30 times the upper limit of normal within 24 hours of strenuous physical activity, then slowly decline over the next 7 days. The degree of CK elevation depends on the type and duration of exercise, with greater elevation in those who are untrained.2,4
In assessing asymptomatic or minimally symptomatic CK elevation, the test should be repeated after 7 days without exercise. A large community study in Norway found that repeat CK levels in people with incidentally discovered elevated CK were normal after 3 days of rest in 70% of cases.2
NONNEUROMUSCULAR CAUSES
NEED TO BE INVESTIGATED
Asymptomatic or minimally symptomatic elevated CK can be due to a primary neuromuscular disease or a variety of nonneuromuscular causes.
Patients who still have elevated CK after taking into account the 97.5% threshold, repeat testing after a week of rest, and a level more than 1.5 times the upper limit of normal for sex and race should first be evaluated for the many nonneuromuscular conditions that can cause elevated CK (Table 1).7–9
Cardiac causes should be evaluated by history and physical examination, electrocardiography, and possibly testing for cardiac troponins.
Drugs commonly elevate CK
Prescription drugs and supplements are an important and common cause of CK elevation, so it is important to carefully review medications the patient is taking.
Statins can cause myalgia, muscle weakness, and rhabdomyolysis. Up to 5% of users develop CK elevation, typically 2 to 10 times the upper limit of normal.10 CK usually drops after stopping statins but may require weeks to months to normalize. Rarely, statin users develop a serious immune-mediated necrotizing myopathy.11–13
The diversity of response to statin therapy appears to have a genetic basis. The SEARCH Collaborative Group14 conducted a genome-wide association study of 300,000 markers in 85 patients with definite or incipient myopathy and in 90 controls, all of whom were taking simvastatin 80 mg daily. They identified a single-nucleotide polymorphism in the SLCO1B1 gene on chromosome 12 that was strongly associated with a higher risk of statin-induced myopathy.
Patients with statin-related myopathy seem to have a higher frequency of occult metabolic muscle disease than the general population, also suggesting genetic susceptibility, although ascertainment bias could be a factor.14
Mechanisms of CK elevation in response to statins include increased muscle membrane fragility due to decreased cholesterol content, inhibition of isoprenoid production (a necessary step in the synthesis of membrane proteins), and depletion of ubiquinone, leading to mitochondrial dysfunction.
Macro CK: An abnormal enzyme complex
About 4% of patients with asymptomatic or minimally symptomatic elevated CK have “macro CK,” an enzyme complex with an atypically high molecular mass and reduced clearance, resulting in abnormally high blood levels of CK. Macro CK type 1 is more common and is found in up to 1.2% of the general population: complexes are composed of CK and immunoglobulin and are associated with autoimmune diseases.9,15 Macro CK type 2 complexes consist of CK and an undetermined protein and are associated with malignancies.
CK electrophoresis is required to detect macro CK. Types 1 and 2 can be distinguished by protein G affinity chromatography.9,15
Endocrine disorders
Muscle involvement in endocrine disorders often presents with muscle weakness in addition to muscle enzyme abnormalities.
Hypothyroidism often causes weakness, cramps, myalgia, and a mild to moderate serum CK elevation.16 Severe CK elevation has been reported to occur after vigorous exercise.17 Thyroid replacement usually results in normalization of serum CK levels in 1 to 2 months.18
Hyperthyroidism is typically associated with normal serum CK concentrations, but in rare cases it can cause rhabdomyolysis.19
NEUROMUSCULAR CAUSES ARE NOT ALWAYS WORTH PURSUING
Only after the nonneuromuscular causes of elevated CK have been ruled out should neuromuscular disorders be considered (Table 2). Evaluation with electromyography, nerve conduction studies, and muscle biopsy may lead to the diagnosis of a specific neuromuscular disorder: patients may be in the presymptomatic stage of disease and may or may not eventually develop muscle weakness or other symptoms.20,21
Is testing needed?
Most adult dystrophies and metabolic myopathies have no available treatment and their course is often benign, particularly if they present only with asymptomatic elevated CK. The value of a potentially extensive, expensive, and invasive evaluation for a specific neuromuscular cause should be weighed against the limited yield and treatment options. Moreover, specialized testing such as biochemical muscle enzyme analysis, sarcolemmal protein staining, and genetic testing are not available at all centers.
The European Federation of Neurological Societies guidelines recommend biopsy for patients with asymptomatic elevated CK who also have any of the following:
- Abnormal (myopathic) findings on electromyography
- CK more than three times the upper limit of normal
- Age less than 25
- Exercise intolerance.4
Idiopathic inflammatory myopathies rarely present with asymptomatic elevated CK.22–26 In one study,27 they were found in just 5% of patients with asymptomatic elevated CK.
Hypomyopathic dermatomyositis and inclusion body myositis can present with mild CK elevations with normal muscle strength, especially early in the disease course. A myositis subset of antisynthetase syndrome can present with mildly elevated CK and interstitial lung disease.27 Many of the inflammatory myopathies respond to treatment so are worth investigating.
In view of complexities in diagnosis of these conditions, one should proceed with testing only after discussing it with patients. Referral to a rheumatology specialist is preferred.
MUSCLE BIOPSY, ELECTROMYOGRAPHY, AND NERVE CONDUCTION STUDIES
Electromyography, nerve conduction studies, or muscle biopsy, or a combination of these tests, is usually needed to investigate neuromuscular causes of elevated CK.
Muscle biopsy abnormalities are found in about two-thirds of cases of asymptomatic elevated CK, but most abnormalities include nonspecific myopathic changes that are not diagnostic. A muscle biopsy that may include special stains for sarcolemmal proteins for muscular dystrophy and biochemical muscle enzyme analysis for metabolic myopathies is diagnostic in only 20% to 25% cases of asymptomatic elevated CK on average, with a variation between different series of 0% to 79%.7,21,27–33
Electromyography and nerve conduction studies alone add little to the workup of asymptomatic elevated CK apart from a modest negative predictive value and as a guide for muscle biopsy. For a very few neuromuscular disorders causing an elevated CK (eg, motor neuron disease, Charcot-Marie-Tooth disease, myotonic dystrophy), electromyography and nerve conduction studies could suffice to make the diagnosis.
Electromyography and nerve conduction studies detect abnormalities in nearly half of cases of asymptomatic CK elevation,7,21,27,28,30,31,33 but, as with biopsy, most changes are nonspecific. Although electromyography and nerve conduction studies can help distinguish primary neuropathic from myopathic disorders, the sensitivity and specificity are low for diagnosis. Normal studies do not rule out a condition, and abnormal studies are not diagnostic of a particular condition, although completely normal studies provide strong evidence against a severe neuromuscular disorder.
Combined testing
Using combined muscle biopsy, electromyography, and nerve conduction studies, the likelihood of making a diagnosis in patients with asymptomatic elevated CK is 28% on average (range of studies 4%–79%),2,7,21,26–28,30–32 and findings are nonspecific in 30% to 40% of cases. Findings are normal in about 30% to 40% of cases, which are thus diagnosed as idiopathic asymptomatic elevated CK.28–31,34
Prelle et al31 retrospectively reviewed the cases of 114 patients, ages 3 to 70, with incidentally discovered elevated CK and few or no symptoms, who underwent muscle biopsy, electromyography, and nerve conduction studies after nonneuromuscular causes were ruled out. Although muscle biopsy findings were abnormal in 39% of cases, a diagnosis was established in only 18% of cases after an extensive workup: the diagnosis was definitive in only 10% and included dystrophinopathies, metabolic myopathies, and rare noninflammatory myopathies. For the remaining 8%, the diagnosis was probable and included four cases of partial carnitine palmitoyl transferase deficiency, three cases of malignant hyperthermia, and two rare inherited disorders.
DNA testing
In women with a serum CK less than three times the upper limit of normal who have a family history of Duchenne or Becker muscular dystrophy, DNA analysis of blood lymphocytes identifies 70% of carriers.4
IDIOPATHIC ELEVATED SERUM CK
Rowland et al35 first coined the term “idiopathic hyper-CK-emia” and defined it as persistent elevation of serum CK despite a normal neurologic examination and testing, including electromyography, nerve conduction studies, and muscle biopsy.35,36 To receive this diagnosis, patients must also have no family history or clinical evidence of neuromuscular disease.
Idiopathic elevated serum CK is sometimes familial. In one study,37 elevated CK was found in family members of 13 of 28 unrelated probands. In the 13 families, 41 individuals had elevated CK. Genetic studies revealed that the condition is genetically heterogeneous and autosomal dominant in at least 60% of cases, with higher penetrance in men.
D’Adda et al26 followed 55 people with idiopathic elevated CK for 7 years. Ten percent were eventually diagnosed with a neuromuscular disorder, 10% developed malignancy, and the remaining 80% developed no new condition. The CK level normalized or decreased in many patients, but most continued to have persistent CK elevations with minimal or no symptoms.
Measuring serum creatine kinase (CK) is an important part of the evaluation of patients with muscle weakness or myalgia, and of assessing patients with myopathies or rhabdomyolysis. But elevated CK sometimes is an incidental finding in a patient without muscle-related symptoms or with only minimal nonspecific muscle symptoms (eg, cramps, spasms, fatigue) that do not significantly interfere with activities of daily living. This condition is sometimes referred to as “asymptomatic hyper-CK-emia.” Four other muscle enzymes that may also be elevated are aspartate aminotransferase, alanine aminotransferase, lactate dehydrogenase, and aldolase.
This review focuses on the evaluation of patients with elevated CK without significant muscle-related symptoms and proposes an algorithm for this purpose (Figure 1).
CURRENT THRESHOLDS MAY BE LOW
What appears to be an elevated CK level may in fact be normal, and it is important to determine in the initial assessment whether a CK value is truly abnormal.
Most laboratories use the central 95% of observations in white people as a reference range for serum CK, assuming that levels have a gaussian (bell-shaped) distribution, which is usually about 0 to 200 IU/L. Using these parameters, an abnormal CK level was observed in 19% of men and 5% of women in a study of nearly 1,000 healthy young people,1 leading to overdiagnosis.
The actual distribution of serum CK levels in a healthy population is markedly skewed toward higher values and is nongaussian.1–3 A 97.5% normal threshold is associated with a much lower false-positive rate and is recommended by the European Federation of Neurological Societies (now the European Academy of Neurology).4 This group also recommends pursuing further investigation only for patients whose level is at least 1.5 times the upper limit of normal; this threshold results in only a small reduction in sensitivity.
CK levels vary significantly by sex and race.5 Possible reasons include differences in muscle mass or total body mass and inherited differences in the permeability of the sarcolemma to CK.6 There is also a small reduction in CK levels as people age.2
The European Federation of Neurological Societies suggests redefining elevated CK as values 1.5 times beyond the upper limit of normal. Based on a 97.5% threshold and normal values determined by Brewster et al3 for black and white men and women, the following thresholds can be used to help decide whether to pursue further evaluation4:
- White women—325 IU/L
- White men—504 IU/L
- Black women—621 IU/L
- Black men—1,200 IU/L
PHYSICAL ACTIVITY RAISES CK
CK levels transiently rise after exercise or heavy manual labor. Serum CK levels may increase to as much as 30 times the upper limit of normal within 24 hours of strenuous physical activity, then slowly decline over the next 7 days. The degree of CK elevation depends on the type and duration of exercise, with greater elevation in those who are untrained.2,4
In assessing asymptomatic or minimally symptomatic CK elevation, the test should be repeated after 7 days without exercise. A large community study in Norway found that repeat CK levels in people with incidentally discovered elevated CK were normal after 3 days of rest in 70% of cases.2
NONNEUROMUSCULAR CAUSES
NEED TO BE INVESTIGATED
Asymptomatic or minimally symptomatic elevated CK can be due to a primary neuromuscular disease or a variety of nonneuromuscular causes.
Patients who still have elevated CK after taking into account the 97.5% threshold, repeat testing after a week of rest, and a level more than 1.5 times the upper limit of normal for sex and race should first be evaluated for the many nonneuromuscular conditions that can cause elevated CK (Table 1).7–9
Cardiac causes should be evaluated by history and physical examination, electrocardiography, and possibly testing for cardiac troponins.
Drugs commonly elevate CK
Prescription drugs and supplements are an important and common cause of CK elevation, so it is important to carefully review medications the patient is taking.
Statins can cause myalgia, muscle weakness, and rhabdomyolysis. Up to 5% of users develop CK elevation, typically 2 to 10 times the upper limit of normal.10 CK usually drops after stopping statins but may require weeks to months to normalize. Rarely, statin users develop a serious immune-mediated necrotizing myopathy.11–13
The diversity of response to statin therapy appears to have a genetic basis. The SEARCH Collaborative Group14 conducted a genome-wide association study of 300,000 markers in 85 patients with definite or incipient myopathy and in 90 controls, all of whom were taking simvastatin 80 mg daily. They identified a single-nucleotide polymorphism in the SLCO1B1 gene on chromosome 12 that was strongly associated with a higher risk of statin-induced myopathy.
Patients with statin-related myopathy seem to have a higher frequency of occult metabolic muscle disease than the general population, also suggesting genetic susceptibility, although ascertainment bias could be a factor.14
Mechanisms of CK elevation in response to statins include increased muscle membrane fragility due to decreased cholesterol content, inhibition of isoprenoid production (a necessary step in the synthesis of membrane proteins), and depletion of ubiquinone, leading to mitochondrial dysfunction.
Macro CK: An abnormal enzyme complex
About 4% of patients with asymptomatic or minimally symptomatic elevated CK have “macro CK,” an enzyme complex with an atypically high molecular mass and reduced clearance, resulting in abnormally high blood levels of CK. Macro CK type 1 is more common and is found in up to 1.2% of the general population: complexes are composed of CK and immunoglobulin and are associated with autoimmune diseases.9,15 Macro CK type 2 complexes consist of CK and an undetermined protein and are associated with malignancies.
CK electrophoresis is required to detect macro CK. Types 1 and 2 can be distinguished by protein G affinity chromatography.9,15
Endocrine disorders
Muscle involvement in endocrine disorders often presents with muscle weakness in addition to muscle enzyme abnormalities.
Hypothyroidism often causes weakness, cramps, myalgia, and a mild to moderate serum CK elevation.16 Severe CK elevation has been reported to occur after vigorous exercise.17 Thyroid replacement usually results in normalization of serum CK levels in 1 to 2 months.18
Hyperthyroidism is typically associated with normal serum CK concentrations, but in rare cases it can cause rhabdomyolysis.19
NEUROMUSCULAR CAUSES ARE NOT ALWAYS WORTH PURSUING
Only after the nonneuromuscular causes of elevated CK have been ruled out should neuromuscular disorders be considered (Table 2). Evaluation with electromyography, nerve conduction studies, and muscle biopsy may lead to the diagnosis of a specific neuromuscular disorder: patients may be in the presymptomatic stage of disease and may or may not eventually develop muscle weakness or other symptoms.20,21
Is testing needed?
Most adult dystrophies and metabolic myopathies have no available treatment and their course is often benign, particularly if they present only with asymptomatic elevated CK. The value of a potentially extensive, expensive, and invasive evaluation for a specific neuromuscular cause should be weighed against the limited yield and treatment options. Moreover, specialized testing such as biochemical muscle enzyme analysis, sarcolemmal protein staining, and genetic testing are not available at all centers.
The European Federation of Neurological Societies guidelines recommend biopsy for patients with asymptomatic elevated CK who also have any of the following:
- Abnormal (myopathic) findings on electromyography
- CK more than three times the upper limit of normal
- Age less than 25
- Exercise intolerance.4
Idiopathic inflammatory myopathies rarely present with asymptomatic elevated CK.22–26 In one study,27 they were found in just 5% of patients with asymptomatic elevated CK.
Hypomyopathic dermatomyositis and inclusion body myositis can present with mild CK elevations with normal muscle strength, especially early in the disease course. A myositis subset of antisynthetase syndrome can present with mildly elevated CK and interstitial lung disease.27 Many of the inflammatory myopathies respond to treatment so are worth investigating.
In view of complexities in diagnosis of these conditions, one should proceed with testing only after discussing it with patients. Referral to a rheumatology specialist is preferred.
MUSCLE BIOPSY, ELECTROMYOGRAPHY, AND NERVE CONDUCTION STUDIES
Electromyography, nerve conduction studies, or muscle biopsy, or a combination of these tests, is usually needed to investigate neuromuscular causes of elevated CK.
Muscle biopsy abnormalities are found in about two-thirds of cases of asymptomatic elevated CK, but most abnormalities include nonspecific myopathic changes that are not diagnostic. A muscle biopsy that may include special stains for sarcolemmal proteins for muscular dystrophy and biochemical muscle enzyme analysis for metabolic myopathies is diagnostic in only 20% to 25% cases of asymptomatic elevated CK on average, with a variation between different series of 0% to 79%.7,21,27–33
Electromyography and nerve conduction studies alone add little to the workup of asymptomatic elevated CK apart from a modest negative predictive value and as a guide for muscle biopsy. For a very few neuromuscular disorders causing an elevated CK (eg, motor neuron disease, Charcot-Marie-Tooth disease, myotonic dystrophy), electromyography and nerve conduction studies could suffice to make the diagnosis.
Electromyography and nerve conduction studies detect abnormalities in nearly half of cases of asymptomatic CK elevation,7,21,27,28,30,31,33 but, as with biopsy, most changes are nonspecific. Although electromyography and nerve conduction studies can help distinguish primary neuropathic from myopathic disorders, the sensitivity and specificity are low for diagnosis. Normal studies do not rule out a condition, and abnormal studies are not diagnostic of a particular condition, although completely normal studies provide strong evidence against a severe neuromuscular disorder.
Combined testing
Using combined muscle biopsy, electromyography, and nerve conduction studies, the likelihood of making a diagnosis in patients with asymptomatic elevated CK is 28% on average (range of studies 4%–79%),2,7,21,26–28,30–32 and findings are nonspecific in 30% to 40% of cases. Findings are normal in about 30% to 40% of cases, which are thus diagnosed as idiopathic asymptomatic elevated CK.28–31,34
Prelle et al31 retrospectively reviewed the cases of 114 patients, ages 3 to 70, with incidentally discovered elevated CK and few or no symptoms, who underwent muscle biopsy, electromyography, and nerve conduction studies after nonneuromuscular causes were ruled out. Although muscle biopsy findings were abnormal in 39% of cases, a diagnosis was established in only 18% of cases after an extensive workup: the diagnosis was definitive in only 10% and included dystrophinopathies, metabolic myopathies, and rare noninflammatory myopathies. For the remaining 8%, the diagnosis was probable and included four cases of partial carnitine palmitoyl transferase deficiency, three cases of malignant hyperthermia, and two rare inherited disorders.
DNA testing
In women with a serum CK less than three times the upper limit of normal who have a family history of Duchenne or Becker muscular dystrophy, DNA analysis of blood lymphocytes identifies 70% of carriers.4
IDIOPATHIC ELEVATED SERUM CK
Rowland et al35 first coined the term “idiopathic hyper-CK-emia” and defined it as persistent elevation of serum CK despite a normal neurologic examination and testing, including electromyography, nerve conduction studies, and muscle biopsy.35,36 To receive this diagnosis, patients must also have no family history or clinical evidence of neuromuscular disease.
Idiopathic elevated serum CK is sometimes familial. In one study,37 elevated CK was found in family members of 13 of 28 unrelated probands. In the 13 families, 41 individuals had elevated CK. Genetic studies revealed that the condition is genetically heterogeneous and autosomal dominant in at least 60% of cases, with higher penetrance in men.
D’Adda et al26 followed 55 people with idiopathic elevated CK for 7 years. Ten percent were eventually diagnosed with a neuromuscular disorder, 10% developed malignancy, and the remaining 80% developed no new condition. The CK level normalized or decreased in many patients, but most continued to have persistent CK elevations with minimal or no symptoms.
- Lev EI, Tur-Kaspa I, Ashkenazy I, et al. Distribution of serum creatine kinase activity in young healthy persons. Clin Chim Acta 1999; 279:107–115.
- Lilleng H, Abeler K, Johnsen SH, et al. Variation of serum creatine kinase (CK) levels and prevalence of persistent hyperCKemia in a Norwegian normal population. The Tromsø Study. Neuromuscul Disord 2011; 21:494–500.
- Brewster LM, Mairuhu G, Sturk A, van Montfrans GA. Distribution of creatine kinase in the general population: implications for statin therapy. Am Heart J 2007; 154:655–661.
- Kyriakides T, Angelini C, Schaefer J, et al; European Federation of Neurological Societies. EFNS guidelines on the diagnostic approach to pauci- or asymptomatic hyperCKemia. Eur J Neurol 2010; 17:767–773.
- Prisant LM, Downton M, Watkins LO, et al. Efficacy and tolerability of lovastatin in 459 African-Americans with hypercholesterolemia. Am J Cardiol 1996; 78:420–444.
- Wong ET, Cobb C, Umehara MK, et al. Heterogeneity of serum creatine kinase activity among racial and gender groups of the population. Am J Clin Pathol 1983; 79:582–586.
- Brewster LM, de Visser M. Persistent hyperCKemia: fourteen patients studied in retrospect. Acta Neurol Scand 1988; 77:60–63.
- Weglinski MR, Wedel DJ, Engel AG. Malignant hyperthermia testing in patients with persistently increased serum creatine kinase levels. Anesth Analg 1997; 84:1038–1041.
- Galarraga B, Sinclair D, Fahie-Wilson MN, McCrae FC, Hull RG, Ledingham JM. A rare but important cause for a raised serum creatine kinase concentration: two case reports and a literature review. Rheumatology (Oxford) 2003; 42:186–188.
- Mancini GB, Tashakkor AY, Baker S, et al. Diagnosis, prevention, and management of statin adverse effects and intolerance: Canadian Working Group Consensus update. Can J Cardiol 2013; 29:1553–1568.
- Arora R, Liebo M, Maldonado F. Statin-induced myopathy: the two faces of Janus. J Cardiovasc Pharmacol Ther 2006; 11:105–112.
- Joy TR, Hegele RA. Narrative review: statin-related myopathy. Ann Intern Med 2009; 150:858–868.
- Talbert RL. Safety issues with statin therapy. J Am Pharm Assoc (2003) 2006; 46:479–490.
- SEARCH Collaborative Group; Link E, Parish S, Armitage J, et al. SLCO1B1 variants and statin-induced myopathy—a genomewide study. N Engl J Med 2008; 359:789–799.
- Wyness SP, Hunsaker JJ, La’ulu SL, Rao LV, Roberts WL. Detection of macro-creatine kinase and macroamylase by polyethylene glycol precipitation and ultrafiltration methods. Clin Chim Acta 2011; 412:2052–2057.
- Duyff RF, Van den Bosch J, Laman DM, van Loon BJ, Linssen WH. Neuromuscular findings in thyroid dysfunction: a prospective clinical and electrodiagnostic study. J Neurol Neurosurg Psychiatry 2000; 68:750–755.
- Riggs JE. Acute exertional rhabdomyolysis in hypothyroidism: the result of a reversible defect in glycogenolysis? Mil Med 1990; 155:171–172.
- Mastaglia FL, Ojeda VJ, Sarnat HB, Kakulas BA. Myopathies associated with hypothyroidism: a review based upon 13 cases. Aust N Z J Med 1988; 18:799–806.
- Alshanti M, Eledrisi MS, Jones E. Rhabdomyolysis associated with hyperthyroidism. Am J Emerg Med 2001; 19:317.
- Rosalki SB. Serum enzymes in disease of skeletal muscle. Clin Lab Med 1989; 9:767–781.
- Joy JL, Oh SJ. Asymptomatic hyper-CK-emia: an electrophysiologic and histopathologic study. Muscle Nerve 1989; 12:206–209.
- Merlini L, Sabatelli P, Columbaro M, et al. Hyper-CK-emia as the sole manifestation of myotonic dystrophy type 2. Muscle Nerve 2005; 31:764–767.
- Eeg-Olofsson O, Kalimo H, Eeg-Olofsson KE, et al. Duchenne muscular dystrophy and idiopathic hyperCKemia in the same family. Eur J Paediatr Neurol 2008; 12:404–407.
- Dwianingsih EK, Takeshima Y, Itoh K, et al. A Japanese child with asymptomatic elevation of serum creatine kinase shows PTRF-CAVIN mutation matching with congenital generalized lipodystrophy type 4. Mol Genet Metab 2010; 101:233–237.
- Carbone I, Bruno C, Sotgia F, et al. Mutation in the CAV3 gene causes partial caveolin-3 deficiency and hyperCKemia. Neurology 2000; 54:1373–1376.
- D’Adda E, Sciacco M, Fruguglietti ME, et al. Follow-up of a large population of asymptomatic/oligosymptomatic hyperckemic subjects. J Neurol 2006; 253:1399–1403.
- Fernandez C, de Paula AM, Figarella-Branger D, et al. Diagnostic evaluation of clinically normal subjects with chronic hyperCKemia. Neurology 2006; 66:1585–1587.
- Simmons Z, Peterlin BL, Boyer PJ, Towfighi J. Muscle biopsy in the evaluation of patients with modestly elevated creatine kinase levels. Muscle Nerve 2003; 27:242–244.
- Filosto M, Tonin P, Vattemi G, et al. The role of muscle biopsy in investigating isolated muscle pain. Neurology 2007; 68:181–186.
- Malandrini A, Orrico A, Gaudiano C, et al. Muscle biopsy and in vitro contracture test in subjects with idiopathic hyperCKemia. Anesthesiology 2008; 109:625–628.
- Prelle A, Tancredi L, Sciacco M, et al. Retrospective study of a large population of patients with asymptomatic or minimally symptomatic raised serum creatine kinase levels. J Neurol 2002; 249:305–311.
- Dabby R, Sadeh M, Herman O, et al. Asymptomatic or minimally symptomatic hyperCKemia: histopathologic correlates. Isr Med Assoc J 2006; 8:110–113.
- Reijneveld JC, Notermans NC, Linssen WH, Wokke JH. Benign prognosis in idiopathic hyper-CK-emia. Muscle Nerve 2000; 23:575–579.
- Restivo DA, Pavone V, Nicotra A. Single-fiber electromyography in hyperCKemia: the value of fiber density. Neurol Sci 2012; 33:819–824.
- Rowland LP, Willner J, Cerri C, DiMauro S, Miranda A. Approaches to the membrane theory of Duchenne muscular dystrophy. In: Angelini C, Danielli GA, Fontanari D, editors. Muscular Dystrophy Research: Advances and New Trends, Amsterdam: Excerpta Medica; 1980:3–13.
- Reijneveld JC, Notermans NC, Linssen WH, Bär PR, Wokke JH. Hyper-CK-aemia revisited. Neuromuscul Disord 2001; 11:163–164.
- Capasso M, De Angelis MV, Di Muzio A, et al. Familial idiopathic hyper-CK-emia: an underrecognized condition. Muscle Nerve 2006; 33:760–765.
- Lev EI, Tur-Kaspa I, Ashkenazy I, et al. Distribution of serum creatine kinase activity in young healthy persons. Clin Chim Acta 1999; 279:107–115.
- Lilleng H, Abeler K, Johnsen SH, et al. Variation of serum creatine kinase (CK) levels and prevalence of persistent hyperCKemia in a Norwegian normal population. The Tromsø Study. Neuromuscul Disord 2011; 21:494–500.
- Brewster LM, Mairuhu G, Sturk A, van Montfrans GA. Distribution of creatine kinase in the general population: implications for statin therapy. Am Heart J 2007; 154:655–661.
- Kyriakides T, Angelini C, Schaefer J, et al; European Federation of Neurological Societies. EFNS guidelines on the diagnostic approach to pauci- or asymptomatic hyperCKemia. Eur J Neurol 2010; 17:767–773.
- Prisant LM, Downton M, Watkins LO, et al. Efficacy and tolerability of lovastatin in 459 African-Americans with hypercholesterolemia. Am J Cardiol 1996; 78:420–444.
- Wong ET, Cobb C, Umehara MK, et al. Heterogeneity of serum creatine kinase activity among racial and gender groups of the population. Am J Clin Pathol 1983; 79:582–586.
- Brewster LM, de Visser M. Persistent hyperCKemia: fourteen patients studied in retrospect. Acta Neurol Scand 1988; 77:60–63.
- Weglinski MR, Wedel DJ, Engel AG. Malignant hyperthermia testing in patients with persistently increased serum creatine kinase levels. Anesth Analg 1997; 84:1038–1041.
- Galarraga B, Sinclair D, Fahie-Wilson MN, McCrae FC, Hull RG, Ledingham JM. A rare but important cause for a raised serum creatine kinase concentration: two case reports and a literature review. Rheumatology (Oxford) 2003; 42:186–188.
- Mancini GB, Tashakkor AY, Baker S, et al. Diagnosis, prevention, and management of statin adverse effects and intolerance: Canadian Working Group Consensus update. Can J Cardiol 2013; 29:1553–1568.
- Arora R, Liebo M, Maldonado F. Statin-induced myopathy: the two faces of Janus. J Cardiovasc Pharmacol Ther 2006; 11:105–112.
- Joy TR, Hegele RA. Narrative review: statin-related myopathy. Ann Intern Med 2009; 150:858–868.
- Talbert RL. Safety issues with statin therapy. J Am Pharm Assoc (2003) 2006; 46:479–490.
- SEARCH Collaborative Group; Link E, Parish S, Armitage J, et al. SLCO1B1 variants and statin-induced myopathy—a genomewide study. N Engl J Med 2008; 359:789–799.
- Wyness SP, Hunsaker JJ, La’ulu SL, Rao LV, Roberts WL. Detection of macro-creatine kinase and macroamylase by polyethylene glycol precipitation and ultrafiltration methods. Clin Chim Acta 2011; 412:2052–2057.
- Duyff RF, Van den Bosch J, Laman DM, van Loon BJ, Linssen WH. Neuromuscular findings in thyroid dysfunction: a prospective clinical and electrodiagnostic study. J Neurol Neurosurg Psychiatry 2000; 68:750–755.
- Riggs JE. Acute exertional rhabdomyolysis in hypothyroidism: the result of a reversible defect in glycogenolysis? Mil Med 1990; 155:171–172.
- Mastaglia FL, Ojeda VJ, Sarnat HB, Kakulas BA. Myopathies associated with hypothyroidism: a review based upon 13 cases. Aust N Z J Med 1988; 18:799–806.
- Alshanti M, Eledrisi MS, Jones E. Rhabdomyolysis associated with hyperthyroidism. Am J Emerg Med 2001; 19:317.
- Rosalki SB. Serum enzymes in disease of skeletal muscle. Clin Lab Med 1989; 9:767–781.
- Joy JL, Oh SJ. Asymptomatic hyper-CK-emia: an electrophysiologic and histopathologic study. Muscle Nerve 1989; 12:206–209.
- Merlini L, Sabatelli P, Columbaro M, et al. Hyper-CK-emia as the sole manifestation of myotonic dystrophy type 2. Muscle Nerve 2005; 31:764–767.
- Eeg-Olofsson O, Kalimo H, Eeg-Olofsson KE, et al. Duchenne muscular dystrophy and idiopathic hyperCKemia in the same family. Eur J Paediatr Neurol 2008; 12:404–407.
- Dwianingsih EK, Takeshima Y, Itoh K, et al. A Japanese child with asymptomatic elevation of serum creatine kinase shows PTRF-CAVIN mutation matching with congenital generalized lipodystrophy type 4. Mol Genet Metab 2010; 101:233–237.
- Carbone I, Bruno C, Sotgia F, et al. Mutation in the CAV3 gene causes partial caveolin-3 deficiency and hyperCKemia. Neurology 2000; 54:1373–1376.
- D’Adda E, Sciacco M, Fruguglietti ME, et al. Follow-up of a large population of asymptomatic/oligosymptomatic hyperckemic subjects. J Neurol 2006; 253:1399–1403.
- Fernandez C, de Paula AM, Figarella-Branger D, et al. Diagnostic evaluation of clinically normal subjects with chronic hyperCKemia. Neurology 2006; 66:1585–1587.
- Simmons Z, Peterlin BL, Boyer PJ, Towfighi J. Muscle biopsy in the evaluation of patients with modestly elevated creatine kinase levels. Muscle Nerve 2003; 27:242–244.
- Filosto M, Tonin P, Vattemi G, et al. The role of muscle biopsy in investigating isolated muscle pain. Neurology 2007; 68:181–186.
- Malandrini A, Orrico A, Gaudiano C, et al. Muscle biopsy and in vitro contracture test in subjects with idiopathic hyperCKemia. Anesthesiology 2008; 109:625–628.
- Prelle A, Tancredi L, Sciacco M, et al. Retrospective study of a large population of patients with asymptomatic or minimally symptomatic raised serum creatine kinase levels. J Neurol 2002; 249:305–311.
- Dabby R, Sadeh M, Herman O, et al. Asymptomatic or minimally symptomatic hyperCKemia: histopathologic correlates. Isr Med Assoc J 2006; 8:110–113.
- Reijneveld JC, Notermans NC, Linssen WH, Wokke JH. Benign prognosis in idiopathic hyper-CK-emia. Muscle Nerve 2000; 23:575–579.
- Restivo DA, Pavone V, Nicotra A. Single-fiber electromyography in hyperCKemia: the value of fiber density. Neurol Sci 2012; 33:819–824.
- Rowland LP, Willner J, Cerri C, DiMauro S, Miranda A. Approaches to the membrane theory of Duchenne muscular dystrophy. In: Angelini C, Danielli GA, Fontanari D, editors. Muscular Dystrophy Research: Advances and New Trends, Amsterdam: Excerpta Medica; 1980:3–13.
- Reijneveld JC, Notermans NC, Linssen WH, Bär PR, Wokke JH. Hyper-CK-aemia revisited. Neuromuscul Disord 2001; 11:163–164.
- Capasso M, De Angelis MV, Di Muzio A, et al. Familial idiopathic hyper-CK-emia: an underrecognized condition. Muscle Nerve 2006; 33:760–765.
KEY POINTS
- Standard reference ranges for serum CK levels used by most laboratories are too low and lead to overdiagnosis of abnormal values.
- Serum CK levels are strongly affected by race, sex, and physical activity.
- A patient with truly elevated levels should be evaluated for a variety of nonneuromuscular causes, including endocrine disorders, metabolic disturbances, drug effects, and malignancy.
- Neuromuscular causes should be investigated only after ruling out nonneuromuscular causes and after considering whether potential benefits of a diagnosis outweigh the risks and expense of extensive testing.
Autoantibody-mediated encephalitis: Not just paraneoplastic, not just limbic, and not untreatable
A 79-year-old woman with a history of breast cancer in remission and hypertension presented to a local emergency department because of subacute memory loss and compulsive shopping. Her serum sodium concentration was 127 mmol/L (reference range 132–148). Computed tomography (CT) and magnetic resonance imaging (MRI) of the brain were normal, and she was sent home.
Three days later, she experienced a generalized tonic-clonic seizure that evolved into status epilepticus. She was intubated and admitted to the intensive care unit. Cerebrospinal fluid analysis was normal, and infectious causes of encephalitis were ruled out. MRI showed increased signal in both hippocampi (Figure 1). Her seizures were refractory to treatment, and she was given pentobarbital to induce a coma.
Serum evaluation of neuronal antibodies revealed elevated titers of the voltage-gated potassium channel (VGKC) complex antibody, with subsequent subtyping confirming the leucine-rich glioma-inactivated protein 1 (LGI1) protein as the antigenic target.
She received a 5-day course of intravenous immunoglobulin and methylprednisolone, pentobarbital was withdrawn, and the seizures did not recur, but weeks later she remained comatose. Positron emission tomography (PET) of the brain revealed hypermetabolism in the medial and anterior aspects of both temporal lobes. She underwent five sessions of plasma exchange, after which she began to improve and follow commands. She was ultimately discharged to an acute rehabilitation facility after a 4-week hospital stay.
She received infusions of intravenous immunoglobulin twice a month for 6 months. At her last follow-up visit, she was seizure-free and neurologically intact except for mild inattention.
NEWLY RECOGNIZED DISEASES
Although autoantibody-mediated encephalitic syndromes were first described more than 50 years ago,1,2 their autoimmune basis was not recognized until the early 1980s.3 In the past 10 years, a flood of novel clinical syndromes associated with neuronal autoantibodies has been described that may be markedly improved or even completely resolved with immunotherapy. In cases of unexplained seizure, encephalitis, or acute-onset psychiatric syndromes, suspecting these syndromes can lead to diagnosis, treatment, and a good outcome.
This review describes the key clinical autoantibody-mediated encephalitic syndromes, explains the better-characterized antibody associations, and discusses their diagnosis and treatment.
CLASSIFIED ANATOMICALLY, IMMUNOLOGICALLY, OR EPONYMOUSLY
Autoantibody-mediated encephalitis is also known as autoimmune-mediated encephalitis, autoimmune-mediated limbic encephalitis, and autoimmune synaptic encephalitis.
How to categorize these syndromes is still in flux: they can be listed by the area of the brain affected, the antibody involved, or the name of the discoverer (eg, Morvan syndrome).
Autoantibodies identified in autoimmune encephalitis fall under two broad categories:
- Those targeting intracellular (intranuclear or intracytoplasmic) antigens; the syndromes they cause are more likely to be paraneoplastic and less responsive to immunotherapy
- Those targeting antigens on the neuronal surface: the syndromes they cause are less likely to be paraneoplastic and are more responsive to immunotherapy.4
SYNDROMES DEFINED BY BRAIN AREA AFFECTED
Below, we provide examples of neurologic syndromes of autoantibody-mediated encephalitis according to the region of the brain most affected, ie, the limbic system, the brainstem, or the cerebellum (Figure 2).
LIMBIC ENCEPHALITIS
Memory loss, behavioral changes, seizures
Patients with limbic encephalitis (such as the patient described in the vignette above) present with symptoms attributed to dysfunction of mesial temporal lobe structures, most notably the hippocampus. Prominent symptoms include short-term memory loss, behavioral disturbances such as agitation and confusion, and psychiatric problems such as depression and psychosis. Recurrent seizures are a salient feature and, not uncommonly, progress to status epilepticus.
Antibodies are not all cancer-associated
Cerebrospinal fluid analysis can be normal or show abnormalities suggesting immune activation, eg, slight pleocytosis, elevated protein, increased immunoglobulin G synthesis, and oligoclonal banding.5
In many cases, an autoantibody is found in the blood or in the cerebrospinal fluid. Some patients may express more than one autoantibody, so the traditional view of “one antibody, one syndrome” is incorrect.
Although initially identified as a rare paraneoplastic disorder, limbic encephalitis sometimes occurs in the absence of malignancy.
Multiple antibodies have been linked to the syndrome (Table 1).6–9 The “classic” antibodies initially found in paraneoplastic forms are now generally viewed as nonpathogenic, in part because they are directed against intracellular antigens. Neuronal injury in paraneoplastic limbic encephalitis is believed to be mediated by cytotoxic T lymphocytes, with neuronal autoantibodies being produced after the injury.4 Recently defined antibodies, such as those targeting the N-methyl-d-aspartate (NMDA) receptor6 and the LGI1 protein,7 are now understood to be common causes of limbic encephalitis.
Imaging usually shows limbic focal changes
Structural MRI or functional fluorodeoxyglucose (FDG)-PET imaging may show focal changes in limbic system structures, such as the mesial temporal lobes. It is now recognized that other cortical areas may be involved, and the term “limbic encephalitis” may give way to “cortical” or “focal encephalitis.”
In about 60% of patients, MRI shows hyperintense fluid-attenuated inversion recovery (FLAIR) or T2 signal changes in the mesial temporal lobes, likely reflecting inflammatory changes.4,10,11 On FDG-PET, hypermetabolism may be observed in the mesial temporal lobes early in the disease despite normal findings on MRI.12 Hypometabolism, either diffuse or localized to the mesial temporal lobes, eventually sets in, likely reflecting cytotoxic injury in the aftermath of prolonged inflammation or seizures.
Consider other causes
Before diagnosing limbic encephalitis, it is essential to evaluate for infectious meningoencephalitis, especially herpes simplex viral encephalitis. Thiamine deficiency (Wernicke encephalopathy), drug intoxication, prion disease, Hashimoto encephalopathy, tumor, and subclinical status epilepticus should also be considered. Some of these conditions are associated with the same neuronal autoantibodies detected in limbic encephalitis. Further complicating the picture, case reports have shown the presence of serum neuronal autoantibodies—VGKC complex13–15 and NMDA-receptor antibodies16,17—in confirmed cases of prion disease. In addition, adequately treated herpes simplex viral encephalitis can precipitate the production of NMDA-receptor antibodies and their characteristic syndrome.18–20
BRAINSTEM ENCEPHALITIS
The brainstem—the midbrain, pons, and medulla—can be affected, either in isolation or more commonly as part of a more widespread autoantibody-mediated encephalitis. Symptoms and signs include eye movement abnormalities, ptosis, dysphagia, dysarthria, ataxia, facial palsy, vertigo, hearing impairment, reduced consciousness, and hypoventilation.21
Anti-Hu, anti-Ri, and anti-Ma2 antibodies are most commonly associated with brainstem encephalitis (Table 2). Anti-Ma2-associated encephalitis may improve after a combination of immunotherapy and tumor removal21; the others have a poor prognosis.
Neuromyelitis optica spectrum disorders
Neuromyelitis optica spectrum disorders most commonly involve demyelination affecting the optic nerves and spinal cord, leading to unilateral or bilateral optic neuritis and transverse myelitis spanning three or more vertebral segments.22 The initial clinical manifestation may be an encephalitic pattern, affecting predominantly the brainstem in a restricted fashion,22 or the central nervous system in a more diffuse pattern, mimicking either acute disseminated encephalomyelitis or, in less severe cases, posterior reversible encephalopathy syndrome.23
Testing for antiaquaporin-4 antibody, also known as neuromyelitis optica immunoglobulin G, is the single most decisive laboratory test for diagnosing neuromyelitis optica spectrum disorders, so serum and cerebrospinal fluid evaluation for this autoantibody should be considered when caring for a patient whose clinical picture suggests brainstem encephalitis.22
Bickerstaff brainstem encephalitis
Bickerstaff brainstem encephalitis was first described more than half a century ago in patients with postinfectious ataxia, ophthalmoparesis, and altered consciousness. This rare disease was later found to be associated with antiganglioside GQ1b (anti-GQ1b) autoantibody. MRI is normal in about 90% of cases, so recognizing the clinical presentation and analyzing anti-GQ1b serum titers are critical to diagnosis.
Recovery is usually spontaneous and complete and can be hastened by immunotherapy, especially intravenous immunoglobulin.24
Other causes of brainstem encephalitis
The differential diagnosis of a presentation of brainstem encephalitis includes:
- Infectious causes, the most common being Listeria species followed by enterovirus 71 and herpes simplex virus.25 Tuberculosis, brucellosis, and Whipple disease should also be considered.
- Primary central nervous system inflammatory and demyelinating conditions, eg, multiple sclerosis and acute disseminated encephalomyelitis.
- Systemic inflammatory conditions, eg, Behçet disease, systemic lupus erythematosus, and sarcoidosis.
- Direct brainstem neoplastic involvement, as might occur in primary central nervous system lymphoma or leptomeningeal carcinomatosis.
CEREBELLAR SYNDROME
Patients with autoantibody-mediated encephalitis localized predominantly to the cerebellum typically present with dizziness, vertigo, and unsteady gait, progressing eventually to limb and gait ataxia.4 Symptoms are often subacute, progressing over weeks.
Multiple neuronal autoantibodies have been found to occur with cerebellar encephalitis (Table 2). In most cases, they are paraneoplastic and considered not to be pathogenic, given the intracellular location of their target antigen.4 In such cases, the syndrome is more accurately described as autoantibody-associated rather than autoantibody-mediated. Only in a minority of cases have neuronal autoantibodies been demonstrated to be directly pathogenic, ie, antimetabotropic glutamate receptor type 1 (anti-mGluR1) antibody-associated cerebellitis26 and antiglutamic acid decarboxylase (anti-GAD)-associated cerebellar ataxia.27
Differential diagnosis of cerebellar syndromes
The differential diagnosis of autoantibody-associated cerebellar syndromes is broad and includes:
- Alcohol-induced atrophy
- Drug-induced cerebellar atrophy (eg, from lithium, phenytoin, gabapentin, metronidazole, amiodarone, carbamazepine)
- Vitamin B1 and E deficiency
- Hypothyroidism, hypoparathyroidism
- Neurodegenerative disease (eg, prion disease, multiple system atrophy)
- Parainfectious causes (eg, after infection with Epstein-Barr virus)
- Immune-mediated diseases (Miller-Fisher syndrome, associated with anti-GQ1b antibodies, and antigliadin-associated ataxia, which can occur in isolation or as part of celiac disease).4
SYNDROMES ASSOCIATED WITH SPECIFIC ANTIBODIES
A few of the autoantibody-mediated encephalitic syndromes have specific antibody associations and characteristic clinical presentations. The most prominent of these syndromes are VGKC complex antibody encephalitis (as in the patient described at the beginning of this article) and anti-NMDA receptor encephalitis.
VGKC COMPLEX ANTIBODY-MEDIATED LIMBIC ENCEPHALITIS
VGKC complex antibodies, initially reported to be associated with the peripheral nerve hyperexcitability disorder neuromyotonia, were subsequently found in Morvan syndrome.28,29 Patients with this syndrome often present with autonomic dysfunction and peripheral nerve hyperexcitability but also develop insomnia, confusion, hallucinations, and memory loss. Drawing on the clinical overlap between Morvan syndrome and limbic encephalitis, Buckley et al30 were the first to report VGKC complex antibodies in two cases of limbic encephalitis.
VGKC complex antibodies are now understood to be associated with a wide variety of neurologic conditions, including chronic idiopathic pain, epilepsy,31 movement disorders, cranial nerve abnormalities, autonomic dysfunction,32 and gut dysmotility.33 In contrast, these antibodies are rare in healthy people.34 Limbic encephalitis associated with VGKC complex antibody usually lacks cerebellar and brainstem dysfunction, which may help distinguish it from other types of autoantibody-mediated limbic encephalitis.12
VGKC complex antibody does not bind to the potassium channel itself. Instead it recognizes other constituents of the channel complex, most notably LGI1 and contactin-associated protein 2 (CASPR2). LGI1 antibody is more commonly associated with limbic encephalitis—as illustrated in our case study—in addition to a distinctive type of seizure affecting the arm and face (faciobrachial dystonic seizure).34 The CASPR2 antibody, on the other hand, more often correlates with peripheral nerve manifestations and Morvan syndrome.29 Hyponatremia is commonly seen on serum chemical analysis and provides a clue that these syndromes are present.12
Good response to immunotherapy
A critical change in therapy came as clinicians realized that seizures were often refractory to standard antiepileptic drugs but responded well to immunotherapies. On the basis of these observations, sera of patients with long-standing epilepsy have been reanalyzed to look for neuronal autoantibodies.31 These antibodies should be checked in cases of new-onset refractory status epilepticus of unknown origin that does not respond to antiepileptic medications.
About half of patients with VGKC complex antibody-mediated limbic encephalitis have normal findings on brain MRI.5 Seven of 10 patients who were prospectively followed for VGKC complex antibody-mediated faciobrachial dystonic seizures had normal brain MRIs.35
VGKC complex antibody-mediated limbic encephalitis does not usually recur.36 Most cases are nonparaneoplastic, as evidenced by failure to detect a single active tumor in 64 patients after a median follow-up of 3 years. The prognosis is generally favorable except in cases with coexisting tumors.12
ANTI-NMDA RECEPTOR ENCEPHALITIS
Often associated with ovarian teratoma
Anti-NMDA receptor encephalitis typically affects women in their 20s and 30s, and about half of patients have an ovarian teratoma. It can also occur in younger patients and in men, in whom it is less likely to be associated with a neoplasm.37
Typical initial symptoms include striking and often stereotyped neuropsychiatric disturbances manifesting as psychosis, confusion, seizures, and amnesia. After 1 to 2 weeks, new symptoms set in, including reduced consciousness, movement disorders (ranging from orolingualfacial dyskinesia to rigidity and choreoathetosis), autonomic dysfunction, and hypoventilation, often prompting admission to the intensive care unit.38
Although the outcome is favorable in most cases, recovery, in contrast to VGKC complex antibody-mediated limbic encephalitis, is slow and may take longer than 1 year. Up to a quarter of patients have a relapse, underscoring the importance of maintenance immunotherapy.
It is important to undertake an intensive search for possible ovarian and extraovarian teratomas in young women with this syndrome—including CT of the pelvis, vaginal ultrasonography, and PET imaging—as removal of the teratoma may be curative.37
DIAGNOSIS OF AUTOANTIBODY-MEDIATED ENCEPHALITIS
Critical to diagnosing autoantibody-mediated encephalitis is awareness of these disorders. Since antibody testing may be very specific and is not usually part of the standard batteries of tests, a high level of suspicion is needed. Patients may present to different specialists in different settings; therefore, clinicians in pediatrics, rheumatology, psychiatry, and intensive care medicine need to be aware of these syndromes to avoid delay and misdiagnosis.
Clinical features suggesting autoantibody-mediated encephalitis include:
- Acute or subacute onset of a neurologic syndrome
- New-onset refractory status epilepticus of unknown etiology
- Acute or subacute psychiatric illness with unexpected progression to neurologic symptoms or delirium
- Unusual movement disorders not conforming to standard syndromes
- Cognitive impairment, psychosis, or behavioral or language disorders with atypical findings on imaging or cerebrospinal fluid analysis.
Imaging. Diagnosis of autoantibody-mediated encephalitis focuses on evidence suggesting an inflammatory central nervous system syndrome. MRI may show hyperintense signals on T2, FLAIR, or diffusion-weighted imaging changes in various brain regions. In many cases, however, MRI is negative despite severe clinical symptoms. In a study of 72 patients suspected of having autoimmune dementia of various etiologies, including but not restricted to antineuronal surface antibody-mediated causes, Flanagan et al39 identified atypical neuroimaging findings in only 29%. PET imaging may show hypermetabolism in certain brain areas correlating to clinical syndromes but is often difficult to obtain in a timely fashion.
Cerebrospinal fluid is often abnormal, showing elevated protein, increased immunoglobulin G synthesis, or oligoclonal banding. As with imaging studies, the cerebrospinal fluid may be normal despite severe clinical manifestations.
Electroencephalography may show focal slowing or seizure activity. Neuropsychologic testing may show different patterns of abnormalities.
Antibody testing. None of these tests can be used in isolation, and the diagnosis of autoantibody-mediated encephalitis hinges on recognizing a clinical syndrome and ordering supportive testing. Specific antibodies are more likely in different clinical syndromes and should be sought (Table 3).
Patients who have autoantibody-mediated encephalitis may test negative for autoantibodies for many possible reasons:
- Blood testing for antibodies may be less sensitive than cerebrospinal fluid testing
- Antibody titers may vary in the course of the disease
- The patient may be expressing an antibody that is less often tested for (eg, anti-AMPA receptor or antigamma-aminobutyric acid B) or one that has not yet been isolated.
Evaluating for malignancy is recommended in all cases of autoantibody-mediated encephalitis. The initial workup may involve CT of the chest, abdomen, and pelvis, as well as mammography in women and serum prostate-specific antigen testing and testicular ultrasonography in men. Ordering FDG-PET in cases in which CT is negative or inconclusive increases cancer detection.40 If no cancer is found, close tumor surveillance—every 3 to 6 months—is recommended for at least 2 years.41
TREATMENT
Owing in large part to the rarity of autoantibody-mediated encephalitides, no randomized trials of therapy have been performed. Treatment at present is guided mostly by case series and expert consensus, which suggest first-line therapy with intravenous immunoglobulin, high-dose corticosteroids, plasmapheresis, or a combination.
Different syndromes and antibody-related disorders respond differently to therapy. Syndromes associated with antibodies against intracellular antigens tend to be more resistant to immune therapy than cell surface antigen-related syndromes.4
Tiered approach
Combined treatment with intravenous immunoglobulin and high-dose corticosteroids may be superior to treatment with steroids alone for LGI1-antibody mediated limbic encephalitis.42
In cases refractory to first-line (“tier 1”) therapy, second-line immunotherapy with drugs affecting B-cell populations (eg, rituximab, cyclophosphamide, and mycophenolate mofetil) has been used.
A tiered approach has been most extensively studied for anti-NMDA-receptor encephalitis, with better outcomes found using second-line therapy.43
Treatment strategies for these disorders will likely evolve over time with additional experience.
Outpatient management
Once the patient is discharged from the hospital, a multidisciplinary approach to care is recommended, including physical rehabilitation, speech therapy, neuropsychiatric and neuroimmunologic follow-up, and annual surveillance for malignancies.
- Brierley JB, Corsellis JAN, Hierons R, Nevin S. Subacute encephalitis of later adult life mainly affecting the limbic areas. Brain 1960; 83:357–368.
- Corsellis JA, Goldberg GJ, Norton AR. “Limbic encephalitis” and its association with carcinoma. Brain 1968; 91:481–496.
- Greenlee JE, Brashear HR. Antibodies to cerebellar Purkinje cells in patients with paraneoplastic cerebellar degeneration and ovarian carcinoma. Ann Neurol 1983; 14:609–613.
- Rosenfeld MR, Dalmau JO. Paraneoplastic disorders of the CNS and autoimmune synaptic encephalitis. Continuum (Minneap Minn) 2012; 18:366–383.
- Irani SR, Gelfand JM, Al-Diwani A, Vincent A. Cell-surface central nervous system autoantibodies: clinical relevance and emerging paradigms. Ann Neurol 2014; 76:168–184.
- Dalmau J, Tüzün E, Wu HY, et al. Paraneoplastic anti-N-methyl-D-aspartate receptor encephalitis associated with ovarian teratoma. Ann Neurol 2007; 61:25–36.
- Irani SR, Alexander S, Waters P, et al. Antibodies to Kv1 potassium channel-complex proteins leucine-rich, glioma inactivated 1 protein and contactin-associated protein-2 in limbic encephalitis, Morvan’s syndrome and acquired neuromyotonia. Brain 2010; 133:2734–2748.
- Lancaster E, Lai M, Peng X, et al. Antibodies to the GABA(B) receptor in limbic encephalitis with seizures: case series and characterisation of the antigen. Lancet Neurol 2010; 9:67–76.
- Lai M, Hughes EG, Peng X, et al. AMPA receptor antibodies in limbic encephalitis alter synaptic receptor location. Ann Neurol 2009; 65:424–434.
- Zuliani L, Graus F, Giometto B, Bien C, Vincent A. Central nervous system neuronal surface antibody associated syndromes: review and guidelines for recognition. J Neurol Neurosurg Psychiatry 2012; 83:638–645.
- Ances BM, Vitaliani R, Taylor RA, et al. Treatment-responsive limbic encephalitis identified by neuropil antibodies: MRI and PET correlates. Brain 2005; 128:1764–1777.
- Vincent A, Buckley C, Schott JM, et al. Potassium channel antibody-associated encephalopathy: a potentially immunotherapy-responsive form of limbic encephalitis. Brain 2004; 127:701–712.
- Jammoul A, Lederman RJ, Tavee J, Li Y. Presence of voltage-gated potassium channel complex antibody in a case of genetic prion disease. BMJ Case Rep 2014; pii:bcr2013201622.
- Angus-Leppan H, Rudge P, Mead S, Collinge J, Vincent A. Autoantibodies in sporadic Creutzfeldt-Jakob disease. JAMA Neurol 2013; 70:919–922.
- Fujita K, Yuasa T, Watanabe O, et al. Voltage-gated potassium channel complex antibodies in Creutzfeldt-Jakob disease. J Neurol 2012; 259:2249–2250.
- Fujita K, Yuasa T, Takahashi Y, et al. Antibodies to N-methyl-D-aspartate glutamate receptors in Creutzfeldt–Jakob disease patients. J Neuroimmunol 2012; 251:90–93.
- Mackay G, Ahmad K, Stone J, et al. NMDA receptor autoantibodies in sporadic Creutzfeldt-Jakob disease. J Neurol 2012; 259:1979–1981.
- Leypoldt F, Titulaer MJ, Aguilar E, et al. Herpes simplex virus–1 encephalitis can trigger anti-NMDA receptor encephalitis: case report. Neurology 2013; 81:1637–1639.
- Desena A, Graves D, Warnack W, Greenberg BM. Herpes simplex encephalitis as a potential cause of anti-N-methyl-D-aspartate receptor antibody encephalitis: report of 2 cases. JAMA Neurol 2014; 71:344–346.
- Armangue T, Leypoldt F, Málaga I, et al. Herpes simplex virus encephalitis is a trigger of brain autoimmunity. Ann Neurol 2014; 75:317–323.
- Blaes F. Paraneoplastic brain stem encephalitis. Curr Treat Options Neurol 2013; 15:201–209.
- Wildemann B, Jarius S. The expanding range of autoimmune disorders of the nervous system. Lancet Neurol 2013; 12:22–24.
- Kim W, Kim SH, Lee SH, Li XF, Kim HJ. Brain abnormalities as an initial manifestation of neuromyelitis optica spectrum disorder. Mult Scler 2011; 17:1107–1112.
- Shahrizaila N, Yuki N. Bickerstaff brainstem encephalitis and Fisher syndrome: anti-GQ1b antibody syndrome. J Neurol Neurosurg Psychiatry 2013; 84:576–583.
- Jubelt B, Mihai C, Li MT, Veerapaneni P. Rhombencephalitis/brainstem encephalitis. Curr Neurol Neurosci Rep 2011; 11:543–552.
- Sillevis Smitt P, Kinoshita A, De Leeuw B, et al. Paraneoplastic cerebellar ataxia due to autoantibodies against a glutamate receptor. N Engl J Med 2000; 342:21–27.
- Ishida K, Mitoma H, Son SY, et al. Selective suppression of cerebellar GABAergic transmission by an autoantibody to glutamic acid decarboxylase. Ann Neurol 1999; 46:263–267.
- Hart IK, Waters C, Vincent A, et al. Autoantibodies detected to expressed K+ channels are implicated in neuromyotonia. Ann Neurol 1997; 41:238–246.
- Barber P, Anderson NE, Vincent A. Morvan’s syndrome associated with voltage-gated K+ channel antibodies. Neurology 2000; 54:771–772.
- Buckley C, Oger J, Clover L, et al. Potassium channel antibodies in two patients with reversible limbic encephalitis. Ann Neurol 2001; 50:73–78.
- Majoie HJ, de Baets M, Renier W, Lang B, Vincent A. Antibodies to voltage-gated potassium and calcium channels in epilepsy. Epilepsy Res 2006; 71:135–141.
- Tan KM, Lennon VA, Klein CJ, Boeve BF, Pittock SJ. Clinical spectrum of voltage-gated potassium channel autoimmunity. Neurology 2008; 70:1883–1890.
- Knowles CH, Lang B, Clover L, et al. A role for autoantibodies in some cases of acquired non-paraneoplastic gut dysmotility. Scand J Gastroenterol 2002; 37:166–170.
- Irani SR, Michell AW, Lang B, et al. Faciobrachial dystonic seizures precede Lgi1 antibody limbic encephalitis. Ann Neurol 2011; 69:892–900.
- Irani SR, Stagg CJ, Schott JM, et al. Faciobrachial dystonic seizures: the influence of immunotherapy on seizure control and prevention of cognitive impairment in a broadening phenotype. Brain 2013: 136:3151–3162.
- Vincent A, Bien CG, Irani SR, Waters P. Autoantibodies associated with diseases of the CNS: new developments and future challenges. Lancet Neurol 2011; 10:759–772.
- Dalmau J, Lancaster E, Martinez-Hernandez E, Rosenfeld MR, Balice-Gordon R. Clinical experience and laboratory investigations in patients with anti-NMDAR encephalitis. Lancet Neurol 2011; 10:63–74.
- Irani SR, Bera K, Waters P, et al. N-methyl-D-aspartate antibody encephalitis: temporal progression of clinical and paraclinical observations in a predominantly non-paraneoplastic disorder of both sexes. Brain 2010; 133:1655–1667.
- Flanagan EP, McKeon A, Lennon VA, et al. Autoimmune dementia: clinical course and predictors of immunotherapy response. Mayo Clin Proc 2010; 85:881–897.
- Younes-Mhenni S, Janier MF, Cinotti L, et al. FDG-PET improves tumour detection in patients with paraneoplastic neurological syndromes. Brain 2004; 127:2331–2338.
- Lancaster E, Martinez-Hernandez E, Dalmau J. Encephalitis and antibodies to synaptic and neuronal cell surface proteins. Neurology 2011; 77:179–189.
- Shin YW, Lee ST, Shin JW, et al. VGKC-complex/LGI1-antibody encephalitis: clinical manifestations and response to immunotherapy. J Neuroimmunol 2013; 265:75–81.
- Titulaer MJ, McCracken L, Gabilondo I, et al. Treatment and prognostic factors for long-term outcome in patients with anti-NMDA receptor encephalitis: an observational cohort study. Lancet Neurol 2013; 12:157–165.
A 79-year-old woman with a history of breast cancer in remission and hypertension presented to a local emergency department because of subacute memory loss and compulsive shopping. Her serum sodium concentration was 127 mmol/L (reference range 132–148). Computed tomography (CT) and magnetic resonance imaging (MRI) of the brain were normal, and she was sent home.
Three days later, she experienced a generalized tonic-clonic seizure that evolved into status epilepticus. She was intubated and admitted to the intensive care unit. Cerebrospinal fluid analysis was normal, and infectious causes of encephalitis were ruled out. MRI showed increased signal in both hippocampi (Figure 1). Her seizures were refractory to treatment, and she was given pentobarbital to induce a coma.
Serum evaluation of neuronal antibodies revealed elevated titers of the voltage-gated potassium channel (VGKC) complex antibody, with subsequent subtyping confirming the leucine-rich glioma-inactivated protein 1 (LGI1) protein as the antigenic target.
She received a 5-day course of intravenous immunoglobulin and methylprednisolone, pentobarbital was withdrawn, and the seizures did not recur, but weeks later she remained comatose. Positron emission tomography (PET) of the brain revealed hypermetabolism in the medial and anterior aspects of both temporal lobes. She underwent five sessions of plasma exchange, after which she began to improve and follow commands. She was ultimately discharged to an acute rehabilitation facility after a 4-week hospital stay.
She received infusions of intravenous immunoglobulin twice a month for 6 months. At her last follow-up visit, she was seizure-free and neurologically intact except for mild inattention.
NEWLY RECOGNIZED DISEASES
Although autoantibody-mediated encephalitic syndromes were first described more than 50 years ago,1,2 their autoimmune basis was not recognized until the early 1980s.3 In the past 10 years, a flood of novel clinical syndromes associated with neuronal autoantibodies has been described that may be markedly improved or even completely resolved with immunotherapy. In cases of unexplained seizure, encephalitis, or acute-onset psychiatric syndromes, suspecting these syndromes can lead to diagnosis, treatment, and a good outcome.
This review describes the key clinical autoantibody-mediated encephalitic syndromes, explains the better-characterized antibody associations, and discusses their diagnosis and treatment.
CLASSIFIED ANATOMICALLY, IMMUNOLOGICALLY, OR EPONYMOUSLY
Autoantibody-mediated encephalitis is also known as autoimmune-mediated encephalitis, autoimmune-mediated limbic encephalitis, and autoimmune synaptic encephalitis.
How to categorize these syndromes is still in flux: they can be listed by the area of the brain affected, the antibody involved, or the name of the discoverer (eg, Morvan syndrome).
Autoantibodies identified in autoimmune encephalitis fall under two broad categories:
- Those targeting intracellular (intranuclear or intracytoplasmic) antigens; the syndromes they cause are more likely to be paraneoplastic and less responsive to immunotherapy
- Those targeting antigens on the neuronal surface: the syndromes they cause are less likely to be paraneoplastic and are more responsive to immunotherapy.4
SYNDROMES DEFINED BY BRAIN AREA AFFECTED
Below, we provide examples of neurologic syndromes of autoantibody-mediated encephalitis according to the region of the brain most affected, ie, the limbic system, the brainstem, or the cerebellum (Figure 2).
LIMBIC ENCEPHALITIS
Memory loss, behavioral changes, seizures
Patients with limbic encephalitis (such as the patient described in the vignette above) present with symptoms attributed to dysfunction of mesial temporal lobe structures, most notably the hippocampus. Prominent symptoms include short-term memory loss, behavioral disturbances such as agitation and confusion, and psychiatric problems such as depression and psychosis. Recurrent seizures are a salient feature and, not uncommonly, progress to status epilepticus.
Antibodies are not all cancer-associated
Cerebrospinal fluid analysis can be normal or show abnormalities suggesting immune activation, eg, slight pleocytosis, elevated protein, increased immunoglobulin G synthesis, and oligoclonal banding.5
In many cases, an autoantibody is found in the blood or in the cerebrospinal fluid. Some patients may express more than one autoantibody, so the traditional view of “one antibody, one syndrome” is incorrect.
Although initially identified as a rare paraneoplastic disorder, limbic encephalitis sometimes occurs in the absence of malignancy.
Multiple antibodies have been linked to the syndrome (Table 1).6–9 The “classic” antibodies initially found in paraneoplastic forms are now generally viewed as nonpathogenic, in part because they are directed against intracellular antigens. Neuronal injury in paraneoplastic limbic encephalitis is believed to be mediated by cytotoxic T lymphocytes, with neuronal autoantibodies being produced after the injury.4 Recently defined antibodies, such as those targeting the N-methyl-d-aspartate (NMDA) receptor6 and the LGI1 protein,7 are now understood to be common causes of limbic encephalitis.
Imaging usually shows limbic focal changes
Structural MRI or functional fluorodeoxyglucose (FDG)-PET imaging may show focal changes in limbic system structures, such as the mesial temporal lobes. It is now recognized that other cortical areas may be involved, and the term “limbic encephalitis” may give way to “cortical” or “focal encephalitis.”
In about 60% of patients, MRI shows hyperintense fluid-attenuated inversion recovery (FLAIR) or T2 signal changes in the mesial temporal lobes, likely reflecting inflammatory changes.4,10,11 On FDG-PET, hypermetabolism may be observed in the mesial temporal lobes early in the disease despite normal findings on MRI.12 Hypometabolism, either diffuse or localized to the mesial temporal lobes, eventually sets in, likely reflecting cytotoxic injury in the aftermath of prolonged inflammation or seizures.
Consider other causes
Before diagnosing limbic encephalitis, it is essential to evaluate for infectious meningoencephalitis, especially herpes simplex viral encephalitis. Thiamine deficiency (Wernicke encephalopathy), drug intoxication, prion disease, Hashimoto encephalopathy, tumor, and subclinical status epilepticus should also be considered. Some of these conditions are associated with the same neuronal autoantibodies detected in limbic encephalitis. Further complicating the picture, case reports have shown the presence of serum neuronal autoantibodies—VGKC complex13–15 and NMDA-receptor antibodies16,17—in confirmed cases of prion disease. In addition, adequately treated herpes simplex viral encephalitis can precipitate the production of NMDA-receptor antibodies and their characteristic syndrome.18–20
BRAINSTEM ENCEPHALITIS
The brainstem—the midbrain, pons, and medulla—can be affected, either in isolation or more commonly as part of a more widespread autoantibody-mediated encephalitis. Symptoms and signs include eye movement abnormalities, ptosis, dysphagia, dysarthria, ataxia, facial palsy, vertigo, hearing impairment, reduced consciousness, and hypoventilation.21
Anti-Hu, anti-Ri, and anti-Ma2 antibodies are most commonly associated with brainstem encephalitis (Table 2). Anti-Ma2-associated encephalitis may improve after a combination of immunotherapy and tumor removal21; the others have a poor prognosis.
Neuromyelitis optica spectrum disorders
Neuromyelitis optica spectrum disorders most commonly involve demyelination affecting the optic nerves and spinal cord, leading to unilateral or bilateral optic neuritis and transverse myelitis spanning three or more vertebral segments.22 The initial clinical manifestation may be an encephalitic pattern, affecting predominantly the brainstem in a restricted fashion,22 or the central nervous system in a more diffuse pattern, mimicking either acute disseminated encephalomyelitis or, in less severe cases, posterior reversible encephalopathy syndrome.23
Testing for antiaquaporin-4 antibody, also known as neuromyelitis optica immunoglobulin G, is the single most decisive laboratory test for diagnosing neuromyelitis optica spectrum disorders, so serum and cerebrospinal fluid evaluation for this autoantibody should be considered when caring for a patient whose clinical picture suggests brainstem encephalitis.22
Bickerstaff brainstem encephalitis
Bickerstaff brainstem encephalitis was first described more than half a century ago in patients with postinfectious ataxia, ophthalmoparesis, and altered consciousness. This rare disease was later found to be associated with antiganglioside GQ1b (anti-GQ1b) autoantibody. MRI is normal in about 90% of cases, so recognizing the clinical presentation and analyzing anti-GQ1b serum titers are critical to diagnosis.
Recovery is usually spontaneous and complete and can be hastened by immunotherapy, especially intravenous immunoglobulin.24
Other causes of brainstem encephalitis
The differential diagnosis of a presentation of brainstem encephalitis includes:
- Infectious causes, the most common being Listeria species followed by enterovirus 71 and herpes simplex virus.25 Tuberculosis, brucellosis, and Whipple disease should also be considered.
- Primary central nervous system inflammatory and demyelinating conditions, eg, multiple sclerosis and acute disseminated encephalomyelitis.
- Systemic inflammatory conditions, eg, Behçet disease, systemic lupus erythematosus, and sarcoidosis.
- Direct brainstem neoplastic involvement, as might occur in primary central nervous system lymphoma or leptomeningeal carcinomatosis.
CEREBELLAR SYNDROME
Patients with autoantibody-mediated encephalitis localized predominantly to the cerebellum typically present with dizziness, vertigo, and unsteady gait, progressing eventually to limb and gait ataxia.4 Symptoms are often subacute, progressing over weeks.
Multiple neuronal autoantibodies have been found to occur with cerebellar encephalitis (Table 2). In most cases, they are paraneoplastic and considered not to be pathogenic, given the intracellular location of their target antigen.4 In such cases, the syndrome is more accurately described as autoantibody-associated rather than autoantibody-mediated. Only in a minority of cases have neuronal autoantibodies been demonstrated to be directly pathogenic, ie, antimetabotropic glutamate receptor type 1 (anti-mGluR1) antibody-associated cerebellitis26 and antiglutamic acid decarboxylase (anti-GAD)-associated cerebellar ataxia.27
Differential diagnosis of cerebellar syndromes
The differential diagnosis of autoantibody-associated cerebellar syndromes is broad and includes:
- Alcohol-induced atrophy
- Drug-induced cerebellar atrophy (eg, from lithium, phenytoin, gabapentin, metronidazole, amiodarone, carbamazepine)
- Vitamin B1 and E deficiency
- Hypothyroidism, hypoparathyroidism
- Neurodegenerative disease (eg, prion disease, multiple system atrophy)
- Parainfectious causes (eg, after infection with Epstein-Barr virus)
- Immune-mediated diseases (Miller-Fisher syndrome, associated with anti-GQ1b antibodies, and antigliadin-associated ataxia, which can occur in isolation or as part of celiac disease).4
SYNDROMES ASSOCIATED WITH SPECIFIC ANTIBODIES
A few of the autoantibody-mediated encephalitic syndromes have specific antibody associations and characteristic clinical presentations. The most prominent of these syndromes are VGKC complex antibody encephalitis (as in the patient described at the beginning of this article) and anti-NMDA receptor encephalitis.
VGKC COMPLEX ANTIBODY-MEDIATED LIMBIC ENCEPHALITIS
VGKC complex antibodies, initially reported to be associated with the peripheral nerve hyperexcitability disorder neuromyotonia, were subsequently found in Morvan syndrome.28,29 Patients with this syndrome often present with autonomic dysfunction and peripheral nerve hyperexcitability but also develop insomnia, confusion, hallucinations, and memory loss. Drawing on the clinical overlap between Morvan syndrome and limbic encephalitis, Buckley et al30 were the first to report VGKC complex antibodies in two cases of limbic encephalitis.
VGKC complex antibodies are now understood to be associated with a wide variety of neurologic conditions, including chronic idiopathic pain, epilepsy,31 movement disorders, cranial nerve abnormalities, autonomic dysfunction,32 and gut dysmotility.33 In contrast, these antibodies are rare in healthy people.34 Limbic encephalitis associated with VGKC complex antibody usually lacks cerebellar and brainstem dysfunction, which may help distinguish it from other types of autoantibody-mediated limbic encephalitis.12
VGKC complex antibody does not bind to the potassium channel itself. Instead it recognizes other constituents of the channel complex, most notably LGI1 and contactin-associated protein 2 (CASPR2). LGI1 antibody is more commonly associated with limbic encephalitis—as illustrated in our case study—in addition to a distinctive type of seizure affecting the arm and face (faciobrachial dystonic seizure).34 The CASPR2 antibody, on the other hand, more often correlates with peripheral nerve manifestations and Morvan syndrome.29 Hyponatremia is commonly seen on serum chemical analysis and provides a clue that these syndromes are present.12
Good response to immunotherapy
A critical change in therapy came as clinicians realized that seizures were often refractory to standard antiepileptic drugs but responded well to immunotherapies. On the basis of these observations, sera of patients with long-standing epilepsy have been reanalyzed to look for neuronal autoantibodies.31 These antibodies should be checked in cases of new-onset refractory status epilepticus of unknown origin that does not respond to antiepileptic medications.
About half of patients with VGKC complex antibody-mediated limbic encephalitis have normal findings on brain MRI.5 Seven of 10 patients who were prospectively followed for VGKC complex antibody-mediated faciobrachial dystonic seizures had normal brain MRIs.35
VGKC complex antibody-mediated limbic encephalitis does not usually recur.36 Most cases are nonparaneoplastic, as evidenced by failure to detect a single active tumor in 64 patients after a median follow-up of 3 years. The prognosis is generally favorable except in cases with coexisting tumors.12
ANTI-NMDA RECEPTOR ENCEPHALITIS
Often associated with ovarian teratoma
Anti-NMDA receptor encephalitis typically affects women in their 20s and 30s, and about half of patients have an ovarian teratoma. It can also occur in younger patients and in men, in whom it is less likely to be associated with a neoplasm.37
Typical initial symptoms include striking and often stereotyped neuropsychiatric disturbances manifesting as psychosis, confusion, seizures, and amnesia. After 1 to 2 weeks, new symptoms set in, including reduced consciousness, movement disorders (ranging from orolingualfacial dyskinesia to rigidity and choreoathetosis), autonomic dysfunction, and hypoventilation, often prompting admission to the intensive care unit.38
Although the outcome is favorable in most cases, recovery, in contrast to VGKC complex antibody-mediated limbic encephalitis, is slow and may take longer than 1 year. Up to a quarter of patients have a relapse, underscoring the importance of maintenance immunotherapy.
It is important to undertake an intensive search for possible ovarian and extraovarian teratomas in young women with this syndrome—including CT of the pelvis, vaginal ultrasonography, and PET imaging—as removal of the teratoma may be curative.37
DIAGNOSIS OF AUTOANTIBODY-MEDIATED ENCEPHALITIS
Critical to diagnosing autoantibody-mediated encephalitis is awareness of these disorders. Since antibody testing may be very specific and is not usually part of the standard batteries of tests, a high level of suspicion is needed. Patients may present to different specialists in different settings; therefore, clinicians in pediatrics, rheumatology, psychiatry, and intensive care medicine need to be aware of these syndromes to avoid delay and misdiagnosis.
Clinical features suggesting autoantibody-mediated encephalitis include:
- Acute or subacute onset of a neurologic syndrome
- New-onset refractory status epilepticus of unknown etiology
- Acute or subacute psychiatric illness with unexpected progression to neurologic symptoms or delirium
- Unusual movement disorders not conforming to standard syndromes
- Cognitive impairment, psychosis, or behavioral or language disorders with atypical findings on imaging or cerebrospinal fluid analysis.
Imaging. Diagnosis of autoantibody-mediated encephalitis focuses on evidence suggesting an inflammatory central nervous system syndrome. MRI may show hyperintense signals on T2, FLAIR, or diffusion-weighted imaging changes in various brain regions. In many cases, however, MRI is negative despite severe clinical symptoms. In a study of 72 patients suspected of having autoimmune dementia of various etiologies, including but not restricted to antineuronal surface antibody-mediated causes, Flanagan et al39 identified atypical neuroimaging findings in only 29%. PET imaging may show hypermetabolism in certain brain areas correlating to clinical syndromes but is often difficult to obtain in a timely fashion.
Cerebrospinal fluid is often abnormal, showing elevated protein, increased immunoglobulin G synthesis, or oligoclonal banding. As with imaging studies, the cerebrospinal fluid may be normal despite severe clinical manifestations.
Electroencephalography may show focal slowing or seizure activity. Neuropsychologic testing may show different patterns of abnormalities.
Antibody testing. None of these tests can be used in isolation, and the diagnosis of autoantibody-mediated encephalitis hinges on recognizing a clinical syndrome and ordering supportive testing. Specific antibodies are more likely in different clinical syndromes and should be sought (Table 3).
Patients who have autoantibody-mediated encephalitis may test negative for autoantibodies for many possible reasons:
- Blood testing for antibodies may be less sensitive than cerebrospinal fluid testing
- Antibody titers may vary in the course of the disease
- The patient may be expressing an antibody that is less often tested for (eg, anti-AMPA receptor or antigamma-aminobutyric acid B) or one that has not yet been isolated.
Evaluating for malignancy is recommended in all cases of autoantibody-mediated encephalitis. The initial workup may involve CT of the chest, abdomen, and pelvis, as well as mammography in women and serum prostate-specific antigen testing and testicular ultrasonography in men. Ordering FDG-PET in cases in which CT is negative or inconclusive increases cancer detection.40 If no cancer is found, close tumor surveillance—every 3 to 6 months—is recommended for at least 2 years.41
TREATMENT
Owing in large part to the rarity of autoantibody-mediated encephalitides, no randomized trials of therapy have been performed. Treatment at present is guided mostly by case series and expert consensus, which suggest first-line therapy with intravenous immunoglobulin, high-dose corticosteroids, plasmapheresis, or a combination.
Different syndromes and antibody-related disorders respond differently to therapy. Syndromes associated with antibodies against intracellular antigens tend to be more resistant to immune therapy than cell surface antigen-related syndromes.4
Tiered approach
Combined treatment with intravenous immunoglobulin and high-dose corticosteroids may be superior to treatment with steroids alone for LGI1-antibody mediated limbic encephalitis.42
In cases refractory to first-line (“tier 1”) therapy, second-line immunotherapy with drugs affecting B-cell populations (eg, rituximab, cyclophosphamide, and mycophenolate mofetil) has been used.
A tiered approach has been most extensively studied for anti-NMDA-receptor encephalitis, with better outcomes found using second-line therapy.43
Treatment strategies for these disorders will likely evolve over time with additional experience.
Outpatient management
Once the patient is discharged from the hospital, a multidisciplinary approach to care is recommended, including physical rehabilitation, speech therapy, neuropsychiatric and neuroimmunologic follow-up, and annual surveillance for malignancies.
A 79-year-old woman with a history of breast cancer in remission and hypertension presented to a local emergency department because of subacute memory loss and compulsive shopping. Her serum sodium concentration was 127 mmol/L (reference range 132–148). Computed tomography (CT) and magnetic resonance imaging (MRI) of the brain were normal, and she was sent home.
Three days later, she experienced a generalized tonic-clonic seizure that evolved into status epilepticus. She was intubated and admitted to the intensive care unit. Cerebrospinal fluid analysis was normal, and infectious causes of encephalitis were ruled out. MRI showed increased signal in both hippocampi (Figure 1). Her seizures were refractory to treatment, and she was given pentobarbital to induce a coma.
Serum evaluation of neuronal antibodies revealed elevated titers of the voltage-gated potassium channel (VGKC) complex antibody, with subsequent subtyping confirming the leucine-rich glioma-inactivated protein 1 (LGI1) protein as the antigenic target.
She received a 5-day course of intravenous immunoglobulin and methylprednisolone, pentobarbital was withdrawn, and the seizures did not recur, but weeks later she remained comatose. Positron emission tomography (PET) of the brain revealed hypermetabolism in the medial and anterior aspects of both temporal lobes. She underwent five sessions of plasma exchange, after which she began to improve and follow commands. She was ultimately discharged to an acute rehabilitation facility after a 4-week hospital stay.
She received infusions of intravenous immunoglobulin twice a month for 6 months. At her last follow-up visit, she was seizure-free and neurologically intact except for mild inattention.
NEWLY RECOGNIZED DISEASES
Although autoantibody-mediated encephalitic syndromes were first described more than 50 years ago,1,2 their autoimmune basis was not recognized until the early 1980s.3 In the past 10 years, a flood of novel clinical syndromes associated with neuronal autoantibodies has been described that may be markedly improved or even completely resolved with immunotherapy. In cases of unexplained seizure, encephalitis, or acute-onset psychiatric syndromes, suspecting these syndromes can lead to diagnosis, treatment, and a good outcome.
This review describes the key clinical autoantibody-mediated encephalitic syndromes, explains the better-characterized antibody associations, and discusses their diagnosis and treatment.
CLASSIFIED ANATOMICALLY, IMMUNOLOGICALLY, OR EPONYMOUSLY
Autoantibody-mediated encephalitis is also known as autoimmune-mediated encephalitis, autoimmune-mediated limbic encephalitis, and autoimmune synaptic encephalitis.
How to categorize these syndromes is still in flux: they can be listed by the area of the brain affected, the antibody involved, or the name of the discoverer (eg, Morvan syndrome).
Autoantibodies identified in autoimmune encephalitis fall under two broad categories:
- Those targeting intracellular (intranuclear or intracytoplasmic) antigens; the syndromes they cause are more likely to be paraneoplastic and less responsive to immunotherapy
- Those targeting antigens on the neuronal surface: the syndromes they cause are less likely to be paraneoplastic and are more responsive to immunotherapy.4
SYNDROMES DEFINED BY BRAIN AREA AFFECTED
Below, we provide examples of neurologic syndromes of autoantibody-mediated encephalitis according to the region of the brain most affected, ie, the limbic system, the brainstem, or the cerebellum (Figure 2).
LIMBIC ENCEPHALITIS
Memory loss, behavioral changes, seizures
Patients with limbic encephalitis (such as the patient described in the vignette above) present with symptoms attributed to dysfunction of mesial temporal lobe structures, most notably the hippocampus. Prominent symptoms include short-term memory loss, behavioral disturbances such as agitation and confusion, and psychiatric problems such as depression and psychosis. Recurrent seizures are a salient feature and, not uncommonly, progress to status epilepticus.
Antibodies are not all cancer-associated
Cerebrospinal fluid analysis can be normal or show abnormalities suggesting immune activation, eg, slight pleocytosis, elevated protein, increased immunoglobulin G synthesis, and oligoclonal banding.5
In many cases, an autoantibody is found in the blood or in the cerebrospinal fluid. Some patients may express more than one autoantibody, so the traditional view of “one antibody, one syndrome” is incorrect.
Although initially identified as a rare paraneoplastic disorder, limbic encephalitis sometimes occurs in the absence of malignancy.
Multiple antibodies have been linked to the syndrome (Table 1).6–9 The “classic” antibodies initially found in paraneoplastic forms are now generally viewed as nonpathogenic, in part because they are directed against intracellular antigens. Neuronal injury in paraneoplastic limbic encephalitis is believed to be mediated by cytotoxic T lymphocytes, with neuronal autoantibodies being produced after the injury.4 Recently defined antibodies, such as those targeting the N-methyl-d-aspartate (NMDA) receptor6 and the LGI1 protein,7 are now understood to be common causes of limbic encephalitis.
Imaging usually shows limbic focal changes
Structural MRI or functional fluorodeoxyglucose (FDG)-PET imaging may show focal changes in limbic system structures, such as the mesial temporal lobes. It is now recognized that other cortical areas may be involved, and the term “limbic encephalitis” may give way to “cortical” or “focal encephalitis.”
In about 60% of patients, MRI shows hyperintense fluid-attenuated inversion recovery (FLAIR) or T2 signal changes in the mesial temporal lobes, likely reflecting inflammatory changes.4,10,11 On FDG-PET, hypermetabolism may be observed in the mesial temporal lobes early in the disease despite normal findings on MRI.12 Hypometabolism, either diffuse or localized to the mesial temporal lobes, eventually sets in, likely reflecting cytotoxic injury in the aftermath of prolonged inflammation or seizures.
Consider other causes
Before diagnosing limbic encephalitis, it is essential to evaluate for infectious meningoencephalitis, especially herpes simplex viral encephalitis. Thiamine deficiency (Wernicke encephalopathy), drug intoxication, prion disease, Hashimoto encephalopathy, tumor, and subclinical status epilepticus should also be considered. Some of these conditions are associated with the same neuronal autoantibodies detected in limbic encephalitis. Further complicating the picture, case reports have shown the presence of serum neuronal autoantibodies—VGKC complex13–15 and NMDA-receptor antibodies16,17—in confirmed cases of prion disease. In addition, adequately treated herpes simplex viral encephalitis can precipitate the production of NMDA-receptor antibodies and their characteristic syndrome.18–20
BRAINSTEM ENCEPHALITIS
The brainstem—the midbrain, pons, and medulla—can be affected, either in isolation or more commonly as part of a more widespread autoantibody-mediated encephalitis. Symptoms and signs include eye movement abnormalities, ptosis, dysphagia, dysarthria, ataxia, facial palsy, vertigo, hearing impairment, reduced consciousness, and hypoventilation.21
Anti-Hu, anti-Ri, and anti-Ma2 antibodies are most commonly associated with brainstem encephalitis (Table 2). Anti-Ma2-associated encephalitis may improve after a combination of immunotherapy and tumor removal21; the others have a poor prognosis.
Neuromyelitis optica spectrum disorders
Neuromyelitis optica spectrum disorders most commonly involve demyelination affecting the optic nerves and spinal cord, leading to unilateral or bilateral optic neuritis and transverse myelitis spanning three or more vertebral segments.22 The initial clinical manifestation may be an encephalitic pattern, affecting predominantly the brainstem in a restricted fashion,22 or the central nervous system in a more diffuse pattern, mimicking either acute disseminated encephalomyelitis or, in less severe cases, posterior reversible encephalopathy syndrome.23
Testing for antiaquaporin-4 antibody, also known as neuromyelitis optica immunoglobulin G, is the single most decisive laboratory test for diagnosing neuromyelitis optica spectrum disorders, so serum and cerebrospinal fluid evaluation for this autoantibody should be considered when caring for a patient whose clinical picture suggests brainstem encephalitis.22
Bickerstaff brainstem encephalitis
Bickerstaff brainstem encephalitis was first described more than half a century ago in patients with postinfectious ataxia, ophthalmoparesis, and altered consciousness. This rare disease was later found to be associated with antiganglioside GQ1b (anti-GQ1b) autoantibody. MRI is normal in about 90% of cases, so recognizing the clinical presentation and analyzing anti-GQ1b serum titers are critical to diagnosis.
Recovery is usually spontaneous and complete and can be hastened by immunotherapy, especially intravenous immunoglobulin.24
Other causes of brainstem encephalitis
The differential diagnosis of a presentation of brainstem encephalitis includes:
- Infectious causes, the most common being Listeria species followed by enterovirus 71 and herpes simplex virus.25 Tuberculosis, brucellosis, and Whipple disease should also be considered.
- Primary central nervous system inflammatory and demyelinating conditions, eg, multiple sclerosis and acute disseminated encephalomyelitis.
- Systemic inflammatory conditions, eg, Behçet disease, systemic lupus erythematosus, and sarcoidosis.
- Direct brainstem neoplastic involvement, as might occur in primary central nervous system lymphoma or leptomeningeal carcinomatosis.
CEREBELLAR SYNDROME
Patients with autoantibody-mediated encephalitis localized predominantly to the cerebellum typically present with dizziness, vertigo, and unsteady gait, progressing eventually to limb and gait ataxia.4 Symptoms are often subacute, progressing over weeks.
Multiple neuronal autoantibodies have been found to occur with cerebellar encephalitis (Table 2). In most cases, they are paraneoplastic and considered not to be pathogenic, given the intracellular location of their target antigen.4 In such cases, the syndrome is more accurately described as autoantibody-associated rather than autoantibody-mediated. Only in a minority of cases have neuronal autoantibodies been demonstrated to be directly pathogenic, ie, antimetabotropic glutamate receptor type 1 (anti-mGluR1) antibody-associated cerebellitis26 and antiglutamic acid decarboxylase (anti-GAD)-associated cerebellar ataxia.27
Differential diagnosis of cerebellar syndromes
The differential diagnosis of autoantibody-associated cerebellar syndromes is broad and includes:
- Alcohol-induced atrophy
- Drug-induced cerebellar atrophy (eg, from lithium, phenytoin, gabapentin, metronidazole, amiodarone, carbamazepine)
- Vitamin B1 and E deficiency
- Hypothyroidism, hypoparathyroidism
- Neurodegenerative disease (eg, prion disease, multiple system atrophy)
- Parainfectious causes (eg, after infection with Epstein-Barr virus)
- Immune-mediated diseases (Miller-Fisher syndrome, associated with anti-GQ1b antibodies, and antigliadin-associated ataxia, which can occur in isolation or as part of celiac disease).4
SYNDROMES ASSOCIATED WITH SPECIFIC ANTIBODIES
A few of the autoantibody-mediated encephalitic syndromes have specific antibody associations and characteristic clinical presentations. The most prominent of these syndromes are VGKC complex antibody encephalitis (as in the patient described at the beginning of this article) and anti-NMDA receptor encephalitis.
VGKC COMPLEX ANTIBODY-MEDIATED LIMBIC ENCEPHALITIS
VGKC complex antibodies, initially reported to be associated with the peripheral nerve hyperexcitability disorder neuromyotonia, were subsequently found in Morvan syndrome.28,29 Patients with this syndrome often present with autonomic dysfunction and peripheral nerve hyperexcitability but also develop insomnia, confusion, hallucinations, and memory loss. Drawing on the clinical overlap between Morvan syndrome and limbic encephalitis, Buckley et al30 were the first to report VGKC complex antibodies in two cases of limbic encephalitis.
VGKC complex antibodies are now understood to be associated with a wide variety of neurologic conditions, including chronic idiopathic pain, epilepsy,31 movement disorders, cranial nerve abnormalities, autonomic dysfunction,32 and gut dysmotility.33 In contrast, these antibodies are rare in healthy people.34 Limbic encephalitis associated with VGKC complex antibody usually lacks cerebellar and brainstem dysfunction, which may help distinguish it from other types of autoantibody-mediated limbic encephalitis.12
VGKC complex antibody does not bind to the potassium channel itself. Instead it recognizes other constituents of the channel complex, most notably LGI1 and contactin-associated protein 2 (CASPR2). LGI1 antibody is more commonly associated with limbic encephalitis—as illustrated in our case study—in addition to a distinctive type of seizure affecting the arm and face (faciobrachial dystonic seizure).34 The CASPR2 antibody, on the other hand, more often correlates with peripheral nerve manifestations and Morvan syndrome.29 Hyponatremia is commonly seen on serum chemical analysis and provides a clue that these syndromes are present.12
Good response to immunotherapy
A critical change in therapy came as clinicians realized that seizures were often refractory to standard antiepileptic drugs but responded well to immunotherapies. On the basis of these observations, sera of patients with long-standing epilepsy have been reanalyzed to look for neuronal autoantibodies.31 These antibodies should be checked in cases of new-onset refractory status epilepticus of unknown origin that does not respond to antiepileptic medications.
About half of patients with VGKC complex antibody-mediated limbic encephalitis have normal findings on brain MRI.5 Seven of 10 patients who were prospectively followed for VGKC complex antibody-mediated faciobrachial dystonic seizures had normal brain MRIs.35
VGKC complex antibody-mediated limbic encephalitis does not usually recur.36 Most cases are nonparaneoplastic, as evidenced by failure to detect a single active tumor in 64 patients after a median follow-up of 3 years. The prognosis is generally favorable except in cases with coexisting tumors.12
ANTI-NMDA RECEPTOR ENCEPHALITIS
Often associated with ovarian teratoma
Anti-NMDA receptor encephalitis typically affects women in their 20s and 30s, and about half of patients have an ovarian teratoma. It can also occur in younger patients and in men, in whom it is less likely to be associated with a neoplasm.37
Typical initial symptoms include striking and often stereotyped neuropsychiatric disturbances manifesting as psychosis, confusion, seizures, and amnesia. After 1 to 2 weeks, new symptoms set in, including reduced consciousness, movement disorders (ranging from orolingualfacial dyskinesia to rigidity and choreoathetosis), autonomic dysfunction, and hypoventilation, often prompting admission to the intensive care unit.38
Although the outcome is favorable in most cases, recovery, in contrast to VGKC complex antibody-mediated limbic encephalitis, is slow and may take longer than 1 year. Up to a quarter of patients have a relapse, underscoring the importance of maintenance immunotherapy.
It is important to undertake an intensive search for possible ovarian and extraovarian teratomas in young women with this syndrome—including CT of the pelvis, vaginal ultrasonography, and PET imaging—as removal of the teratoma may be curative.37
DIAGNOSIS OF AUTOANTIBODY-MEDIATED ENCEPHALITIS
Critical to diagnosing autoantibody-mediated encephalitis is awareness of these disorders. Since antibody testing may be very specific and is not usually part of the standard batteries of tests, a high level of suspicion is needed. Patients may present to different specialists in different settings; therefore, clinicians in pediatrics, rheumatology, psychiatry, and intensive care medicine need to be aware of these syndromes to avoid delay and misdiagnosis.
Clinical features suggesting autoantibody-mediated encephalitis include:
- Acute or subacute onset of a neurologic syndrome
- New-onset refractory status epilepticus of unknown etiology
- Acute or subacute psychiatric illness with unexpected progression to neurologic symptoms or delirium
- Unusual movement disorders not conforming to standard syndromes
- Cognitive impairment, psychosis, or behavioral or language disorders with atypical findings on imaging or cerebrospinal fluid analysis.
Imaging. Diagnosis of autoantibody-mediated encephalitis focuses on evidence suggesting an inflammatory central nervous system syndrome. MRI may show hyperintense signals on T2, FLAIR, or diffusion-weighted imaging changes in various brain regions. In many cases, however, MRI is negative despite severe clinical symptoms. In a study of 72 patients suspected of having autoimmune dementia of various etiologies, including but not restricted to antineuronal surface antibody-mediated causes, Flanagan et al39 identified atypical neuroimaging findings in only 29%. PET imaging may show hypermetabolism in certain brain areas correlating to clinical syndromes but is often difficult to obtain in a timely fashion.
Cerebrospinal fluid is often abnormal, showing elevated protein, increased immunoglobulin G synthesis, or oligoclonal banding. As with imaging studies, the cerebrospinal fluid may be normal despite severe clinical manifestations.
Electroencephalography may show focal slowing or seizure activity. Neuropsychologic testing may show different patterns of abnormalities.
Antibody testing. None of these tests can be used in isolation, and the diagnosis of autoantibody-mediated encephalitis hinges on recognizing a clinical syndrome and ordering supportive testing. Specific antibodies are more likely in different clinical syndromes and should be sought (Table 3).
Patients who have autoantibody-mediated encephalitis may test negative for autoantibodies for many possible reasons:
- Blood testing for antibodies may be less sensitive than cerebrospinal fluid testing
- Antibody titers may vary in the course of the disease
- The patient may be expressing an antibody that is less often tested for (eg, anti-AMPA receptor or antigamma-aminobutyric acid B) or one that has not yet been isolated.
Evaluating for malignancy is recommended in all cases of autoantibody-mediated encephalitis. The initial workup may involve CT of the chest, abdomen, and pelvis, as well as mammography in women and serum prostate-specific antigen testing and testicular ultrasonography in men. Ordering FDG-PET in cases in which CT is negative or inconclusive increases cancer detection.40 If no cancer is found, close tumor surveillance—every 3 to 6 months—is recommended for at least 2 years.41
TREATMENT
Owing in large part to the rarity of autoantibody-mediated encephalitides, no randomized trials of therapy have been performed. Treatment at present is guided mostly by case series and expert consensus, which suggest first-line therapy with intravenous immunoglobulin, high-dose corticosteroids, plasmapheresis, or a combination.
Different syndromes and antibody-related disorders respond differently to therapy. Syndromes associated with antibodies against intracellular antigens tend to be more resistant to immune therapy than cell surface antigen-related syndromes.4
Tiered approach
Combined treatment with intravenous immunoglobulin and high-dose corticosteroids may be superior to treatment with steroids alone for LGI1-antibody mediated limbic encephalitis.42
In cases refractory to first-line (“tier 1”) therapy, second-line immunotherapy with drugs affecting B-cell populations (eg, rituximab, cyclophosphamide, and mycophenolate mofetil) has been used.
A tiered approach has been most extensively studied for anti-NMDA-receptor encephalitis, with better outcomes found using second-line therapy.43
Treatment strategies for these disorders will likely evolve over time with additional experience.
Outpatient management
Once the patient is discharged from the hospital, a multidisciplinary approach to care is recommended, including physical rehabilitation, speech therapy, neuropsychiatric and neuroimmunologic follow-up, and annual surveillance for malignancies.
- Brierley JB, Corsellis JAN, Hierons R, Nevin S. Subacute encephalitis of later adult life mainly affecting the limbic areas. Brain 1960; 83:357–368.
- Corsellis JA, Goldberg GJ, Norton AR. “Limbic encephalitis” and its association with carcinoma. Brain 1968; 91:481–496.
- Greenlee JE, Brashear HR. Antibodies to cerebellar Purkinje cells in patients with paraneoplastic cerebellar degeneration and ovarian carcinoma. Ann Neurol 1983; 14:609–613.
- Rosenfeld MR, Dalmau JO. Paraneoplastic disorders of the CNS and autoimmune synaptic encephalitis. Continuum (Minneap Minn) 2012; 18:366–383.
- Irani SR, Gelfand JM, Al-Diwani A, Vincent A. Cell-surface central nervous system autoantibodies: clinical relevance and emerging paradigms. Ann Neurol 2014; 76:168–184.
- Dalmau J, Tüzün E, Wu HY, et al. Paraneoplastic anti-N-methyl-D-aspartate receptor encephalitis associated with ovarian teratoma. Ann Neurol 2007; 61:25–36.
- Irani SR, Alexander S, Waters P, et al. Antibodies to Kv1 potassium channel-complex proteins leucine-rich, glioma inactivated 1 protein and contactin-associated protein-2 in limbic encephalitis, Morvan’s syndrome and acquired neuromyotonia. Brain 2010; 133:2734–2748.
- Lancaster E, Lai M, Peng X, et al. Antibodies to the GABA(B) receptor in limbic encephalitis with seizures: case series and characterisation of the antigen. Lancet Neurol 2010; 9:67–76.
- Lai M, Hughes EG, Peng X, et al. AMPA receptor antibodies in limbic encephalitis alter synaptic receptor location. Ann Neurol 2009; 65:424–434.
- Zuliani L, Graus F, Giometto B, Bien C, Vincent A. Central nervous system neuronal surface antibody associated syndromes: review and guidelines for recognition. J Neurol Neurosurg Psychiatry 2012; 83:638–645.
- Ances BM, Vitaliani R, Taylor RA, et al. Treatment-responsive limbic encephalitis identified by neuropil antibodies: MRI and PET correlates. Brain 2005; 128:1764–1777.
- Vincent A, Buckley C, Schott JM, et al. Potassium channel antibody-associated encephalopathy: a potentially immunotherapy-responsive form of limbic encephalitis. Brain 2004; 127:701–712.
- Jammoul A, Lederman RJ, Tavee J, Li Y. Presence of voltage-gated potassium channel complex antibody in a case of genetic prion disease. BMJ Case Rep 2014; pii:bcr2013201622.
- Angus-Leppan H, Rudge P, Mead S, Collinge J, Vincent A. Autoantibodies in sporadic Creutzfeldt-Jakob disease. JAMA Neurol 2013; 70:919–922.
- Fujita K, Yuasa T, Watanabe O, et al. Voltage-gated potassium channel complex antibodies in Creutzfeldt-Jakob disease. J Neurol 2012; 259:2249–2250.
- Fujita K, Yuasa T, Takahashi Y, et al. Antibodies to N-methyl-D-aspartate glutamate receptors in Creutzfeldt–Jakob disease patients. J Neuroimmunol 2012; 251:90–93.
- Mackay G, Ahmad K, Stone J, et al. NMDA receptor autoantibodies in sporadic Creutzfeldt-Jakob disease. J Neurol 2012; 259:1979–1981.
- Leypoldt F, Titulaer MJ, Aguilar E, et al. Herpes simplex virus–1 encephalitis can trigger anti-NMDA receptor encephalitis: case report. Neurology 2013; 81:1637–1639.
- Desena A, Graves D, Warnack W, Greenberg BM. Herpes simplex encephalitis as a potential cause of anti-N-methyl-D-aspartate receptor antibody encephalitis: report of 2 cases. JAMA Neurol 2014; 71:344–346.
- Armangue T, Leypoldt F, Málaga I, et al. Herpes simplex virus encephalitis is a trigger of brain autoimmunity. Ann Neurol 2014; 75:317–323.
- Blaes F. Paraneoplastic brain stem encephalitis. Curr Treat Options Neurol 2013; 15:201–209.
- Wildemann B, Jarius S. The expanding range of autoimmune disorders of the nervous system. Lancet Neurol 2013; 12:22–24.
- Kim W, Kim SH, Lee SH, Li XF, Kim HJ. Brain abnormalities as an initial manifestation of neuromyelitis optica spectrum disorder. Mult Scler 2011; 17:1107–1112.
- Shahrizaila N, Yuki N. Bickerstaff brainstem encephalitis and Fisher syndrome: anti-GQ1b antibody syndrome. J Neurol Neurosurg Psychiatry 2013; 84:576–583.
- Jubelt B, Mihai C, Li MT, Veerapaneni P. Rhombencephalitis/brainstem encephalitis. Curr Neurol Neurosci Rep 2011; 11:543–552.
- Sillevis Smitt P, Kinoshita A, De Leeuw B, et al. Paraneoplastic cerebellar ataxia due to autoantibodies against a glutamate receptor. N Engl J Med 2000; 342:21–27.
- Ishida K, Mitoma H, Son SY, et al. Selective suppression of cerebellar GABAergic transmission by an autoantibody to glutamic acid decarboxylase. Ann Neurol 1999; 46:263–267.
- Hart IK, Waters C, Vincent A, et al. Autoantibodies detected to expressed K+ channels are implicated in neuromyotonia. Ann Neurol 1997; 41:238–246.
- Barber P, Anderson NE, Vincent A. Morvan’s syndrome associated with voltage-gated K+ channel antibodies. Neurology 2000; 54:771–772.
- Buckley C, Oger J, Clover L, et al. Potassium channel antibodies in two patients with reversible limbic encephalitis. Ann Neurol 2001; 50:73–78.
- Majoie HJ, de Baets M, Renier W, Lang B, Vincent A. Antibodies to voltage-gated potassium and calcium channels in epilepsy. Epilepsy Res 2006; 71:135–141.
- Tan KM, Lennon VA, Klein CJ, Boeve BF, Pittock SJ. Clinical spectrum of voltage-gated potassium channel autoimmunity. Neurology 2008; 70:1883–1890.
- Knowles CH, Lang B, Clover L, et al. A role for autoantibodies in some cases of acquired non-paraneoplastic gut dysmotility. Scand J Gastroenterol 2002; 37:166–170.
- Irani SR, Michell AW, Lang B, et al. Faciobrachial dystonic seizures precede Lgi1 antibody limbic encephalitis. Ann Neurol 2011; 69:892–900.
- Irani SR, Stagg CJ, Schott JM, et al. Faciobrachial dystonic seizures: the influence of immunotherapy on seizure control and prevention of cognitive impairment in a broadening phenotype. Brain 2013: 136:3151–3162.
- Vincent A, Bien CG, Irani SR, Waters P. Autoantibodies associated with diseases of the CNS: new developments and future challenges. Lancet Neurol 2011; 10:759–772.
- Dalmau J, Lancaster E, Martinez-Hernandez E, Rosenfeld MR, Balice-Gordon R. Clinical experience and laboratory investigations in patients with anti-NMDAR encephalitis. Lancet Neurol 2011; 10:63–74.
- Irani SR, Bera K, Waters P, et al. N-methyl-D-aspartate antibody encephalitis: temporal progression of clinical and paraclinical observations in a predominantly non-paraneoplastic disorder of both sexes. Brain 2010; 133:1655–1667.
- Flanagan EP, McKeon A, Lennon VA, et al. Autoimmune dementia: clinical course and predictors of immunotherapy response. Mayo Clin Proc 2010; 85:881–897.
- Younes-Mhenni S, Janier MF, Cinotti L, et al. FDG-PET improves tumour detection in patients with paraneoplastic neurological syndromes. Brain 2004; 127:2331–2338.
- Lancaster E, Martinez-Hernandez E, Dalmau J. Encephalitis and antibodies to synaptic and neuronal cell surface proteins. Neurology 2011; 77:179–189.
- Shin YW, Lee ST, Shin JW, et al. VGKC-complex/LGI1-antibody encephalitis: clinical manifestations and response to immunotherapy. J Neuroimmunol 2013; 265:75–81.
- Titulaer MJ, McCracken L, Gabilondo I, et al. Treatment and prognostic factors for long-term outcome in patients with anti-NMDA receptor encephalitis: an observational cohort study. Lancet Neurol 2013; 12:157–165.
- Brierley JB, Corsellis JAN, Hierons R, Nevin S. Subacute encephalitis of later adult life mainly affecting the limbic areas. Brain 1960; 83:357–368.
- Corsellis JA, Goldberg GJ, Norton AR. “Limbic encephalitis” and its association with carcinoma. Brain 1968; 91:481–496.
- Greenlee JE, Brashear HR. Antibodies to cerebellar Purkinje cells in patients with paraneoplastic cerebellar degeneration and ovarian carcinoma. Ann Neurol 1983; 14:609–613.
- Rosenfeld MR, Dalmau JO. Paraneoplastic disorders of the CNS and autoimmune synaptic encephalitis. Continuum (Minneap Minn) 2012; 18:366–383.
- Irani SR, Gelfand JM, Al-Diwani A, Vincent A. Cell-surface central nervous system autoantibodies: clinical relevance and emerging paradigms. Ann Neurol 2014; 76:168–184.
- Dalmau J, Tüzün E, Wu HY, et al. Paraneoplastic anti-N-methyl-D-aspartate receptor encephalitis associated with ovarian teratoma. Ann Neurol 2007; 61:25–36.
- Irani SR, Alexander S, Waters P, et al. Antibodies to Kv1 potassium channel-complex proteins leucine-rich, glioma inactivated 1 protein and contactin-associated protein-2 in limbic encephalitis, Morvan’s syndrome and acquired neuromyotonia. Brain 2010; 133:2734–2748.
- Lancaster E, Lai M, Peng X, et al. Antibodies to the GABA(B) receptor in limbic encephalitis with seizures: case series and characterisation of the antigen. Lancet Neurol 2010; 9:67–76.
- Lai M, Hughes EG, Peng X, et al. AMPA receptor antibodies in limbic encephalitis alter synaptic receptor location. Ann Neurol 2009; 65:424–434.
- Zuliani L, Graus F, Giometto B, Bien C, Vincent A. Central nervous system neuronal surface antibody associated syndromes: review and guidelines for recognition. J Neurol Neurosurg Psychiatry 2012; 83:638–645.
- Ances BM, Vitaliani R, Taylor RA, et al. Treatment-responsive limbic encephalitis identified by neuropil antibodies: MRI and PET correlates. Brain 2005; 128:1764–1777.
- Vincent A, Buckley C, Schott JM, et al. Potassium channel antibody-associated encephalopathy: a potentially immunotherapy-responsive form of limbic encephalitis. Brain 2004; 127:701–712.
- Jammoul A, Lederman RJ, Tavee J, Li Y. Presence of voltage-gated potassium channel complex antibody in a case of genetic prion disease. BMJ Case Rep 2014; pii:bcr2013201622.
- Angus-Leppan H, Rudge P, Mead S, Collinge J, Vincent A. Autoantibodies in sporadic Creutzfeldt-Jakob disease. JAMA Neurol 2013; 70:919–922.
- Fujita K, Yuasa T, Watanabe O, et al. Voltage-gated potassium channel complex antibodies in Creutzfeldt-Jakob disease. J Neurol 2012; 259:2249–2250.
- Fujita K, Yuasa T, Takahashi Y, et al. Antibodies to N-methyl-D-aspartate glutamate receptors in Creutzfeldt–Jakob disease patients. J Neuroimmunol 2012; 251:90–93.
- Mackay G, Ahmad K, Stone J, et al. NMDA receptor autoantibodies in sporadic Creutzfeldt-Jakob disease. J Neurol 2012; 259:1979–1981.
- Leypoldt F, Titulaer MJ, Aguilar E, et al. Herpes simplex virus–1 encephalitis can trigger anti-NMDA receptor encephalitis: case report. Neurology 2013; 81:1637–1639.
- Desena A, Graves D, Warnack W, Greenberg BM. Herpes simplex encephalitis as a potential cause of anti-N-methyl-D-aspartate receptor antibody encephalitis: report of 2 cases. JAMA Neurol 2014; 71:344–346.
- Armangue T, Leypoldt F, Málaga I, et al. Herpes simplex virus encephalitis is a trigger of brain autoimmunity. Ann Neurol 2014; 75:317–323.
- Blaes F. Paraneoplastic brain stem encephalitis. Curr Treat Options Neurol 2013; 15:201–209.
- Wildemann B, Jarius S. The expanding range of autoimmune disorders of the nervous system. Lancet Neurol 2013; 12:22–24.
- Kim W, Kim SH, Lee SH, Li XF, Kim HJ. Brain abnormalities as an initial manifestation of neuromyelitis optica spectrum disorder. Mult Scler 2011; 17:1107–1112.
- Shahrizaila N, Yuki N. Bickerstaff brainstem encephalitis and Fisher syndrome: anti-GQ1b antibody syndrome. J Neurol Neurosurg Psychiatry 2013; 84:576–583.
- Jubelt B, Mihai C, Li MT, Veerapaneni P. Rhombencephalitis/brainstem encephalitis. Curr Neurol Neurosci Rep 2011; 11:543–552.
- Sillevis Smitt P, Kinoshita A, De Leeuw B, et al. Paraneoplastic cerebellar ataxia due to autoantibodies against a glutamate receptor. N Engl J Med 2000; 342:21–27.
- Ishida K, Mitoma H, Son SY, et al. Selective suppression of cerebellar GABAergic transmission by an autoantibody to glutamic acid decarboxylase. Ann Neurol 1999; 46:263–267.
- Hart IK, Waters C, Vincent A, et al. Autoantibodies detected to expressed K+ channels are implicated in neuromyotonia. Ann Neurol 1997; 41:238–246.
- Barber P, Anderson NE, Vincent A. Morvan’s syndrome associated with voltage-gated K+ channel antibodies. Neurology 2000; 54:771–772.
- Buckley C, Oger J, Clover L, et al. Potassium channel antibodies in two patients with reversible limbic encephalitis. Ann Neurol 2001; 50:73–78.
- Majoie HJ, de Baets M, Renier W, Lang B, Vincent A. Antibodies to voltage-gated potassium and calcium channels in epilepsy. Epilepsy Res 2006; 71:135–141.
- Tan KM, Lennon VA, Klein CJ, Boeve BF, Pittock SJ. Clinical spectrum of voltage-gated potassium channel autoimmunity. Neurology 2008; 70:1883–1890.
- Knowles CH, Lang B, Clover L, et al. A role for autoantibodies in some cases of acquired non-paraneoplastic gut dysmotility. Scand J Gastroenterol 2002; 37:166–170.
- Irani SR, Michell AW, Lang B, et al. Faciobrachial dystonic seizures precede Lgi1 antibody limbic encephalitis. Ann Neurol 2011; 69:892–900.
- Irani SR, Stagg CJ, Schott JM, et al. Faciobrachial dystonic seizures: the influence of immunotherapy on seizure control and prevention of cognitive impairment in a broadening phenotype. Brain 2013: 136:3151–3162.
- Vincent A, Bien CG, Irani SR, Waters P. Autoantibodies associated with diseases of the CNS: new developments and future challenges. Lancet Neurol 2011; 10:759–772.
- Dalmau J, Lancaster E, Martinez-Hernandez E, Rosenfeld MR, Balice-Gordon R. Clinical experience and laboratory investigations in patients with anti-NMDAR encephalitis. Lancet Neurol 2011; 10:63–74.
- Irani SR, Bera K, Waters P, et al. N-methyl-D-aspartate antibody encephalitis: temporal progression of clinical and paraclinical observations in a predominantly non-paraneoplastic disorder of both sexes. Brain 2010; 133:1655–1667.
- Flanagan EP, McKeon A, Lennon VA, et al. Autoimmune dementia: clinical course and predictors of immunotherapy response. Mayo Clin Proc 2010; 85:881–897.
- Younes-Mhenni S, Janier MF, Cinotti L, et al. FDG-PET improves tumour detection in patients with paraneoplastic neurological syndromes. Brain 2004; 127:2331–2338.
- Lancaster E, Martinez-Hernandez E, Dalmau J. Encephalitis and antibodies to synaptic and neuronal cell surface proteins. Neurology 2011; 77:179–189.
- Shin YW, Lee ST, Shin JW, et al. VGKC-complex/LGI1-antibody encephalitis: clinical manifestations and response to immunotherapy. J Neuroimmunol 2013; 265:75–81.
- Titulaer MJ, McCracken L, Gabilondo I, et al. Treatment and prognostic factors for long-term outcome in patients with anti-NMDA receptor encephalitis: an observational cohort study. Lancet Neurol 2013; 12:157–165.
KEY POINTS
- Autoantibody-mediated encephalitis accounts for a portion of cases of unexplained status epilepticus, encephalitis, and acute-onset psychiatric symptoms.
- Magnetic resonance imaging and cerebrospinal fluid analysis may be normal early in the disease course.
- Patients can express more than one autoantibody and present with more than one neuronal syndrome.
- Syndromes in which antibodies attack antigens on the surface of neurons are more likely to respond to immunotherapy than those involving intracellular antigens.
- Anti-N-methyl-d-aspartate receptor encephalitis typically presents with psychosis, seizures, and movement disorders in young women and is often associated with an ovarian teratoma.
- Limbic encephalitis, mediated by antibody to the voltage-gated potassium channel complex, is typically nonneoplastic and responds well to immunotherapy.
Not all abdominal pain is gastrointestinal
A 31-year-old woman presents to the office with a chief complaint of right mid-abdominal pain that began 1 day ago. She says she did not seek medical attention earlier because she had to be at work that morning and she thought the pain would resolve on its own.
She reports no fever, headache, anorexia, nausea, vomiting, malaise, loss of weight, melena, or changes in bowel habits. She describes the pain as sharp, localized to the right side, and radiating to the vulva upon sitting up. She denies any association of pain with current dietary habits or bowel function. She has no recollection of precipitating or alleviating factors, including the use of analgesics to reduce the pain.
On further discussion, she mentions that 1 year ago she began experiencing chronic abdominal pain, which she says is sometimes exacerbated by coughing, by standing for extended periods of time, and during menses, and is alleviated upon lying down.
She has regular menstrual periods, and her last one ended 7 days ago.
Her surgical history includes two uncomplicated cesarean deliveries. She does not use tobacco, alcohol, or illicit substances. She is not aware of any allergies to drugs or foods.
She appears to be in no acute distress and has been sitting quietly thus far. She seems to have positioned her hand on her abdomen over the corresponding area of pain.
On physical examination, vital signs are within normal limits, and she is alert and oriented to person, place, and time. Her sclerae are anicteric, and the pupils are equal, round, and reactive to light.
Cardiovascular and pulmonary examinations are also within normal limits. Examination of the abdomen elicits tenderness and guarding along the lateral border of the rectus abdominis muscle on the right side at the level of umbilicus, with no rebound tenderness or rigidity. The liver and spleen are not enlarged, and no abdominal mass is detected. No skin rash, joint swelling, or peripheral edema is noted. A neurologic examination is normal.
1. With the information provided, which of the following is least likely to be causing her symptoms?
- Chronic mesenteric ischemia
- Peptic ulcer
- Acute cholecystitis
- Slipping rib syndrome
CHRONIC MESENTERIC ISCHEMIA
Chronic mesenteric ischemia is the least likely diagnosis because the patient lacks risk factors for atherosclerosis and because she does not have postprandial pain, which is pathognomonic for chronic mesenteric ischemia. It is thought to be caused by a decrease in blood flow through the splanchnic vessels.1 Symptoms tend to arise after eating because of a postprandial increase in metabolic demands.1 These patients also often have atherosclerotic risk factors such as hypertension, hyperlipidemia, and smoking causing coronary artery disease, or a history of stroke.
The primary symptom is abdominal pain, most often described as achy, crampy, or spastic episodes of pain, usually occurring within 2 hours of eating.2 Weight loss is common, as patients can develop a fear of eating. Postprandial pain may also be associated with nausea, vomiting, and bloating.
Findings on clinical examination are usually less severe than the actual symptoms. Visceral duplex or multidetector computed tomography (CT) is an excellent tool to detect blood flow in potential stenotic vessels.2
PEPTIC ULCER DISEASE
Peptic ulcer disease is not a likely diagnosis in this patient because she has no history of taking nonsteroidal anti-inflammatory drugs (NSAIDs).
A study of US patients between 1997 and 2007 reported an annual incidence of peptic ulcer disease of 0.05% to 0.19% depending on the method of diagnosis.3 Peptic ulcer is thought to result from increased gastric acid secretion with a resultant inflammatory response, leading to erosion and ulceration.
The most common possible catalysts include Helicobacter pylori infection, NSAIDs, smoking, alcohol use, and hypersecretory states such as Zollinger-Ellison syndrome.4–6 Complications include internal bleeding, perforation causing peritonitis, and penetration to adjacent organs.
Pathophysiology
Peptic ulcer is the result of an increase in the normal level of gastric acid and a decrease in the protective ability of the gastric mucosa.7 Cytoprotection may be lost through a decrease in the products of arachidonic acid metabolism (eg, prostaglandins, which have a protective effect) or an increase in leukotriene B4 (LTB4), which has a damaging effect. Prostaglandins are thought not only to protect the normal gastric mucosa, but also to provide an antisecretory effect.
On the other hand, leukotrienes—specifically LTB4 and LTC4—are proinflammatory agents and can damage the gastric mucosa. NSAIDs enhance the production of leukotrienes through the 5-lipoxygenase pathway. The ability of LTB4 to cause degranulation and release of lysosomal enzymes may play a vital role in the inflammatory response to NSAIDs.8–10 LTC4 may promote gastric mucosal damage through a reduction of tissue perfusion resulting from the promotion of vascular stasis.8,11,12
Symptoms help differentiate ulcer type
The classic symptom is burning epigastric pain after meals. Pain that occurs immediately after meals is a classic symptom of gastric ulcer. Pain that occurs 2 to 3 hours after meals and that is relieved by food or antacids is a strong indicator of duodenal ulcer.13 Other symptoms include dyspepsia, bloating, distention, heartburn, and chest discomfort.13
Accurate diagnosis is vital in selecting the proper treatment. Diagnostic tests may include H pylori testing, upper-gastrointestinal endoscopy, and radiography with barium swallow.
CHOLECYSTITIS
In cholecystitis, the primary complaint is pain, usually in the right upper quadrant of the abdomen. Patients describe sudden, sharp, and intense pain that radiates to the back or shoulder. Patients may report pain after heavy meals, and some report nausea and vomiting. Cholecystitis is in the differential diagnosis of this patient because of the anatomic location of her pain.
The diagnosis is confirmed by imaging. Abdominal ultrasonography, technetium-99m hepatic iminodiacetic acid scanning, and CT are the most commonly used studies.14
Cholecystitis can be acute or chronic. Acute cholecystitis is categorized as calculous or acalculous. Calculous cholecystitis is multifactorial, but the primary cause is blockage of the cystic duct by gallstones.15 Other factors include irritants such as lysolecithin (released during bile stasis), which can trigger gallbladder inflammation,15–17 and infection.18
When the cystic duct is blocked, bile builds up inside the gallbladder, causing irritation and inflammation of the walls of the gallbladder.14
Acalculous cholecystitis, which resembles calculous cholecystitis but without the gallstones,19 accounts for 2% to 15% of all cases of acute cholecystitis.19,20 It has been observed in hospitalized critically ill patients, but it can also present in an outpatient setting, most often in elderly men with vascular disease.21 Causes include infection, trauma, and tumor obstruction, resulting in endothelial injury, gallbladder stasis, ischemia, and eventually necrosis.14,20,22,23
SLIPPING RIB SYNDROME
Slipping rib syndrome, also known as Tietze syndrome, is believed to be caused by hypermobile costal cartilage. The affected rib slips behind the rib above on contraction of the abdominal wall. This displacement increases the probability of costal nerve impingement and tissue inflammation producing unilateral, sharp, subcostal and upper-abdominal pain.
In this patient, slipping rib syndrome is a possible diagnosis because of the location of the pain and because the pain described by the patient is highly suggestive of neuropathic pain.
Slipping rib syndrome is diagnosed clinically by a “hooking” maneuver: the clinician hooks his or her fingers at the patient’s subcostal area, reproducing the pain by movement of the ribs anteriorly.24 When this test is performed in our patient the result is negative, ruling out slipping rib syndrome.
THE WORKUP CONTINUES
A complete blood cell count and comprehensive metabolic panel are within normal limits. Abdominal duplex ultrasonography reveals no celiac or mesenteric occlusions, thus ruling out chronic mesenteric ischemia.
Noncontrast CT shows no renal or ureteric stones and no evidence of bleeding in the urinary tract. CT with contrast shows no bowel distention, no evidence of hernia, and a normal appendix and ovaries.
2. After exclusion of the previous choices, which of the following is the most likely cause of her symptoms?
- Anterior cutaneous nerve entrapment syndrome (ACNES)
- Ovarian cyst
- Renal stones
- Appendicitis
- Ventral hernia
- Median arcuate ligament syndrome
ANTERIOR CUTANEOUS NERVE ENTRAPMENT SYNDROME
ACNES is the most likely diagnosis. A study published in 2013 indicated that many cases of functional abdominal pain may actually be undiagnosed cases of chronic abdominal wall pain such as ACNES.25 The condition, first described in 1972,26 is thought to be caused by thoracic cutaneous intercostal nerve entrapment between the abdominal muscles, causing pain at the point of entrapment.
The patient may present with pain that is either acute or chronic. Acute pain is localized more on the right side close to an old scar, or at the outer edge of the rectus abdominis muscle. The pain may vary from dull to burning to sharp; it can radiate horizontally in the upper half of the abdomen or obliquely in the lower half of the abdomen with movements such as twisting and sitting up.27
Despite the acute pain, patients are able to carry on daily functions. The pain may be alleviated by lying down.
The pain may be misdiagnosed as gynecologic or renal. In younger men, the pain may raise concern about hernia, and in older patients, cancer.27 Patients may complain of chronic intermittent pain, usually unilateral, and to a lesser extent bilateral.27
The anatomic location of the pain usually reflects the intercostal nerve involved. The pain is not related to eating or to bowel movements.25 Some patients report exacerbation upon coughing or standing, during menses, and with use of oral contraceptives.28,29 When inquiring about surgical history, it is common to find that the patient has had multiple abdominal surgical procedures.
On examination, the patient has nondistressing pain, with a hand often placed over the painful area.27 On firm palpation, a tender spot of less than 2 cm can be detected.
The diagnosis can be confirmed with a positive Carnett test. The patient lies supine on the examination table with the arms crossed over the chest, then elevates the head or the feet to tense the abdominal muscles.26,27 If doing so reproduces the pain (ie, a positive test), this increases the suspicion of ACNES; if the pain decreases or is not reproducible, an intra-abdominal cause is more likely.
If the pain is difficult to localize, the “pinch test” can be done by using the thumb and index finger to pinch and lift the skin of the abdomen, including the subcutaneous layer of fat, first on one side and then on the other. This helps determine the side with greater pain.27
OVARIAN CYSTS
Ovarian cysts are fluid-filled sacs on the surface of or within the ovary. They are often benign and require no intervention. However, 5% to 10% of US women with a suspicious ovarian mass undergo a surgical procedure, and 13% to 21% of these are found to have a malignancy.30,31
Ovarian cysts are usually painless unless complicated by rupture or bleeding. Patients who present with pain describe it as dull and aching and in the abdomen or pelvis. In rare cases, ovarian cysts can be large enough to cause pain from torsion. Other symptoms may include delayed menses and bleeding outside of the menstrual period.32–34
Ovarian cysts are thought to be caused by hormonal changes during the menstrual cycle. They can be detected during pelvic examination or during pelvic ultrasonography. Cysts that are primarily fluid-filled are generally benign and require no intervention. On the other hand, cysts composed of solid material require intervention.
Treatment depends on several factors, including size and type of cyst, the patient’s age, and whether torsion is present. Treatment can range from observation to medical or surgical management. Laparoscopic surgery is commonly used when surgical treatment is warranted.
RENAL STONES
From 10% to 15% of US adults develop a kidney stone at some time during their life.35 There is no single cause, but one factor that promotes stone formation is a greater amount of crystal-forming substances in the urine, such as calcium, oxalate, and uric acid.36 Most renal stones are calcium oxalate, uric acid, struvite, or cysteine.
Symptoms arise when the stone moves within the urinary tract. Patients present to the emergency room in severe distress, usually with flank pain that radiates to the lower abdomen or groin. The pain is episodic, fluctuates in intensity, and may present with dysuria, frequency, or urgency. It is also associated with nausea and vomiting.37
Renal stones are diagnosed through a series of laboratory and imaging studies. Imaging studies include plain radiography (which can miss small stones), renal sonography, and computed tomography without contrast.
APPENDICITIS
In the United States, the lifetime risk of developing appendicitis is 8.6% in men and 6.7% in women.38 Appendicitis is one of the most common reasons for emergency surgery.
Appendicitis is thought to result from obstruction by fecal matter blocking the opening of the appendix or from a viral infection (eg, with an adenovirus).39,40 The resulting bacterial growth can cause the appendix to become inflamed and purulent.
Patients typically present with umbilical or epigastric pain radiating to the right lower quadrant of the abdomen. Over time, the pain becomes sharper. Certain movements can exacerbate the pain, and lying down may alleviate it. Other symptoms are nausea, vomiting, loss of appetite, and low-grade fever.
Findings on the abdominal examination that help to confirm the diagnosis include rigidity and tenderness, classically localized to a point two-thirds of the way from the umbilicus to the anterior superior iliac spine. Rebound tenderness is usually present. Up to 25% of cases in some series presented atypically, with variable location and findings on physical examination (eg, bowel irregularities, indigestion, flatulence, generalized malaise). In addition to the physical examination, laboratory testing and imaging (ultrasonography, CT) may aid in confirming the diagnosis of appendicitis or any other cause of the pain.38
VENTRAL HERNIA
Ventral hernia is a bulging of abdominal organs or other tissues through a defect of the musculature of the abdominal wall. Ventral hernia is categorized by its location as epigastric, abdominal, or incisional. An open abdominal procedure is the cause in nearly 10% of cases41; the herniation occurs with weakening of the surgical scar.
Ventral hernia is usually detected on physical examination, and patients may present after noting a bulge in the abdominal wall. Symptoms vary. Some patients have no symptoms, while others have mild abdominal discomfort or severe abdominal pain as well as nausea and vomiting. Imaging with CT, ultrasonography, or magnetic resonance imaging helps confirm the diagnosis. Complications of ventral hernia include incarceration and bowel strangulation.
MEDIAN ARCUATE LIGAMENT SYNDROME
Median arcuate ligament syndrome is a challenging diagnosis and a very rare cause of abdominal pain. It is thought to be caused by celiac artery compression by fibroligamentous bands. Pain fluctuates with respiration and is greater during expiration.
Patients may present with recurrent episodes of crampy postprandial pain that cause them to avoid eating, resulting in weight loss. The pain may be associated with nausea, vomiting, and bloating.
The diagnosis is confirmed by duplex ultrasonography, angiography, or magnetic resonance angiography. Treatment is surgical division of the fibroligamentous band and crus, and this is often done laparascopically. In patients with severe persistent celiac artery stenosis, angioplasty and stenting may be considered.2
CASE CONTINUED
Before the physical examination, our patient identifies the location of her pain. A Carnett test is performed, as for ACNES: the patient is placed in the supine position and is instructed to cross both arms over her chest. In an effort to promote muscle tension, she is asked to elevate her head off the examination table, as if performing a mini sit-up, and as she does this, pressure is applied to the identified tender area. The pain is easily reproduced, further confirming involvement of the abdominal wall rather than the viscera. After this, electromyography shows abnormal findings. The patient is then referred to the pain management clinic for a diagnostic nerve block.
3. Which of the following is the first-line treatment of ACNES?
- Local injection of anesthetic
- Surgical neurectomy
LOCAL INJECTION OF ANESTHETIC
Local injection of anesthetic is the first-line treatment of ACNES.
Since ACNES is underdiagnosed, the patient may be less likely to be familiar with it. He or she should receive a detailed explanation of the condition and its management; this will help achieve a successful outcome.
Local anesthetic injection is used for both diagnosis and treatment; 2% lidocaine (or an equivalent) or dehydrated (absolute) alcohol or both can eliminate the pain caused by ACNES. The injection is commonly done under ultrasonographic guidance (Figure 1).42
Complete pain relief may be achieved with a single injection, but some patients require up to five injections.
The adjuvant use of corticosteroids in ACNES to reduce inflammation is controversial.
If anesthetic injections bring only minimal pain relief or if the patient has nerve entrapment in a scar, then surgical neurectomy is an option.43 The procedure is performed under local anesthesia, as the patient’s response aids in identifying the specific nerve or nerves involved.
RETURNING TO THE PATIENT
After a long discussion with our patient about ACNES and the treatment options, she agrees to undergo nerve block in the hope of relieving her pain. She receives a 0.5-mL injection of 2% lidocaine subcutaneously, and within minutes she reports relief of pain. She cannot believe that with a simple injection her pain was relieved. We advise her to return if her pain recurs or if new symptoms arise.
KEEP ACNES IN MIND
ACNES is one of the most commonly misdiagnosed conditions of patients presenting to the outpatient clinic with acute or chronic abdominal pain. This is because the focus is directed to intra-abdominal causes. But if ACNES is kept in consideration from the beginning of the patient encounter, extensive testing, time, and patient anxiety may be reduced significantly. A simple physical examination and the Carnett test aid in raising suspicion of ACNES. If ACNES is confirmed, ultrasonographically guided local anesthetic injection is both diagnostic and therapeutic.
- American Gastroenterological Association Medical Position Statement: Guidelines On Intestinal Ischemia. Gastroenterology 2000; 118:951–953.
- Bobadilla JL. Mesenteric ischemia. Surg Clin North Am 2013; 93:925–940.
- Sung JJ, Kuipers EJ, El-Serag HB. Systematic review: the global incidence and prevalence of peptic ulcer disease. Aliment Pharmacol Ther 2009; 29:938–946.
- Najm WI. Peptic ulcer disease. Prim Care 2011; 38:383–394.
- Malfertheiner P, Chan FK, McColl KE. Peptic ulcer disease. Lancet 2009; 374:1449–1461.
- Chan FK, Leung WK. Peptic-ulcer disease. Lancet 2002; 360:933–941.
- Bright-Asare P, Habte T, Yirgou B, Benjamin J. Prostaglandins, H2-receptor antagonists and peptic ulcer disease. Drugs 1988; 35(suppl 3):1–9.
- Hudson N, Balsitis M, Everitt S, Hawkey CJ. Enhanced gastric mucosal leukotriene B4 synthesis in patients taking non-steroidal anti-inflammatory drugs. Gut 1993; 34:742–747.
- Ford-Hutchinson AW, Bray MA, Doig MV, Shipley ME, Smith MJ. Leukotriene B, a potent chemokinetic and aggregating substance released from polymorphonuclear leukocytes. Nature 1980; 286:264–265.
- Bokoch GM, Reed PW. Effect of various lipoxygenase metabolites of arachidonic acid on degranulation of polymorphonuclear leukocytes. J Biol Chem 1981; 256:5317–5320.
- Whittle BJ, Oren-Wolman N, Guth PH. Gastric vasoconstrictor actions of leukotriene C4, PGF2 alpha, and thromboxane mimetic U-46619 on rat submucosal microcirculation in vivo. Am J Physiol 1985; 248:G580–G586.
- Pihan G, Rogers C, Szabo S. Vascular injury in acute gastric mucosal damage. Mediatory role of leukotrienes. Dig Dis Sci 1988; 33:625–632.
- Ramakrishnan K, Salinas RC. Peptic ulcer disease. Am Fam Physician 2007; 76:1005–1012.
- Parmet S, Lynm C, Glass RM. JAMA patient page. Acute cholecystitis. JAMA 2003; 289:124.
- Roslyn JJ, DenBesten L, Thompson JE Jr, Silverman BF. Roles of lithogenic bile and cystic duct occlusion in the pathogenesis of acute cholecystitis. Am J Surg 1980; 140:126–130.
- Kaminski DL. Arachidonic acid metabolites in hepatobiliary physiology and disease. Gastroenterology 1989; 97:781–792.
- Jivegård L, Thornell E, Svanvik J. Pathophysiology of acute obstructive cholecystitis: implications for non-operative management. Br J Surg 1987; 74:1084–1086.
- Csendes A, Burdiles P, Maluenda F, Diaz JC, Csendes P, Mitru N. Simultaneous bacteriologic assessment of bile from gallbladder and common bile duct in control subjects and patients with gallstones and common duct stones. Arch Surg 1996; 131:389–394.
- Barie PS, Fischer E. Acute acalculous cholecystitis. J Am Coll Surg 1995; 180:232–244.
- Shapiro MJ, Luchtefeld WB, Kurzweil S, Kaminski DL, Durham RM, Mazuski JE. Acute acalculous cholecystitis in the critically ill. Am Surg 1994; 60:335–339.
- Savoca PE, Longo WE, Zucker KA, McMillen MM, Modlin IM. The increasing prevalence of acalculous cholecystitis in outpatients. Results of a 7-year study. Ann Surg 1990; 211:433–437.
- Gofrit O, Eid A, Pikarsky A, Lebensart PD, Pizov G, Rivkind A. Cholesterol embolisation causing chronic acalculous cholecystitis. Eur J Surg 1996; 162:243–245.
- McChesney JA, Northup PG, Bickston SJ. Acute acalculous cholecystitis associated with systemic sepsis and visceral arterial hypoperfusion: a case series and review of pathophysiology. Dig Dis Sci 2003; 48:1960–1967.
- Aeschlimann A, Kahn MF. Tietze’s syndrome: a critical review. Clin Exp Rheumatol 1990; 8:407–412.
- van Assen T, de Jager-Kievit JW, Scheltinga MR, Roumen RM. Chronic abdominal wall pain misdiagnosed as functional abdominal pain. J Am Board Fam Med 2013; 26:738–744.
- Akhnikh S, de Korte N, de Winter P. Anterior cutaneous nerve entrapment syndrome (ACNES): the forgotten diagnosis. Eur J Pediatr 2014; 173:445–449.
- Applegate WV. Abdominal cutaneous nerve entrapment syndrome (ACNES): a commonly overlooked cause of abdominal pain. Perm J 2002; 6:20–27.
- Grover M. UNC Center for Functional GI & Motility Disorders. Chronic abdominal wall pain: a missed diagnosis. www.med.unc.edu/ibs/files/educational-gi-handouts/Chronic%20Abdominal%20Pain.pdf. Accessed September 9, 2015.
- Greenbaum D, Dawson F, Watson R. Chronic abdominal wall pain (CAWP): a common but frequently overlooked disorder. Poster presented at the World Congress of Gastroenterology, Sydney, Australia, August 26–31, 1990.
- National Institutes of Health Consensus Development Conference Statement. Ovarian cancer: screening, treatment, and follow-up. Gynecol Oncol 1994; 55:S4–S14.
- Koonings PP, Campbell K, Mishell DR Jr, Grimes DA. Relative frequency of primary ovarian neoplasms: a 10-year review. Obstet Gynecol 1989; 74:921–926.
- Givens V, Mitchell GE, Harraway-Smith C, Reddy A, Maness DL. Diagnosis and management of adnexal masses. Am Fam Physician 2009; 80:815–820.
- Goff BA, Mandel L, Muntz HG, Melancon CH. Ovarian carcinoma diagnosis. Cancer 2000; 89:2068–2075.
- Friedman GD, Skilling JS, Udaltsova NV, Smith LH. Early symptoms of ovarian cancer: a case-control study without recall bias. Fam Pract 2005; 22:548–553.
- Stamatelou KK, Francis ME, Jones CA, Nyberg LM, Curhan GC. Time trends in reported prevalence of kidney stones in the United States: 1976-1994. Kidney Int 2003; 63:1817–1823.
- Worcester EM, Coe FL. Clinical practice. Calcium kidney stones. N Engl J Med 2010; 363:954–963.
- Miller NL, Lingeman JE. Management of kidney stones. BMJ 2007; 334:468–472.
- Lewis SR, Mahony PJ, Simpson J. Appendicitis. BMJ 2011; 343:d5976.
- Lamps LW. Infectious causes of appendicitis. Infect Dis Clin North Am 2010; 24:995–1018.
- Reif RM. Viral appendicitis. Hum Pathol 1981; 12:193–196.
- Akkary E, Panait L, Roberts K, Duffy A, Bell R. Sutureless laparoscopic ventral hernia repair in obese patients. JSLS 2011; 15:154–159.
- Boelens OB, Scheltinga MR, Houterman S, Roumen RM. Randomized clinical trial of trigger point infiltration with lidocaine to diagnose anterior cutaneous nerve entrapment syndrome. Br J Surg 2013; 100:217–221.
- Boelens OB, Scheltinga MR, Houterman S, Roumen RM. Management of anterior cutaneous nerve entrapment syndrome in a cohort of 139 patients. Ann Surg 2011; 254:1054–1058.
A 31-year-old woman presents to the office with a chief complaint of right mid-abdominal pain that began 1 day ago. She says she did not seek medical attention earlier because she had to be at work that morning and she thought the pain would resolve on its own.
She reports no fever, headache, anorexia, nausea, vomiting, malaise, loss of weight, melena, or changes in bowel habits. She describes the pain as sharp, localized to the right side, and radiating to the vulva upon sitting up. She denies any association of pain with current dietary habits or bowel function. She has no recollection of precipitating or alleviating factors, including the use of analgesics to reduce the pain.
On further discussion, she mentions that 1 year ago she began experiencing chronic abdominal pain, which she says is sometimes exacerbated by coughing, by standing for extended periods of time, and during menses, and is alleviated upon lying down.
She has regular menstrual periods, and her last one ended 7 days ago.
Her surgical history includes two uncomplicated cesarean deliveries. She does not use tobacco, alcohol, or illicit substances. She is not aware of any allergies to drugs or foods.
She appears to be in no acute distress and has been sitting quietly thus far. She seems to have positioned her hand on her abdomen over the corresponding area of pain.
On physical examination, vital signs are within normal limits, and she is alert and oriented to person, place, and time. Her sclerae are anicteric, and the pupils are equal, round, and reactive to light.
Cardiovascular and pulmonary examinations are also within normal limits. Examination of the abdomen elicits tenderness and guarding along the lateral border of the rectus abdominis muscle on the right side at the level of umbilicus, with no rebound tenderness or rigidity. The liver and spleen are not enlarged, and no abdominal mass is detected. No skin rash, joint swelling, or peripheral edema is noted. A neurologic examination is normal.
1. With the information provided, which of the following is least likely to be causing her symptoms?
- Chronic mesenteric ischemia
- Peptic ulcer
- Acute cholecystitis
- Slipping rib syndrome
CHRONIC MESENTERIC ISCHEMIA
Chronic mesenteric ischemia is the least likely diagnosis because the patient lacks risk factors for atherosclerosis and because she does not have postprandial pain, which is pathognomonic for chronic mesenteric ischemia. It is thought to be caused by a decrease in blood flow through the splanchnic vessels.1 Symptoms tend to arise after eating because of a postprandial increase in metabolic demands.1 These patients also often have atherosclerotic risk factors such as hypertension, hyperlipidemia, and smoking causing coronary artery disease, or a history of stroke.
The primary symptom is abdominal pain, most often described as achy, crampy, or spastic episodes of pain, usually occurring within 2 hours of eating.2 Weight loss is common, as patients can develop a fear of eating. Postprandial pain may also be associated with nausea, vomiting, and bloating.
Findings on clinical examination are usually less severe than the actual symptoms. Visceral duplex or multidetector computed tomography (CT) is an excellent tool to detect blood flow in potential stenotic vessels.2
PEPTIC ULCER DISEASE
Peptic ulcer disease is not a likely diagnosis in this patient because she has no history of taking nonsteroidal anti-inflammatory drugs (NSAIDs).
A study of US patients between 1997 and 2007 reported an annual incidence of peptic ulcer disease of 0.05% to 0.19% depending on the method of diagnosis.3 Peptic ulcer is thought to result from increased gastric acid secretion with a resultant inflammatory response, leading to erosion and ulceration.
The most common possible catalysts include Helicobacter pylori infection, NSAIDs, smoking, alcohol use, and hypersecretory states such as Zollinger-Ellison syndrome.4–6 Complications include internal bleeding, perforation causing peritonitis, and penetration to adjacent organs.
Pathophysiology
Peptic ulcer is the result of an increase in the normal level of gastric acid and a decrease in the protective ability of the gastric mucosa.7 Cytoprotection may be lost through a decrease in the products of arachidonic acid metabolism (eg, prostaglandins, which have a protective effect) or an increase in leukotriene B4 (LTB4), which has a damaging effect. Prostaglandins are thought not only to protect the normal gastric mucosa, but also to provide an antisecretory effect.
On the other hand, leukotrienes—specifically LTB4 and LTC4—are proinflammatory agents and can damage the gastric mucosa. NSAIDs enhance the production of leukotrienes through the 5-lipoxygenase pathway. The ability of LTB4 to cause degranulation and release of lysosomal enzymes may play a vital role in the inflammatory response to NSAIDs.8–10 LTC4 may promote gastric mucosal damage through a reduction of tissue perfusion resulting from the promotion of vascular stasis.8,11,12
Symptoms help differentiate ulcer type
The classic symptom is burning epigastric pain after meals. Pain that occurs immediately after meals is a classic symptom of gastric ulcer. Pain that occurs 2 to 3 hours after meals and that is relieved by food or antacids is a strong indicator of duodenal ulcer.13 Other symptoms include dyspepsia, bloating, distention, heartburn, and chest discomfort.13
Accurate diagnosis is vital in selecting the proper treatment. Diagnostic tests may include H pylori testing, upper-gastrointestinal endoscopy, and radiography with barium swallow.
CHOLECYSTITIS
In cholecystitis, the primary complaint is pain, usually in the right upper quadrant of the abdomen. Patients describe sudden, sharp, and intense pain that radiates to the back or shoulder. Patients may report pain after heavy meals, and some report nausea and vomiting. Cholecystitis is in the differential diagnosis of this patient because of the anatomic location of her pain.
The diagnosis is confirmed by imaging. Abdominal ultrasonography, technetium-99m hepatic iminodiacetic acid scanning, and CT are the most commonly used studies.14
Cholecystitis can be acute or chronic. Acute cholecystitis is categorized as calculous or acalculous. Calculous cholecystitis is multifactorial, but the primary cause is blockage of the cystic duct by gallstones.15 Other factors include irritants such as lysolecithin (released during bile stasis), which can trigger gallbladder inflammation,15–17 and infection.18
When the cystic duct is blocked, bile builds up inside the gallbladder, causing irritation and inflammation of the walls of the gallbladder.14
Acalculous cholecystitis, which resembles calculous cholecystitis but without the gallstones,19 accounts for 2% to 15% of all cases of acute cholecystitis.19,20 It has been observed in hospitalized critically ill patients, but it can also present in an outpatient setting, most often in elderly men with vascular disease.21 Causes include infection, trauma, and tumor obstruction, resulting in endothelial injury, gallbladder stasis, ischemia, and eventually necrosis.14,20,22,23
SLIPPING RIB SYNDROME
Slipping rib syndrome, also known as Tietze syndrome, is believed to be caused by hypermobile costal cartilage. The affected rib slips behind the rib above on contraction of the abdominal wall. This displacement increases the probability of costal nerve impingement and tissue inflammation producing unilateral, sharp, subcostal and upper-abdominal pain.
In this patient, slipping rib syndrome is a possible diagnosis because of the location of the pain and because the pain described by the patient is highly suggestive of neuropathic pain.
Slipping rib syndrome is diagnosed clinically by a “hooking” maneuver: the clinician hooks his or her fingers at the patient’s subcostal area, reproducing the pain by movement of the ribs anteriorly.24 When this test is performed in our patient the result is negative, ruling out slipping rib syndrome.
THE WORKUP CONTINUES
A complete blood cell count and comprehensive metabolic panel are within normal limits. Abdominal duplex ultrasonography reveals no celiac or mesenteric occlusions, thus ruling out chronic mesenteric ischemia.
Noncontrast CT shows no renal or ureteric stones and no evidence of bleeding in the urinary tract. CT with contrast shows no bowel distention, no evidence of hernia, and a normal appendix and ovaries.
2. After exclusion of the previous choices, which of the following is the most likely cause of her symptoms?
- Anterior cutaneous nerve entrapment syndrome (ACNES)
- Ovarian cyst
- Renal stones
- Appendicitis
- Ventral hernia
- Median arcuate ligament syndrome
ANTERIOR CUTANEOUS NERVE ENTRAPMENT SYNDROME
ACNES is the most likely diagnosis. A study published in 2013 indicated that many cases of functional abdominal pain may actually be undiagnosed cases of chronic abdominal wall pain such as ACNES.25 The condition, first described in 1972,26 is thought to be caused by thoracic cutaneous intercostal nerve entrapment between the abdominal muscles, causing pain at the point of entrapment.
The patient may present with pain that is either acute or chronic. Acute pain is localized more on the right side close to an old scar, or at the outer edge of the rectus abdominis muscle. The pain may vary from dull to burning to sharp; it can radiate horizontally in the upper half of the abdomen or obliquely in the lower half of the abdomen with movements such as twisting and sitting up.27
Despite the acute pain, patients are able to carry on daily functions. The pain may be alleviated by lying down.
The pain may be misdiagnosed as gynecologic or renal. In younger men, the pain may raise concern about hernia, and in older patients, cancer.27 Patients may complain of chronic intermittent pain, usually unilateral, and to a lesser extent bilateral.27
The anatomic location of the pain usually reflects the intercostal nerve involved. The pain is not related to eating or to bowel movements.25 Some patients report exacerbation upon coughing or standing, during menses, and with use of oral contraceptives.28,29 When inquiring about surgical history, it is common to find that the patient has had multiple abdominal surgical procedures.
On examination, the patient has nondistressing pain, with a hand often placed over the painful area.27 On firm palpation, a tender spot of less than 2 cm can be detected.
The diagnosis can be confirmed with a positive Carnett test. The patient lies supine on the examination table with the arms crossed over the chest, then elevates the head or the feet to tense the abdominal muscles.26,27 If doing so reproduces the pain (ie, a positive test), this increases the suspicion of ACNES; if the pain decreases or is not reproducible, an intra-abdominal cause is more likely.
If the pain is difficult to localize, the “pinch test” can be done by using the thumb and index finger to pinch and lift the skin of the abdomen, including the subcutaneous layer of fat, first on one side and then on the other. This helps determine the side with greater pain.27
OVARIAN CYSTS
Ovarian cysts are fluid-filled sacs on the surface of or within the ovary. They are often benign and require no intervention. However, 5% to 10% of US women with a suspicious ovarian mass undergo a surgical procedure, and 13% to 21% of these are found to have a malignancy.30,31
Ovarian cysts are usually painless unless complicated by rupture or bleeding. Patients who present with pain describe it as dull and aching and in the abdomen or pelvis. In rare cases, ovarian cysts can be large enough to cause pain from torsion. Other symptoms may include delayed menses and bleeding outside of the menstrual period.32–34
Ovarian cysts are thought to be caused by hormonal changes during the menstrual cycle. They can be detected during pelvic examination or during pelvic ultrasonography. Cysts that are primarily fluid-filled are generally benign and require no intervention. On the other hand, cysts composed of solid material require intervention.
Treatment depends on several factors, including size and type of cyst, the patient’s age, and whether torsion is present. Treatment can range from observation to medical or surgical management. Laparoscopic surgery is commonly used when surgical treatment is warranted.
RENAL STONES
From 10% to 15% of US adults develop a kidney stone at some time during their life.35 There is no single cause, but one factor that promotes stone formation is a greater amount of crystal-forming substances in the urine, such as calcium, oxalate, and uric acid.36 Most renal stones are calcium oxalate, uric acid, struvite, or cysteine.
Symptoms arise when the stone moves within the urinary tract. Patients present to the emergency room in severe distress, usually with flank pain that radiates to the lower abdomen or groin. The pain is episodic, fluctuates in intensity, and may present with dysuria, frequency, or urgency. It is also associated with nausea and vomiting.37
Renal stones are diagnosed through a series of laboratory and imaging studies. Imaging studies include plain radiography (which can miss small stones), renal sonography, and computed tomography without contrast.
APPENDICITIS
In the United States, the lifetime risk of developing appendicitis is 8.6% in men and 6.7% in women.38 Appendicitis is one of the most common reasons for emergency surgery.
Appendicitis is thought to result from obstruction by fecal matter blocking the opening of the appendix or from a viral infection (eg, with an adenovirus).39,40 The resulting bacterial growth can cause the appendix to become inflamed and purulent.
Patients typically present with umbilical or epigastric pain radiating to the right lower quadrant of the abdomen. Over time, the pain becomes sharper. Certain movements can exacerbate the pain, and lying down may alleviate it. Other symptoms are nausea, vomiting, loss of appetite, and low-grade fever.
Findings on the abdominal examination that help to confirm the diagnosis include rigidity and tenderness, classically localized to a point two-thirds of the way from the umbilicus to the anterior superior iliac spine. Rebound tenderness is usually present. Up to 25% of cases in some series presented atypically, with variable location and findings on physical examination (eg, bowel irregularities, indigestion, flatulence, generalized malaise). In addition to the physical examination, laboratory testing and imaging (ultrasonography, CT) may aid in confirming the diagnosis of appendicitis or any other cause of the pain.38
VENTRAL HERNIA
Ventral hernia is a bulging of abdominal organs or other tissues through a defect of the musculature of the abdominal wall. Ventral hernia is categorized by its location as epigastric, abdominal, or incisional. An open abdominal procedure is the cause in nearly 10% of cases41; the herniation occurs with weakening of the surgical scar.
Ventral hernia is usually detected on physical examination, and patients may present after noting a bulge in the abdominal wall. Symptoms vary. Some patients have no symptoms, while others have mild abdominal discomfort or severe abdominal pain as well as nausea and vomiting. Imaging with CT, ultrasonography, or magnetic resonance imaging helps confirm the diagnosis. Complications of ventral hernia include incarceration and bowel strangulation.
MEDIAN ARCUATE LIGAMENT SYNDROME
Median arcuate ligament syndrome is a challenging diagnosis and a very rare cause of abdominal pain. It is thought to be caused by celiac artery compression by fibroligamentous bands. Pain fluctuates with respiration and is greater during expiration.
Patients may present with recurrent episodes of crampy postprandial pain that cause them to avoid eating, resulting in weight loss. The pain may be associated with nausea, vomiting, and bloating.
The diagnosis is confirmed by duplex ultrasonography, angiography, or magnetic resonance angiography. Treatment is surgical division of the fibroligamentous band and crus, and this is often done laparascopically. In patients with severe persistent celiac artery stenosis, angioplasty and stenting may be considered.2
CASE CONTINUED
Before the physical examination, our patient identifies the location of her pain. A Carnett test is performed, as for ACNES: the patient is placed in the supine position and is instructed to cross both arms over her chest. In an effort to promote muscle tension, she is asked to elevate her head off the examination table, as if performing a mini sit-up, and as she does this, pressure is applied to the identified tender area. The pain is easily reproduced, further confirming involvement of the abdominal wall rather than the viscera. After this, electromyography shows abnormal findings. The patient is then referred to the pain management clinic for a diagnostic nerve block.
3. Which of the following is the first-line treatment of ACNES?
- Local injection of anesthetic
- Surgical neurectomy
LOCAL INJECTION OF ANESTHETIC
Local injection of anesthetic is the first-line treatment of ACNES.
Since ACNES is underdiagnosed, the patient may be less likely to be familiar with it. He or she should receive a detailed explanation of the condition and its management; this will help achieve a successful outcome.
Local anesthetic injection is used for both diagnosis and treatment; 2% lidocaine (or an equivalent) or dehydrated (absolute) alcohol or both can eliminate the pain caused by ACNES. The injection is commonly done under ultrasonographic guidance (Figure 1).42
Complete pain relief may be achieved with a single injection, but some patients require up to five injections.
The adjuvant use of corticosteroids in ACNES to reduce inflammation is controversial.
If anesthetic injections bring only minimal pain relief or if the patient has nerve entrapment in a scar, then surgical neurectomy is an option.43 The procedure is performed under local anesthesia, as the patient’s response aids in identifying the specific nerve or nerves involved.
RETURNING TO THE PATIENT
After a long discussion with our patient about ACNES and the treatment options, she agrees to undergo nerve block in the hope of relieving her pain. She receives a 0.5-mL injection of 2% lidocaine subcutaneously, and within minutes she reports relief of pain. She cannot believe that with a simple injection her pain was relieved. We advise her to return if her pain recurs or if new symptoms arise.
KEEP ACNES IN MIND
ACNES is one of the most commonly misdiagnosed conditions of patients presenting to the outpatient clinic with acute or chronic abdominal pain. This is because the focus is directed to intra-abdominal causes. But if ACNES is kept in consideration from the beginning of the patient encounter, extensive testing, time, and patient anxiety may be reduced significantly. A simple physical examination and the Carnett test aid in raising suspicion of ACNES. If ACNES is confirmed, ultrasonographically guided local anesthetic injection is both diagnostic and therapeutic.
A 31-year-old woman presents to the office with a chief complaint of right mid-abdominal pain that began 1 day ago. She says she did not seek medical attention earlier because she had to be at work that morning and she thought the pain would resolve on its own.
She reports no fever, headache, anorexia, nausea, vomiting, malaise, loss of weight, melena, or changes in bowel habits. She describes the pain as sharp, localized to the right side, and radiating to the vulva upon sitting up. She denies any association of pain with current dietary habits or bowel function. She has no recollection of precipitating or alleviating factors, including the use of analgesics to reduce the pain.
On further discussion, she mentions that 1 year ago she began experiencing chronic abdominal pain, which she says is sometimes exacerbated by coughing, by standing for extended periods of time, and during menses, and is alleviated upon lying down.
She has regular menstrual periods, and her last one ended 7 days ago.
Her surgical history includes two uncomplicated cesarean deliveries. She does not use tobacco, alcohol, or illicit substances. She is not aware of any allergies to drugs or foods.
She appears to be in no acute distress and has been sitting quietly thus far. She seems to have positioned her hand on her abdomen over the corresponding area of pain.
On physical examination, vital signs are within normal limits, and she is alert and oriented to person, place, and time. Her sclerae are anicteric, and the pupils are equal, round, and reactive to light.
Cardiovascular and pulmonary examinations are also within normal limits. Examination of the abdomen elicits tenderness and guarding along the lateral border of the rectus abdominis muscle on the right side at the level of umbilicus, with no rebound tenderness or rigidity. The liver and spleen are not enlarged, and no abdominal mass is detected. No skin rash, joint swelling, or peripheral edema is noted. A neurologic examination is normal.
1. With the information provided, which of the following is least likely to be causing her symptoms?
- Chronic mesenteric ischemia
- Peptic ulcer
- Acute cholecystitis
- Slipping rib syndrome
CHRONIC MESENTERIC ISCHEMIA
Chronic mesenteric ischemia is the least likely diagnosis because the patient lacks risk factors for atherosclerosis and because she does not have postprandial pain, which is pathognomonic for chronic mesenteric ischemia. It is thought to be caused by a decrease in blood flow through the splanchnic vessels.1 Symptoms tend to arise after eating because of a postprandial increase in metabolic demands.1 These patients also often have atherosclerotic risk factors such as hypertension, hyperlipidemia, and smoking causing coronary artery disease, or a history of stroke.
The primary symptom is abdominal pain, most often described as achy, crampy, or spastic episodes of pain, usually occurring within 2 hours of eating.2 Weight loss is common, as patients can develop a fear of eating. Postprandial pain may also be associated with nausea, vomiting, and bloating.
Findings on clinical examination are usually less severe than the actual symptoms. Visceral duplex or multidetector computed tomography (CT) is an excellent tool to detect blood flow in potential stenotic vessels.2
PEPTIC ULCER DISEASE
Peptic ulcer disease is not a likely diagnosis in this patient because she has no history of taking nonsteroidal anti-inflammatory drugs (NSAIDs).
A study of US patients between 1997 and 2007 reported an annual incidence of peptic ulcer disease of 0.05% to 0.19% depending on the method of diagnosis.3 Peptic ulcer is thought to result from increased gastric acid secretion with a resultant inflammatory response, leading to erosion and ulceration.
The most common possible catalysts include Helicobacter pylori infection, NSAIDs, smoking, alcohol use, and hypersecretory states such as Zollinger-Ellison syndrome.4–6 Complications include internal bleeding, perforation causing peritonitis, and penetration to adjacent organs.
Pathophysiology
Peptic ulcer is the result of an increase in the normal level of gastric acid and a decrease in the protective ability of the gastric mucosa.7 Cytoprotection may be lost through a decrease in the products of arachidonic acid metabolism (eg, prostaglandins, which have a protective effect) or an increase in leukotriene B4 (LTB4), which has a damaging effect. Prostaglandins are thought not only to protect the normal gastric mucosa, but also to provide an antisecretory effect.
On the other hand, leukotrienes—specifically LTB4 and LTC4—are proinflammatory agents and can damage the gastric mucosa. NSAIDs enhance the production of leukotrienes through the 5-lipoxygenase pathway. The ability of LTB4 to cause degranulation and release of lysosomal enzymes may play a vital role in the inflammatory response to NSAIDs.8–10 LTC4 may promote gastric mucosal damage through a reduction of tissue perfusion resulting from the promotion of vascular stasis.8,11,12
Symptoms help differentiate ulcer type
The classic symptom is burning epigastric pain after meals. Pain that occurs immediately after meals is a classic symptom of gastric ulcer. Pain that occurs 2 to 3 hours after meals and that is relieved by food or antacids is a strong indicator of duodenal ulcer.13 Other symptoms include dyspepsia, bloating, distention, heartburn, and chest discomfort.13
Accurate diagnosis is vital in selecting the proper treatment. Diagnostic tests may include H pylori testing, upper-gastrointestinal endoscopy, and radiography with barium swallow.
CHOLECYSTITIS
In cholecystitis, the primary complaint is pain, usually in the right upper quadrant of the abdomen. Patients describe sudden, sharp, and intense pain that radiates to the back or shoulder. Patients may report pain after heavy meals, and some report nausea and vomiting. Cholecystitis is in the differential diagnosis of this patient because of the anatomic location of her pain.
The diagnosis is confirmed by imaging. Abdominal ultrasonography, technetium-99m hepatic iminodiacetic acid scanning, and CT are the most commonly used studies.14
Cholecystitis can be acute or chronic. Acute cholecystitis is categorized as calculous or acalculous. Calculous cholecystitis is multifactorial, but the primary cause is blockage of the cystic duct by gallstones.15 Other factors include irritants such as lysolecithin (released during bile stasis), which can trigger gallbladder inflammation,15–17 and infection.18
When the cystic duct is blocked, bile builds up inside the gallbladder, causing irritation and inflammation of the walls of the gallbladder.14
Acalculous cholecystitis, which resembles calculous cholecystitis but without the gallstones,19 accounts for 2% to 15% of all cases of acute cholecystitis.19,20 It has been observed in hospitalized critically ill patients, but it can also present in an outpatient setting, most often in elderly men with vascular disease.21 Causes include infection, trauma, and tumor obstruction, resulting in endothelial injury, gallbladder stasis, ischemia, and eventually necrosis.14,20,22,23
SLIPPING RIB SYNDROME
Slipping rib syndrome, also known as Tietze syndrome, is believed to be caused by hypermobile costal cartilage. The affected rib slips behind the rib above on contraction of the abdominal wall. This displacement increases the probability of costal nerve impingement and tissue inflammation producing unilateral, sharp, subcostal and upper-abdominal pain.
In this patient, slipping rib syndrome is a possible diagnosis because of the location of the pain and because the pain described by the patient is highly suggestive of neuropathic pain.
Slipping rib syndrome is diagnosed clinically by a “hooking” maneuver: the clinician hooks his or her fingers at the patient’s subcostal area, reproducing the pain by movement of the ribs anteriorly.24 When this test is performed in our patient the result is negative, ruling out slipping rib syndrome.
THE WORKUP CONTINUES
A complete blood cell count and comprehensive metabolic panel are within normal limits. Abdominal duplex ultrasonography reveals no celiac or mesenteric occlusions, thus ruling out chronic mesenteric ischemia.
Noncontrast CT shows no renal or ureteric stones and no evidence of bleeding in the urinary tract. CT with contrast shows no bowel distention, no evidence of hernia, and a normal appendix and ovaries.
2. After exclusion of the previous choices, which of the following is the most likely cause of her symptoms?
- Anterior cutaneous nerve entrapment syndrome (ACNES)
- Ovarian cyst
- Renal stones
- Appendicitis
- Ventral hernia
- Median arcuate ligament syndrome
ANTERIOR CUTANEOUS NERVE ENTRAPMENT SYNDROME
ACNES is the most likely diagnosis. A study published in 2013 indicated that many cases of functional abdominal pain may actually be undiagnosed cases of chronic abdominal wall pain such as ACNES.25 The condition, first described in 1972,26 is thought to be caused by thoracic cutaneous intercostal nerve entrapment between the abdominal muscles, causing pain at the point of entrapment.
The patient may present with pain that is either acute or chronic. Acute pain is localized more on the right side close to an old scar, or at the outer edge of the rectus abdominis muscle. The pain may vary from dull to burning to sharp; it can radiate horizontally in the upper half of the abdomen or obliquely in the lower half of the abdomen with movements such as twisting and sitting up.27
Despite the acute pain, patients are able to carry on daily functions. The pain may be alleviated by lying down.
The pain may be misdiagnosed as gynecologic or renal. In younger men, the pain may raise concern about hernia, and in older patients, cancer.27 Patients may complain of chronic intermittent pain, usually unilateral, and to a lesser extent bilateral.27
The anatomic location of the pain usually reflects the intercostal nerve involved. The pain is not related to eating or to bowel movements.25 Some patients report exacerbation upon coughing or standing, during menses, and with use of oral contraceptives.28,29 When inquiring about surgical history, it is common to find that the patient has had multiple abdominal surgical procedures.
On examination, the patient has nondistressing pain, with a hand often placed over the painful area.27 On firm palpation, a tender spot of less than 2 cm can be detected.
The diagnosis can be confirmed with a positive Carnett test. The patient lies supine on the examination table with the arms crossed over the chest, then elevates the head or the feet to tense the abdominal muscles.26,27 If doing so reproduces the pain (ie, a positive test), this increases the suspicion of ACNES; if the pain decreases or is not reproducible, an intra-abdominal cause is more likely.
If the pain is difficult to localize, the “pinch test” can be done by using the thumb and index finger to pinch and lift the skin of the abdomen, including the subcutaneous layer of fat, first on one side and then on the other. This helps determine the side with greater pain.27
OVARIAN CYSTS
Ovarian cysts are fluid-filled sacs on the surface of or within the ovary. They are often benign and require no intervention. However, 5% to 10% of US women with a suspicious ovarian mass undergo a surgical procedure, and 13% to 21% of these are found to have a malignancy.30,31
Ovarian cysts are usually painless unless complicated by rupture or bleeding. Patients who present with pain describe it as dull and aching and in the abdomen or pelvis. In rare cases, ovarian cysts can be large enough to cause pain from torsion. Other symptoms may include delayed menses and bleeding outside of the menstrual period.32–34
Ovarian cysts are thought to be caused by hormonal changes during the menstrual cycle. They can be detected during pelvic examination or during pelvic ultrasonography. Cysts that are primarily fluid-filled are generally benign and require no intervention. On the other hand, cysts composed of solid material require intervention.
Treatment depends on several factors, including size and type of cyst, the patient’s age, and whether torsion is present. Treatment can range from observation to medical or surgical management. Laparoscopic surgery is commonly used when surgical treatment is warranted.
RENAL STONES
From 10% to 15% of US adults develop a kidney stone at some time during their life.35 There is no single cause, but one factor that promotes stone formation is a greater amount of crystal-forming substances in the urine, such as calcium, oxalate, and uric acid.36 Most renal stones are calcium oxalate, uric acid, struvite, or cysteine.
Symptoms arise when the stone moves within the urinary tract. Patients present to the emergency room in severe distress, usually with flank pain that radiates to the lower abdomen or groin. The pain is episodic, fluctuates in intensity, and may present with dysuria, frequency, or urgency. It is also associated with nausea and vomiting.37
Renal stones are diagnosed through a series of laboratory and imaging studies. Imaging studies include plain radiography (which can miss small stones), renal sonography, and computed tomography without contrast.
APPENDICITIS
In the United States, the lifetime risk of developing appendicitis is 8.6% in men and 6.7% in women.38 Appendicitis is one of the most common reasons for emergency surgery.
Appendicitis is thought to result from obstruction by fecal matter blocking the opening of the appendix or from a viral infection (eg, with an adenovirus).39,40 The resulting bacterial growth can cause the appendix to become inflamed and purulent.
Patients typically present with umbilical or epigastric pain radiating to the right lower quadrant of the abdomen. Over time, the pain becomes sharper. Certain movements can exacerbate the pain, and lying down may alleviate it. Other symptoms are nausea, vomiting, loss of appetite, and low-grade fever.
Findings on the abdominal examination that help to confirm the diagnosis include rigidity and tenderness, classically localized to a point two-thirds of the way from the umbilicus to the anterior superior iliac spine. Rebound tenderness is usually present. Up to 25% of cases in some series presented atypically, with variable location and findings on physical examination (eg, bowel irregularities, indigestion, flatulence, generalized malaise). In addition to the physical examination, laboratory testing and imaging (ultrasonography, CT) may aid in confirming the diagnosis of appendicitis or any other cause of the pain.38
VENTRAL HERNIA
Ventral hernia is a bulging of abdominal organs or other tissues through a defect of the musculature of the abdominal wall. Ventral hernia is categorized by its location as epigastric, abdominal, or incisional. An open abdominal procedure is the cause in nearly 10% of cases41; the herniation occurs with weakening of the surgical scar.
Ventral hernia is usually detected on physical examination, and patients may present after noting a bulge in the abdominal wall. Symptoms vary. Some patients have no symptoms, while others have mild abdominal discomfort or severe abdominal pain as well as nausea and vomiting. Imaging with CT, ultrasonography, or magnetic resonance imaging helps confirm the diagnosis. Complications of ventral hernia include incarceration and bowel strangulation.
MEDIAN ARCUATE LIGAMENT SYNDROME
Median arcuate ligament syndrome is a challenging diagnosis and a very rare cause of abdominal pain. It is thought to be caused by celiac artery compression by fibroligamentous bands. Pain fluctuates with respiration and is greater during expiration.
Patients may present with recurrent episodes of crampy postprandial pain that cause them to avoid eating, resulting in weight loss. The pain may be associated with nausea, vomiting, and bloating.
The diagnosis is confirmed by duplex ultrasonography, angiography, or magnetic resonance angiography. Treatment is surgical division of the fibroligamentous band and crus, and this is often done laparascopically. In patients with severe persistent celiac artery stenosis, angioplasty and stenting may be considered.2
CASE CONTINUED
Before the physical examination, our patient identifies the location of her pain. A Carnett test is performed, as for ACNES: the patient is placed in the supine position and is instructed to cross both arms over her chest. In an effort to promote muscle tension, she is asked to elevate her head off the examination table, as if performing a mini sit-up, and as she does this, pressure is applied to the identified tender area. The pain is easily reproduced, further confirming involvement of the abdominal wall rather than the viscera. After this, electromyography shows abnormal findings. The patient is then referred to the pain management clinic for a diagnostic nerve block.
3. Which of the following is the first-line treatment of ACNES?
- Local injection of anesthetic
- Surgical neurectomy
LOCAL INJECTION OF ANESTHETIC
Local injection of anesthetic is the first-line treatment of ACNES.
Since ACNES is underdiagnosed, the patient may be less likely to be familiar with it. He or she should receive a detailed explanation of the condition and its management; this will help achieve a successful outcome.
Local anesthetic injection is used for both diagnosis and treatment; 2% lidocaine (or an equivalent) or dehydrated (absolute) alcohol or both can eliminate the pain caused by ACNES. The injection is commonly done under ultrasonographic guidance (Figure 1).42
Complete pain relief may be achieved with a single injection, but some patients require up to five injections.
The adjuvant use of corticosteroids in ACNES to reduce inflammation is controversial.
If anesthetic injections bring only minimal pain relief or if the patient has nerve entrapment in a scar, then surgical neurectomy is an option.43 The procedure is performed under local anesthesia, as the patient’s response aids in identifying the specific nerve or nerves involved.
RETURNING TO THE PATIENT
After a long discussion with our patient about ACNES and the treatment options, she agrees to undergo nerve block in the hope of relieving her pain. She receives a 0.5-mL injection of 2% lidocaine subcutaneously, and within minutes she reports relief of pain. She cannot believe that with a simple injection her pain was relieved. We advise her to return if her pain recurs or if new symptoms arise.
KEEP ACNES IN MIND
ACNES is one of the most commonly misdiagnosed conditions of patients presenting to the outpatient clinic with acute or chronic abdominal pain. This is because the focus is directed to intra-abdominal causes. But if ACNES is kept in consideration from the beginning of the patient encounter, extensive testing, time, and patient anxiety may be reduced significantly. A simple physical examination and the Carnett test aid in raising suspicion of ACNES. If ACNES is confirmed, ultrasonographically guided local anesthetic injection is both diagnostic and therapeutic.
- American Gastroenterological Association Medical Position Statement: Guidelines On Intestinal Ischemia. Gastroenterology 2000; 118:951–953.
- Bobadilla JL. Mesenteric ischemia. Surg Clin North Am 2013; 93:925–940.
- Sung JJ, Kuipers EJ, El-Serag HB. Systematic review: the global incidence and prevalence of peptic ulcer disease. Aliment Pharmacol Ther 2009; 29:938–946.
- Najm WI. Peptic ulcer disease. Prim Care 2011; 38:383–394.
- Malfertheiner P, Chan FK, McColl KE. Peptic ulcer disease. Lancet 2009; 374:1449–1461.
- Chan FK, Leung WK. Peptic-ulcer disease. Lancet 2002; 360:933–941.
- Bright-Asare P, Habte T, Yirgou B, Benjamin J. Prostaglandins, H2-receptor antagonists and peptic ulcer disease. Drugs 1988; 35(suppl 3):1–9.
- Hudson N, Balsitis M, Everitt S, Hawkey CJ. Enhanced gastric mucosal leukotriene B4 synthesis in patients taking non-steroidal anti-inflammatory drugs. Gut 1993; 34:742–747.
- Ford-Hutchinson AW, Bray MA, Doig MV, Shipley ME, Smith MJ. Leukotriene B, a potent chemokinetic and aggregating substance released from polymorphonuclear leukocytes. Nature 1980; 286:264–265.
- Bokoch GM, Reed PW. Effect of various lipoxygenase metabolites of arachidonic acid on degranulation of polymorphonuclear leukocytes. J Biol Chem 1981; 256:5317–5320.
- Whittle BJ, Oren-Wolman N, Guth PH. Gastric vasoconstrictor actions of leukotriene C4, PGF2 alpha, and thromboxane mimetic U-46619 on rat submucosal microcirculation in vivo. Am J Physiol 1985; 248:G580–G586.
- Pihan G, Rogers C, Szabo S. Vascular injury in acute gastric mucosal damage. Mediatory role of leukotrienes. Dig Dis Sci 1988; 33:625–632.
- Ramakrishnan K, Salinas RC. Peptic ulcer disease. Am Fam Physician 2007; 76:1005–1012.
- Parmet S, Lynm C, Glass RM. JAMA patient page. Acute cholecystitis. JAMA 2003; 289:124.
- Roslyn JJ, DenBesten L, Thompson JE Jr, Silverman BF. Roles of lithogenic bile and cystic duct occlusion in the pathogenesis of acute cholecystitis. Am J Surg 1980; 140:126–130.
- Kaminski DL. Arachidonic acid metabolites in hepatobiliary physiology and disease. Gastroenterology 1989; 97:781–792.
- Jivegård L, Thornell E, Svanvik J. Pathophysiology of acute obstructive cholecystitis: implications for non-operative management. Br J Surg 1987; 74:1084–1086.
- Csendes A, Burdiles P, Maluenda F, Diaz JC, Csendes P, Mitru N. Simultaneous bacteriologic assessment of bile from gallbladder and common bile duct in control subjects and patients with gallstones and common duct stones. Arch Surg 1996; 131:389–394.
- Barie PS, Fischer E. Acute acalculous cholecystitis. J Am Coll Surg 1995; 180:232–244.
- Shapiro MJ, Luchtefeld WB, Kurzweil S, Kaminski DL, Durham RM, Mazuski JE. Acute acalculous cholecystitis in the critically ill. Am Surg 1994; 60:335–339.
- Savoca PE, Longo WE, Zucker KA, McMillen MM, Modlin IM. The increasing prevalence of acalculous cholecystitis in outpatients. Results of a 7-year study. Ann Surg 1990; 211:433–437.
- Gofrit O, Eid A, Pikarsky A, Lebensart PD, Pizov G, Rivkind A. Cholesterol embolisation causing chronic acalculous cholecystitis. Eur J Surg 1996; 162:243–245.
- McChesney JA, Northup PG, Bickston SJ. Acute acalculous cholecystitis associated with systemic sepsis and visceral arterial hypoperfusion: a case series and review of pathophysiology. Dig Dis Sci 2003; 48:1960–1967.
- Aeschlimann A, Kahn MF. Tietze’s syndrome: a critical review. Clin Exp Rheumatol 1990; 8:407–412.
- van Assen T, de Jager-Kievit JW, Scheltinga MR, Roumen RM. Chronic abdominal wall pain misdiagnosed as functional abdominal pain. J Am Board Fam Med 2013; 26:738–744.
- Akhnikh S, de Korte N, de Winter P. Anterior cutaneous nerve entrapment syndrome (ACNES): the forgotten diagnosis. Eur J Pediatr 2014; 173:445–449.
- Applegate WV. Abdominal cutaneous nerve entrapment syndrome (ACNES): a commonly overlooked cause of abdominal pain. Perm J 2002; 6:20–27.
- Grover M. UNC Center for Functional GI & Motility Disorders. Chronic abdominal wall pain: a missed diagnosis. www.med.unc.edu/ibs/files/educational-gi-handouts/Chronic%20Abdominal%20Pain.pdf. Accessed September 9, 2015.
- Greenbaum D, Dawson F, Watson R. Chronic abdominal wall pain (CAWP): a common but frequently overlooked disorder. Poster presented at the World Congress of Gastroenterology, Sydney, Australia, August 26–31, 1990.
- National Institutes of Health Consensus Development Conference Statement. Ovarian cancer: screening, treatment, and follow-up. Gynecol Oncol 1994; 55:S4–S14.
- Koonings PP, Campbell K, Mishell DR Jr, Grimes DA. Relative frequency of primary ovarian neoplasms: a 10-year review. Obstet Gynecol 1989; 74:921–926.
- Givens V, Mitchell GE, Harraway-Smith C, Reddy A, Maness DL. Diagnosis and management of adnexal masses. Am Fam Physician 2009; 80:815–820.
- Goff BA, Mandel L, Muntz HG, Melancon CH. Ovarian carcinoma diagnosis. Cancer 2000; 89:2068–2075.
- Friedman GD, Skilling JS, Udaltsova NV, Smith LH. Early symptoms of ovarian cancer: a case-control study without recall bias. Fam Pract 2005; 22:548–553.
- Stamatelou KK, Francis ME, Jones CA, Nyberg LM, Curhan GC. Time trends in reported prevalence of kidney stones in the United States: 1976-1994. Kidney Int 2003; 63:1817–1823.
- Worcester EM, Coe FL. Clinical practice. Calcium kidney stones. N Engl J Med 2010; 363:954–963.
- Miller NL, Lingeman JE. Management of kidney stones. BMJ 2007; 334:468–472.
- Lewis SR, Mahony PJ, Simpson J. Appendicitis. BMJ 2011; 343:d5976.
- Lamps LW. Infectious causes of appendicitis. Infect Dis Clin North Am 2010; 24:995–1018.
- Reif RM. Viral appendicitis. Hum Pathol 1981; 12:193–196.
- Akkary E, Panait L, Roberts K, Duffy A, Bell R. Sutureless laparoscopic ventral hernia repair in obese patients. JSLS 2011; 15:154–159.
- Boelens OB, Scheltinga MR, Houterman S, Roumen RM. Randomized clinical trial of trigger point infiltration with lidocaine to diagnose anterior cutaneous nerve entrapment syndrome. Br J Surg 2013; 100:217–221.
- Boelens OB, Scheltinga MR, Houterman S, Roumen RM. Management of anterior cutaneous nerve entrapment syndrome in a cohort of 139 patients. Ann Surg 2011; 254:1054–1058.
- American Gastroenterological Association Medical Position Statement: Guidelines On Intestinal Ischemia. Gastroenterology 2000; 118:951–953.
- Bobadilla JL. Mesenteric ischemia. Surg Clin North Am 2013; 93:925–940.
- Sung JJ, Kuipers EJ, El-Serag HB. Systematic review: the global incidence and prevalence of peptic ulcer disease. Aliment Pharmacol Ther 2009; 29:938–946.
- Najm WI. Peptic ulcer disease. Prim Care 2011; 38:383–394.
- Malfertheiner P, Chan FK, McColl KE. Peptic ulcer disease. Lancet 2009; 374:1449–1461.
- Chan FK, Leung WK. Peptic-ulcer disease. Lancet 2002; 360:933–941.
- Bright-Asare P, Habte T, Yirgou B, Benjamin J. Prostaglandins, H2-receptor antagonists and peptic ulcer disease. Drugs 1988; 35(suppl 3):1–9.
- Hudson N, Balsitis M, Everitt S, Hawkey CJ. Enhanced gastric mucosal leukotriene B4 synthesis in patients taking non-steroidal anti-inflammatory drugs. Gut 1993; 34:742–747.
- Ford-Hutchinson AW, Bray MA, Doig MV, Shipley ME, Smith MJ. Leukotriene B, a potent chemokinetic and aggregating substance released from polymorphonuclear leukocytes. Nature 1980; 286:264–265.
- Bokoch GM, Reed PW. Effect of various lipoxygenase metabolites of arachidonic acid on degranulation of polymorphonuclear leukocytes. J Biol Chem 1981; 256:5317–5320.
- Whittle BJ, Oren-Wolman N, Guth PH. Gastric vasoconstrictor actions of leukotriene C4, PGF2 alpha, and thromboxane mimetic U-46619 on rat submucosal microcirculation in vivo. Am J Physiol 1985; 248:G580–G586.
- Pihan G, Rogers C, Szabo S. Vascular injury in acute gastric mucosal damage. Mediatory role of leukotrienes. Dig Dis Sci 1988; 33:625–632.
- Ramakrishnan K, Salinas RC. Peptic ulcer disease. Am Fam Physician 2007; 76:1005–1012.
- Parmet S, Lynm C, Glass RM. JAMA patient page. Acute cholecystitis. JAMA 2003; 289:124.
- Roslyn JJ, DenBesten L, Thompson JE Jr, Silverman BF. Roles of lithogenic bile and cystic duct occlusion in the pathogenesis of acute cholecystitis. Am J Surg 1980; 140:126–130.
- Kaminski DL. Arachidonic acid metabolites in hepatobiliary physiology and disease. Gastroenterology 1989; 97:781–792.
- Jivegård L, Thornell E, Svanvik J. Pathophysiology of acute obstructive cholecystitis: implications for non-operative management. Br J Surg 1987; 74:1084–1086.
- Csendes A, Burdiles P, Maluenda F, Diaz JC, Csendes P, Mitru N. Simultaneous bacteriologic assessment of bile from gallbladder and common bile duct in control subjects and patients with gallstones and common duct stones. Arch Surg 1996; 131:389–394.
- Barie PS, Fischer E. Acute acalculous cholecystitis. J Am Coll Surg 1995; 180:232–244.
- Shapiro MJ, Luchtefeld WB, Kurzweil S, Kaminski DL, Durham RM, Mazuski JE. Acute acalculous cholecystitis in the critically ill. Am Surg 1994; 60:335–339.
- Savoca PE, Longo WE, Zucker KA, McMillen MM, Modlin IM. The increasing prevalence of acalculous cholecystitis in outpatients. Results of a 7-year study. Ann Surg 1990; 211:433–437.
- Gofrit O, Eid A, Pikarsky A, Lebensart PD, Pizov G, Rivkind A. Cholesterol embolisation causing chronic acalculous cholecystitis. Eur J Surg 1996; 162:243–245.
- McChesney JA, Northup PG, Bickston SJ. Acute acalculous cholecystitis associated with systemic sepsis and visceral arterial hypoperfusion: a case series and review of pathophysiology. Dig Dis Sci 2003; 48:1960–1967.
- Aeschlimann A, Kahn MF. Tietze’s syndrome: a critical review. Clin Exp Rheumatol 1990; 8:407–412.
- van Assen T, de Jager-Kievit JW, Scheltinga MR, Roumen RM. Chronic abdominal wall pain misdiagnosed as functional abdominal pain. J Am Board Fam Med 2013; 26:738–744.
- Akhnikh S, de Korte N, de Winter P. Anterior cutaneous nerve entrapment syndrome (ACNES): the forgotten diagnosis. Eur J Pediatr 2014; 173:445–449.
- Applegate WV. Abdominal cutaneous nerve entrapment syndrome (ACNES): a commonly overlooked cause of abdominal pain. Perm J 2002; 6:20–27.
- Grover M. UNC Center for Functional GI & Motility Disorders. Chronic abdominal wall pain: a missed diagnosis. www.med.unc.edu/ibs/files/educational-gi-handouts/Chronic%20Abdominal%20Pain.pdf. Accessed September 9, 2015.
- Greenbaum D, Dawson F, Watson R. Chronic abdominal wall pain (CAWP): a common but frequently overlooked disorder. Poster presented at the World Congress of Gastroenterology, Sydney, Australia, August 26–31, 1990.
- National Institutes of Health Consensus Development Conference Statement. Ovarian cancer: screening, treatment, and follow-up. Gynecol Oncol 1994; 55:S4–S14.
- Koonings PP, Campbell K, Mishell DR Jr, Grimes DA. Relative frequency of primary ovarian neoplasms: a 10-year review. Obstet Gynecol 1989; 74:921–926.
- Givens V, Mitchell GE, Harraway-Smith C, Reddy A, Maness DL. Diagnosis and management of adnexal masses. Am Fam Physician 2009; 80:815–820.
- Goff BA, Mandel L, Muntz HG, Melancon CH. Ovarian carcinoma diagnosis. Cancer 2000; 89:2068–2075.
- Friedman GD, Skilling JS, Udaltsova NV, Smith LH. Early symptoms of ovarian cancer: a case-control study without recall bias. Fam Pract 2005; 22:548–553.
- Stamatelou KK, Francis ME, Jones CA, Nyberg LM, Curhan GC. Time trends in reported prevalence of kidney stones in the United States: 1976-1994. Kidney Int 2003; 63:1817–1823.
- Worcester EM, Coe FL. Clinical practice. Calcium kidney stones. N Engl J Med 2010; 363:954–963.
- Miller NL, Lingeman JE. Management of kidney stones. BMJ 2007; 334:468–472.
- Lewis SR, Mahony PJ, Simpson J. Appendicitis. BMJ 2011; 343:d5976.
- Lamps LW. Infectious causes of appendicitis. Infect Dis Clin North Am 2010; 24:995–1018.
- Reif RM. Viral appendicitis. Hum Pathol 1981; 12:193–196.
- Akkary E, Panait L, Roberts K, Duffy A, Bell R. Sutureless laparoscopic ventral hernia repair in obese patients. JSLS 2011; 15:154–159.
- Boelens OB, Scheltinga MR, Houterman S, Roumen RM. Randomized clinical trial of trigger point infiltration with lidocaine to diagnose anterior cutaneous nerve entrapment syndrome. Br J Surg 2013; 100:217–221.
- Boelens OB, Scheltinga MR, Houterman S, Roumen RM. Management of anterior cutaneous nerve entrapment syndrome in a cohort of 139 patients. Ann Surg 2011; 254:1054–1058.
Eventration of the diaphragm presenting as spontaneous pneumothorax
A 25-year-old man with a 2-day history of upper respiratory tract infection presents to the emergency department with the sudden onset of right-sided back and chest pain and shortness of breath after a severe coughing fit.
He is morbidly obese, is a long-time smoker, and has had recurrent exacerbations of asthma with frequent upper respiratory tract infections. He has no history of recent trauma.
A review of systems reveals no significant impairment in exercise tolerance. He has been able to continue doing manual labor at his job as a railroad worker.
Radiography shows a large right pneumothorax and an elevated right diaphragm (Figure 1). Computed tomography (CT) (Figure 2) reveals a right anterior apical pneumothorax with hypoplastic lung and significant elevation of the right diaphragm with fat, bowel, and kidney within the right thorax. He is hemodynamically stable and shows no signs of bowel obstruction.
The physical examination is normal except for diminished breath sounds on the right side. He is diagnosed with congenital diaphragmatic hernia and spontaneous pneumothorax. A 10-F locking pigtail catheter is inserted under CT guidance, leading to complete resolution of the pneumothorax. He is discharged home the next day with a plan for elective repair of the hernia.
Two months later, he returns for scheduled right thoracotomy to repair the hernia. However, while preparing the chest cavity, the surgeon finds no diaphragmatic hernia and no intra-abdominal content—but rather, a severely elevated and thinned-out diaphragm with uninterrupted continuity. The diagnosis is changed to congenital diaphragmatic eventration, and plication of the diaphragm is performed with a series of interrupted, pledgeted polypropylene sutures.
CONGENITAL EVENTRATION OF THE DIAPHRAGM
Congenital diaphragmatic eventration is a rare developmental defect of the central, muscular portion of the diaphragm. The true prevalence is not known, but early reports identified this condition in less than 0.1% of adult.1
Symptomatic patients usually experience dyspnea secondary to ventilation-perfusion mismatch resulting from chronic atelectasis and lung hypoplasia, as well as impaired ventilation resulting from the limited caudal migration of the diaphragm.2,3 Increased susceptibility to recurrent upper respiratory tract infections and pneumonia is also a common feature.
Although rare, spontaneous pneumothorax can develop in patients such as ours, whose lengthy history of smoking and asthma predisposed him to the development of emphysema-like blebs and bullae and to subsequent rupture of blebs brought on by vigorous coughing that caused an involuntary Valsalva maneuver.4
As in our patient, distinguishing congenital diaphragmatic eventration from hernia preoperatively can be difficult with plain chest radiography. Spiral CT with multiplanar reconstruction or with magnetic resonance imaging can help establish the diagnosis.3 However, a severely attenuated diaphragm can be difficult to visualize on CT, as in our patient, leading to a presumptive diagnosis of diaphragmatic hernia. In such situations, the diagnosis of eventration can only be made intraoperatively.
Surgical repair is indicated only for patients with symptoms. Other potential causes of the symptoms should first be ruled out, however, including primary pulmonary disease, cardiac dysfunction, and morbid obesity.
- Chin EF, Lynn RB. Surgery of eventration of the diaphragm. J Thorac Surg 1956; 32:6–14.
- Ridyard JB, Stewart RM. Regional lung function in unilateral diaphragmatic paralysis. Thorax 1976; 31:438–442.
- Shen C, Che G. Congenital eventration of hemidiaphragm in an adult. Ann Thorac Surg 2012; 94:e137–e139.
- Porpodis K, Zarogoulidis P, Spyratos D, et al. Pneumothorax and asthma. J Thorac Dis 2014; 6(suppl 1):S152–S161.
A 25-year-old man with a 2-day history of upper respiratory tract infection presents to the emergency department with the sudden onset of right-sided back and chest pain and shortness of breath after a severe coughing fit.
He is morbidly obese, is a long-time smoker, and has had recurrent exacerbations of asthma with frequent upper respiratory tract infections. He has no history of recent trauma.
A review of systems reveals no significant impairment in exercise tolerance. He has been able to continue doing manual labor at his job as a railroad worker.
Radiography shows a large right pneumothorax and an elevated right diaphragm (Figure 1). Computed tomography (CT) (Figure 2) reveals a right anterior apical pneumothorax with hypoplastic lung and significant elevation of the right diaphragm with fat, bowel, and kidney within the right thorax. He is hemodynamically stable and shows no signs of bowel obstruction.
The physical examination is normal except for diminished breath sounds on the right side. He is diagnosed with congenital diaphragmatic hernia and spontaneous pneumothorax. A 10-F locking pigtail catheter is inserted under CT guidance, leading to complete resolution of the pneumothorax. He is discharged home the next day with a plan for elective repair of the hernia.
Two months later, he returns for scheduled right thoracotomy to repair the hernia. However, while preparing the chest cavity, the surgeon finds no diaphragmatic hernia and no intra-abdominal content—but rather, a severely elevated and thinned-out diaphragm with uninterrupted continuity. The diagnosis is changed to congenital diaphragmatic eventration, and plication of the diaphragm is performed with a series of interrupted, pledgeted polypropylene sutures.
CONGENITAL EVENTRATION OF THE DIAPHRAGM
Congenital diaphragmatic eventration is a rare developmental defect of the central, muscular portion of the diaphragm. The true prevalence is not known, but early reports identified this condition in less than 0.1% of adult.1
Symptomatic patients usually experience dyspnea secondary to ventilation-perfusion mismatch resulting from chronic atelectasis and lung hypoplasia, as well as impaired ventilation resulting from the limited caudal migration of the diaphragm.2,3 Increased susceptibility to recurrent upper respiratory tract infections and pneumonia is also a common feature.
Although rare, spontaneous pneumothorax can develop in patients such as ours, whose lengthy history of smoking and asthma predisposed him to the development of emphysema-like blebs and bullae and to subsequent rupture of blebs brought on by vigorous coughing that caused an involuntary Valsalva maneuver.4
As in our patient, distinguishing congenital diaphragmatic eventration from hernia preoperatively can be difficult with plain chest radiography. Spiral CT with multiplanar reconstruction or with magnetic resonance imaging can help establish the diagnosis.3 However, a severely attenuated diaphragm can be difficult to visualize on CT, as in our patient, leading to a presumptive diagnosis of diaphragmatic hernia. In such situations, the diagnosis of eventration can only be made intraoperatively.
Surgical repair is indicated only for patients with symptoms. Other potential causes of the symptoms should first be ruled out, however, including primary pulmonary disease, cardiac dysfunction, and morbid obesity.
A 25-year-old man with a 2-day history of upper respiratory tract infection presents to the emergency department with the sudden onset of right-sided back and chest pain and shortness of breath after a severe coughing fit.
He is morbidly obese, is a long-time smoker, and has had recurrent exacerbations of asthma with frequent upper respiratory tract infections. He has no history of recent trauma.
A review of systems reveals no significant impairment in exercise tolerance. He has been able to continue doing manual labor at his job as a railroad worker.
Radiography shows a large right pneumothorax and an elevated right diaphragm (Figure 1). Computed tomography (CT) (Figure 2) reveals a right anterior apical pneumothorax with hypoplastic lung and significant elevation of the right diaphragm with fat, bowel, and kidney within the right thorax. He is hemodynamically stable and shows no signs of bowel obstruction.
The physical examination is normal except for diminished breath sounds on the right side. He is diagnosed with congenital diaphragmatic hernia and spontaneous pneumothorax. A 10-F locking pigtail catheter is inserted under CT guidance, leading to complete resolution of the pneumothorax. He is discharged home the next day with a plan for elective repair of the hernia.
Two months later, he returns for scheduled right thoracotomy to repair the hernia. However, while preparing the chest cavity, the surgeon finds no diaphragmatic hernia and no intra-abdominal content—but rather, a severely elevated and thinned-out diaphragm with uninterrupted continuity. The diagnosis is changed to congenital diaphragmatic eventration, and plication of the diaphragm is performed with a series of interrupted, pledgeted polypropylene sutures.
CONGENITAL EVENTRATION OF THE DIAPHRAGM
Congenital diaphragmatic eventration is a rare developmental defect of the central, muscular portion of the diaphragm. The true prevalence is not known, but early reports identified this condition in less than 0.1% of adult.1
Symptomatic patients usually experience dyspnea secondary to ventilation-perfusion mismatch resulting from chronic atelectasis and lung hypoplasia, as well as impaired ventilation resulting from the limited caudal migration of the diaphragm.2,3 Increased susceptibility to recurrent upper respiratory tract infections and pneumonia is also a common feature.
Although rare, spontaneous pneumothorax can develop in patients such as ours, whose lengthy history of smoking and asthma predisposed him to the development of emphysema-like blebs and bullae and to subsequent rupture of blebs brought on by vigorous coughing that caused an involuntary Valsalva maneuver.4
As in our patient, distinguishing congenital diaphragmatic eventration from hernia preoperatively can be difficult with plain chest radiography. Spiral CT with multiplanar reconstruction or with magnetic resonance imaging can help establish the diagnosis.3 However, a severely attenuated diaphragm can be difficult to visualize on CT, as in our patient, leading to a presumptive diagnosis of diaphragmatic hernia. In such situations, the diagnosis of eventration can only be made intraoperatively.
Surgical repair is indicated only for patients with symptoms. Other potential causes of the symptoms should first be ruled out, however, including primary pulmonary disease, cardiac dysfunction, and morbid obesity.
- Chin EF, Lynn RB. Surgery of eventration of the diaphragm. J Thorac Surg 1956; 32:6–14.
- Ridyard JB, Stewart RM. Regional lung function in unilateral diaphragmatic paralysis. Thorax 1976; 31:438–442.
- Shen C, Che G. Congenital eventration of hemidiaphragm in an adult. Ann Thorac Surg 2012; 94:e137–e139.
- Porpodis K, Zarogoulidis P, Spyratos D, et al. Pneumothorax and asthma. J Thorac Dis 2014; 6(suppl 1):S152–S161.
- Chin EF, Lynn RB. Surgery of eventration of the diaphragm. J Thorac Surg 1956; 32:6–14.
- Ridyard JB, Stewart RM. Regional lung function in unilateral diaphragmatic paralysis. Thorax 1976; 31:438–442.
- Shen C, Che G. Congenital eventration of hemidiaphragm in an adult. Ann Thorac Surg 2012; 94:e137–e139.
- Porpodis K, Zarogoulidis P, Spyratos D, et al. Pneumothorax and asthma. J Thorac Dis 2014; 6(suppl 1):S152–S161.